Sie sind auf Seite 1von 141

‫بسم هللا الرحمن الرحيم‬

‫هذه مذكرة األطفال ‪ ,‬و تشمل األسئلة الموجودة في كل من ‪:‬‬


‫‪SMLE13‬‬
‫مذكرة الحرس الوطني‬
‫‪Double check‬‬
‫المذكرة البرتقالية ‪Ppo2‬‬
‫تجميعات ديسمبر‪.‬‬

‫قمت بتنقيح األسئلة و إضافة بعض المعلومات عليها ‪ ,‬والتأكد من ص ّحتها ‪ ,‬و إضافة بعض المالحظات‬
‫تحريت الدقة الكاملة في األسئلة ‪ ,‬و أرجو العذر عند وجود أي خطأ‬
‫المهمة التي لها عالقة باألسئلة ‪ ,‬و قد ّ‬
‫‪ ,‬فهذا مجهود بشري يحتمل الخطأ و الصواب ‪.‬‬
‫ي ولمن أحبّ ‪.‬‬
‫دعواتكم لي ولوالد ّ‬
Pediatrics
1. What is the injection that is routinely given to newborn to inhibit hemorrhage: (common Q)
A. Vitamin K
B. Vitamin C
C. Vitamin D
D. Vitamin E

Answer: A
Note : if baby born in home and comes with bleeding its due to ( vit.K ) deficiency
Note : bleeding from umbilical after clamping after birth due to ( factor XIII ) deficiency
Note : bleeding after circumcision due to ( factor VIII ) deficiency : Hemophilia A

2. Cellulitis occurring in the face in young children (6-24 months) and associated with fever
and purple skin discoloration is MOST often caused by : (common Q)
A. Group A beta hemolytic streptococci
B. Haemophilus influenzae type B
C. Streptococcus pneumoniae
D. Staphylococcus aureus

Answer: B or C ( mostly B because of purple discoloration and the age )


Note : for baby less than 6 months most common cause of cellulitis is : GABH
Age between 6 and 24 months most common H.influnza and S.pneumonia
Read more : https://emedicine.medscape.com/article/214222-overview#a4

3. Breastfeeding mother with HCV treated by interferon for more than one year, what is the risk of
breastfeeding on the infant?
A. Cracked Nipple
B. Mother with anemia
C. Infant complain of oral candidiasis
D. Not follow up of infant immunization

Answer: A
Note: HCV has not been shown to be transmitted through breast milk, although HCV-positive mothers should consider
abstaining from breastfeeding if their nipples are cracked or bleeding.

4. Milestone, baby pulls himself to stand, crawls without difficulty, which age is he?
A. 8 months
B. 10 months
C. 12 months
D. 14 months

Answer: A
Note: For all Developmental Qs see Nelson book

5. 2 years old presented with fever for one month with the pic, lab shows Pancytopenia, what is the
cause?
A. Leishmania
B. Leukemia
C. Malaria
D. Brucellosis
Answer: B
Note : Brucellosis, malaria and leishmanial also cause pancytopenia, but it seems the pic shows sign of leukemia.
See peripheral slide in Google image for all above choices
6. A boy dreamed of a bad dream and he woke up crying and feared but he does not remember any of
the dream which stage of sleep he was in:

A. 1
B. 2
C. 3
D. 4

Answer: D
Note : Night terrors happen during deep non-REM sleep stage 3 and 4 more in stage 4. Unlike nightmares (which
occur during REM sleep

7. Which of the following will you find late in a croup patient?

A. Nasal flaring
B. Expiratory wheezing
C. Tachypnea
D. Lip cyanosed

Answer: D
Note : Late and warning sign in croup is ( Lip cyanosis )
Note : best way to dx croup is ( clinical )

8. What is the condition in which the baby will have bone age more than chronological age?

A. Hypothyroidism
B. Chronic kidney disease
C. Congenital adrenal hyperplasia
D. Rickets

Answer: C
Note : bone age more than chronological age in these conditions : ( congenital Adrenal Hyperplasia ,
Hyperthyroidism , early Puberty (precocious Puberty ) , Premature Adrenarche , Gigantism )

9. Child with croup. What will you find in chest auscultation?


A. Wheezing
B. Crepitation
C. Gasp
D. Silent chest ( not sure about this option)

Answer: A , more accurate if ( stridor ) was in choices

10. Child presented with heave on examination, ECG shows RBBB


On ECHO: right ventricle motion abnormality and right
ventricle
Hypertrophy , Whats the most likely cause ?
A. Mitral prolapse
B. ASD
C. VSD
D. Coarctation of aorta

Answer: C
Note : RBBB caused by ( large ASD and VSD ) but R.t ventricle hypertrophy presents in VSD
11. One month boy came for vaccine. His older sister (6years old) had renal transplant and now she is
on immunosuppressive medication. Which vaccine is contraindicated for boy ?
A. MMR
B. Oral polio
C. Salk polio
D. Influenza

Answer: B
Note; Due to the risk of spread OPV for immunocompromised girl , OPV (is a live vaccine).

12. 10 days neonate present with lethargy , irritability , fever , signs of meningitis which organism is
causative :
A. Listeria monocytogenes
B. Streps pneumonia
C. Staph aureus
D. N-menningitidis

Answer; A
Note; most common cause of early sepsis( less than 7 days) is ( GBS then Listeria ) for Late sepsis after 7 days or
hospitalized pt sepsis is ( pseudomonas ) , so from these choices mostly A and Abx tt for early sepsis and Listeria
monocytogenes is
( Ampicillin )

13. Bacterial meningitis in 14 month child , Gram positive cocci, what is the
management?
A-amoxicillin
B-amoxicillin and gentamicin
C-ceftriaxone and vancomycin
D-vancomycin

Answer : C
Note : gram + cocci ( s. pneumonia )

14. Cystic Fibrosis gene mutation location : ( common Q)

A. Long arm 7 Q
B. Long arm 7 P
C. Short arm 14 Q
D. Short arm 14 Q

Answer: A
15. Mother brought her 2 years old child to the ER with history of upper respiratory tract infection for
the last 3 days with mild respiratory distress. This evening the child started to have hard barking
cough with respiratory distress. On examination: RR 40/min, associated with nasal flaring,
suprasternal & intercostal recessions. What is the most likely diagnosis? (Common Q)
A. Viral pneumonia
B. Bacterial pneumonia
C. Bronchiolitis
D. Laryngotracheobronchitis (croup)

Answer: D
Note : Barking cough , stridor , hoarseness of voice are classical signs for croup , best way to dx ( clinically )

16. Child known case of DM 1, lost his consciousness at school. The last insulin injection is unknown ,
whats the tt ? (common Q)
A. Take him to the hospital
B. Iv normal saline
C. Im glucagon
D. Insulin

Answer: C
Note : Loss of consciousness in diabetic pt think about hypoglycemia , tt : Dextrose fluid then Glucagon

17. A child with the history of repeated infections, failure to thrive and anemia. His older brother also
has same condition. What is the most likely diagnosis?
A. Nutritional anemia
B. Leukemia
C. Lymphoma
D. Hemoglobinopathy

Answer: D
Repeated infection, failure to thrive, anemia and family history of same condition make hemoglobinopathy the
best answer.

18. During delivery, when the doctor cut the umbilical cord, bleeding doesn’t stop. Which of the following
factors is deficient in this case?
A. Factor x
B. Factor xi
C. Factor xii
D. Factor xiii

Answer: D
The bleeding diathesis in inherited factor xiii (fxiii) deficiency is severe in most patients. Bleeding from the stump of the
umbilical cord within the first days to weeks of life is a characteristic sign.

19. When does girls get puberty as compared to boys? (common Q)


A. 1 to 2 yrs before
B. 2 to 3 yrs before
C. Same time when boys do
D. After boys
Answer: A
Note : Average age for girls: 10 and a half years old.
Average age for boys: 11 and a half to 12 years old.

20. 6 year old child with fever, malaise, lymphadenopathy and hepatosplenomegaly. Shown is the
CBC and bone marrow aspiration slide. What is the most likely diagnosis?

30,00
Wbc 0 (4,500-11.000)

Hgb 9

50,00
Platelet 0 (150,000-400,000)

Peripheral blast cells 25%

A. Sickle sequestration.
B. Leukemia.
C. Malaria.
D. Leishmaniasis.

Answer: B
The diagnosis of acute lymphoblastic leukemia (all) is made when at least 30% lymphoblasts (french-american-
british [fab] classification) or 20% lymphoblasts (world health organization [who] classification) are present in the
bone marrow and/or peripheral blood.
Approximately 15% of patients with all have a t(9;22) translocation (i.e, philadelphia [ph]
chromosome).

21. Child that throws a ball at you and draws a straight line and stacks “few” cubes on each other (they
didn’t mention the number of cubes). What is the age?
A. 12 months
B. 14 months
C. 18 months
D. 24 months

Answer: D
Note : Drawing straight lines begins at the age of 2 years. Reference: illustrated textbook and nelson 7 th e
22. A child that can raise his head slightly when prone and smiles. He turns his head 180 degrees and
has head lag when you pull him to sit. How many old is he?
A. 4 weeks
B. 8 weeks
C. 12 weeks
D. 16 weeks

Answer: B
Note : social smile begins at birth or 2 months , head lag disappears at 3 months

23. A child with rheumatic heart disease allergic to penicillin. What prophylaxis should be given before a
procedure?
A. Iv amoxicillin
B. Iv vancomycin + iv gentamicin
C. Oral vancomycin + gentamicin
D. Oral amoxicillin

Answer: B (depends on the type of procedure and the ability to tolerate oral medications)
Note: the best prophylaxis is (2g Amoxicillin ) if there is no allergy then Azithromycin or Clarithromycin or gentamicin
So the best answer from these choices is B
24. DTaP vaccine is against what?
A. Whooping cough, tetanus, diphtheria
B. Pertussis, tetanus, measles
C. Rubella, tetanus diphtheria
D. Whooping cough, rubella, diphtheria

Answer: A

25. 11 years old patient with rheumatic fever and cardiac involvement. For how long he will require
prophylaxis?
A. 5 years
B. 6 years
C. 10 years
D. 15 years

Answer: C
Note : Rheumatic heart disease without cardiac involvement : 5 years or till age 21 years ( which longer )
Rheumatic heart disease with cardiac involvement ( carditis ) : 10 years or till age 21 years ( which longer )
Rheumatic heart disease with cardiac involvement ( carditis ) and residual heart disease ( valve disease ) :
at least 10 years or till 40 years ( which longer ) or for life .

26. Which of the following congenital heart disease is the least associated with infective endocarditis?
(common Q)
A. ASD
B. VSD
C. PDA
D. Pulmonary stenosis

Answer: A

27. A boy came to your clinic with yellow discoloration of the eyes noticed 3 days back and
hepatomegaly. His liver enzymes are increased. What is the diagnosis? (common Q )
A. Hepatitis a
B. Hepatitis b
C. Hepatitis c
D. Hepatitis d

Answer: A
Hepatitis a the only type of hepatitis that reveal tender hepatomegaly. Also its acute presentation and more in
pediatrics

28. What is the triple anti toxoid vaccine ?


A. Tetanus, diphtheria, whooping cough
B. Tetanus, diphtheria, tb
C. Diphtheria, pertussis, colorectal ca
D. Diphtheria, tetanus, rabies.

Answer: A

29. Which of the following is most typically seen in 4 years old baby?
A. Print name.
B. Stand on one foot briefly.
C. Copy triangle and square.
D. Toilet trained.

Answer: d
Note : also at 4 years ( tell story ) , important to see Nelson developmental section
30. Child can set without support, cruises around furniture, uses chair to stand, say dada, crawl stairs.
What is the age of this child ? (common Q )
A. 8 months.
B. 10 months.
C. 12 months.
D. 15 months.

Answer: B
Note : important to see Nelson developmental section

31. ( long scenario) child have 1mm defect in muscular atrial septum. What you will do?
A. Surgical repair.
B. Catheter repair.
C. Reduce after load.
D. Watchful waiting.

Answer: D
Note : 80-100% spontaneous closure rate if asd diameter <8 mm. Reference: toronto notes.

32. A baby 8 month old breastfeed for 6 month normally. He devolved vomiting and jaundice after fruit
juice. What component in the fruit juice the baby is allergic to?
A. Glucose
B. Fructose
C. Sucrose
D. Galactose

Answer: B
Note : Hereditary fructose intolerance due to deficiency of fructose-1,6-biphosphate aldolase b in the liver, the hint in
Q is fruit juice
Reference: http://reference.medscape.com/article/944548-overview

33. What is the most common cause of acute bronchiolitis? ( common Q )


A. Respiratory syncytial virus (RSV)
B. Adenovirus
C. Parainfluenza
D. Mycoplasma pneumonia.

Answer: A
Reference: Nelson book

34. To which part of body RSV can go other than Lung ?


A. Spleen.
B. Bladder.
C. Kidney.
D. Liver.

Answer: mostly D ( not sure 100%)


Note : RSV may be recovered from extra pulmonary tissues, such as liver, cerebrospinal fluid, or pericardial fluid
Reference: uptodate

35. Breastfeeding after delivery should start:


A. Immediately
B. 6 hrs
C. 8 hrs
D. 24 hrs

Answer: A
36. 26 years old female G1P1 brought her two weeks old baby who cries a lot. He is on breastfeeding
since birth, he stop crying at night when she gave him formula milk. On examination the baby looks
normal except for increase gurgle sound. What is the most likely diagnosis?
A. Paralytic ileus.
B. Lactose malabsorption.
C. Increase bowel gases.
D. Breast milk jaundice.

Answer: B OR C mostly C
Read more about Lactose intolerance and bowel gases

37. 5 years old girl with uncomplicated cystitis. What is the management?
A. Oral amoxicillin
B. Iv cephalosporin
C. Im ceftriaxone
D. Ciprofloxacine

Answer: A
Note : A 4-day course of an oral antibiotic agent is recommended for the treatment of cystitis. A systematic review of
treatments for cystitis in children showed no difference in efficacy with 7-14 days of therapy compared with 2-4 days.
Single-dose or single-day therapy is not recommended in children with cystitis. If the clinical response is not
satisfactory after 2-3 days, alter therapy on the basis of antibiotic susceptibility.

Reference : https://emedicine.medscape.com/article/969643-treatment#d12

38. 7 years old girl was brought by her mother, she developed pubic hair and her height 70th percentile
and weight 50th percentile. On examination there are no signs of puberty except pubic hair. Abdominal,
chest, cardiac and renal examination were normal. What is the most likely diagnosis?
A. Congenital adrenal hyperplasia
B. 45x (turner syndrome).
C. Premature adrenarche.
D. Normal puberty.

Answer: C
Note : Premature adrenarche: is when hair changes begin early, before age 8 for girls and age 9 for boys.
Precocious puberty : refers to the appearance of physical and hormonal signs of pubertal
development at
earlier age than is considered normal , before age 8 for girls and age 9 for boys.

Reference :http://www.medscape.com/viewarticle/759350_3

39. A child with normochromic normocytic anemia + splenomegaly. Blood smear was attached showing
clear spherocytosis. Which of the following will be abnormal?
A. Platelets
B. Reticulocytes
C. WBC
D. MCV

Answer: B
Note : case of ( Hereditary Spherocytosis ) which is part of hemolytic Anemia , we will find Reticulocytosis

40. What is the most common intraabdominal tumor in children? ( common Q )


A. Hepatoma
B. Rhabdomyosarcoma
C. Ewing tumor
D. Wilms tumor ( Nephroblastoma )

Answer: D
Note : in exam there is no ( neuroblastoma ) in choices , so the answer is D
41. What is the single most important risk factor for cerebral palsy?
A. Prematurity
B. Birth weight less than 1.5 kg
C. Prenatal asphyxia
D. Genetic mutations.

Answer: A
“prematurity is the single most important risk factor for cerebral palsy”

Reference:https://books.google.com.sa/books?Id=bonllhartfac&pg=ra5-pt6753&lpg=ra5-
pt6753&dq=#v=onepage&q&f=false

42. Baby with recurrent infection TB , aspergillosis all type of infection with history other brother death
at 3 year
what Vaccine you will give him ?
A. Influenza Vaccine
B. BCG Vaccine
C. Varicella Vaccine
D. Polio Vaccine

Answer: A
Note : the baby complains of immunodeficiency and contraindication to give Live attenuated Vaccines , also Influenza
Vaccine decrease the risk of infections .

43. A child with hepatosplenomegaly, rcurrent infections. Brother died at 3 years with septic shock. How to
give vaccination?
A. Give all.
B. Don’t give until 3 years.
C. Don’t give live vaccines.
D. Don’t give killed vaccines.

Answer : C
Note : the baby and his brother complain of immunodeficiency disorder , its contraindication to give Live attenuated
Vaccine for immunocompromised pts

44. Child can know color but with difficulty in making square ?
A. 2 years
B. 3 years
C. 4 years
D. 5 years

Answer: C , the most appropriate answer


Note : know color at 4 years

This Table in Nelson :

Table 11-1 Emerging Patterns of Behavior from 1-5 Yr


of Age :

15 MO
Motor: Walks alone; crawls up stairs
Adaptive: Makes tower of 3 cubes; makes a line with crayon;
inserts raisin in bottle
Language: Jargon; follows simple commands; may name a familiar
object (e.g., ball); responds to his/her name
Social: Indicates some desires or needs by pointing; hugs
Parents

18 MO
Motor: Runs stiffly; sits on small chair; walks up stairs with 1
hand held; explores drawers and wastebaskets
Adaptive: Makes tower of 4 cubes; imitates scribbling; imitates
vertical stroke; dumps raisin from bottle
Language: 10 words (average); names pictures; identifies 1 or
more parts of body
Social: Feeds self; seeks help when in trouble; may complain
when wet or soiled; kisses parent with pucker

24 MO
Motor: Runs well, walks up and down stairs, 1 step at a time;
opens doors; climbs on furniture; jumps
Adaptive: Makes tower of 7 cubes (6 at 21 mo); scribbles in
circular pattern; imitates horizontal stroke; folds
paper once imitatively
Language: Puts 3 words together (subject, verb, object)
Social: Handles spoon well; often tells about immediate
experiences; helps to undress; listens to stories when
shown pictures

30 MO
Motor: Goes up stairs alternating feet
Adaptive: Makes tower of 9 cubes; makes vertical and horizontal
strokes, but generally will not join them to make
cross; imitates circular stroke, forming closed figure
Language: Refers to self by pronoun “I”; knows full name
Social: Helps put things away; pretends in play

36 MO
Motor: Rides tricycle; stands momentarily on 1 foot
Adaptive: Makes tower of 10 cubes; imitates construction of
“bridge” of 3 cubes; copies circle; imitates cross
Language: Knows age and sex; counts 3 objects correctly; repeats
3 numbers or a sentence of 6 syllables; most of
speech intelligible to strangers
Social: Plays simple games (in “parallel” with other children);
helps in dressing (unbuttons clothing and puts on
shoes); washes hands

48 MO
Motor: Hops on 1 foot; throws ball overhand; uses scissors to
cut out pictures; climbs well
Adaptive: Copies bridge from model; imitates construction of
“gate” of 5 cubes; copies cross and square; draws
man with 2-4 parts besides head; identifies longer of
2 lines
Language: Counts 4 pennies accurately; tells story
Social: Plays with several children, with beginning of social
interaction and role-playing; goes to toilet alone

60 MO
Motor: Skips
Adaptive: Draws triangle from copy; names heavier of 2 weights
Language: Names 4 colors; repeats sentence of 10 syllables;
counts 10 pennies correctly
Social: Dresses and undresses; asks questions about meaning
of words; engages in domestic role-playing

45. What is the age of child should be know few word ?


A. 6 month
B. 8 month
C. 12 month
D. 24 month
Answer: C
46. Child with fever, general swelling , and dark colored of urine , which best evaluate for this pt ?
A. Us
B. KFT
C. Urine culture
D. Urine specimen

Answer: D

Note : urinalysis and sediment examination are crucial in the evaluation of patients with acute nephritic syndrome.
Look for the following:
Protein
Blood
Red blood cells (rbcs)
White blood cells (wbcs)
Dysmorphic rbcs
Acanthocytes
Cellular (ie, rbc, wbc) casts
Granular casts
Oval fat bodies

Reference : Http://emedicine.medscape.com/article/239278-workup

47. Mother and her child visited the pediatrician for 6 months vaccination, however the mother stated that
her child was hospitalized after receiving the 4 months vaccination he develop anaphylaxis , what is the
right thing to do?
A. Test the child for which antigen is allergic from
B. Give him all vaccines except DTP
C. Vaccinate him and discharge home
D. Vaccinate him and hospitalize the child for 1 hour

Answer: B
Note : if B not in choices choose A or D according 2
different consultants

48. Attention deficit hyperactivity disorder ( they give me the symptom not the diagnosis sx of
hyperactivity and deficit in concentration in school and home ) child what is the manegment?
A. Ecitalpram
B. Atomoxetine
C. Olanzapine
D. Clonazepam

Answer : B
Note : other medication : methylphenidate

Reference : https://www.nhs.uk/conditions/attention-deficit-hyperactivity-disorder-adhd/treatment/

49. Which syndrome is associated with coarctation of aorta ?


A. Down
B. Patau
C. Edward’s
D. Turner

Answer: D
Refernce :
Http://www.merckmanuals.com/professional/pediatrics/chromosomal-anomalies/sex-chromosome-anomalies

50. Child with aspirin intake overdose … what kind of acid-base balance ?
A. Metabolic alkalosis
B. Metabolic acidosis
C. Respiratory alkalosis
D. Respiratory acidosis
Answer : B
Note : Respiratory alkalosis first then metabolic acidosis.

51. Child presented with croup presentation (barking cough … etc.), what is the management?
A. Inhaled steroid
B. Inhaled epinephrine and oral steroid
C. Oral steroid with antibiotics
D. Empirical antibiotics

Answer : B
Note : A single dose of dexamethasone has been shown to be effective in reducing the overall severity of croup ( use
for mild croup ) , Nebulized racemic epinephrine is a 1:1 mixture of dextro (D) isomers and levo (L) isomers of
epinephrine with the L form (L-epinephrine) as the active component. Its use is typically reserved for patients in the
hospital setting with moderate-to-severe respiratory distress .

Reference : https://emedicine.medscape.com/article/962972-treatment#d10

52. During otoscopy examination of a child, pulling the ear at which direction is going to help to see
tympanic membrane?
A. Anterior and inferior
B. Posterior and inferior
C. Anterior and superior
D. Posterior and superior

Answer: B
Note : for Adult Answer : D

53. What is the daily fluid requirement for 15kg baby?


A. 1000
B. 1200
C. 1400
D. 1600

Answer: B
Explanation:
For the first 10 kg we give 100 ml per each kilo
For the second 10 kg (from 10 to 20 kg) we give 50 ml per each kilo
For every kilo after 20 kg we give 20 ml per each kilo
So this baby is 15 kg which will be (1000 ml from the first 10 kg) and (5 kg multiplied by 50 = 250 ml) so
the baby’s
daily requirement will be 1250 ml per day.

54. 12 year boy came to the clinic complaining that he is short comparing to his colleagues, his height
was 155 and his weight is similar to 10 years boys.
His mother height is 145, father is 178, what is the expected height of the pt.?
A. 140_155
B. 155_160
C. 160_175
D. 175_195

Answer : C , closest
To calculate mid-parental height:
For Girl : ( father height + mother height ) / 2 then subtract – 13
For Boy : ( father height + mother height ) / 2 then Add + 13

55. Baby full term flax..irritable..what to give ? not clear


A. Ca
B. Mg
C. Ringer
D. Glucose
Answer : A
Http://emedicine.medscape.com/article/241893-treatment
56. 11-year-old (typical bacterial meningitis case) which ab× will be given ?
A. Ceftriaxone and gentamycin
B. Ampicillin and gentamycin
C. Penicillin and gentamycin
D. Vancomycin

Answer: A
Note : 3rd-generation cephalosporins (ceftriaxone or cefotaxime) for s. Pneumoniae and n. Meningitides and
vancomycin for penicillin-resistant strains of s. Pneumoniae and for s. Aureus , so the best answer is : Vancomycin +
Ceftriaxone

57. Infant with mother for routine checkup, when the mother put the baby he was laughing and when he
saw the doctor he tried to reach his mother what is most likely his age
A. 2 month
B. 4 month
C. 6 month
D. 8 month

Answer: C
Note : prefer mother at 6 months and try to reach her

Nelson Table for 1st year :


Emerging Patterns of Behavior During the 1st Yr of Life*

NEONATAL PERIOD (1ST 4 WK)


Prone: Lies in flexed attitude; turns head from side to side; head sags on ventral suspension
Supine: Generally flexed and a little stiff
Visual: May fixate face on light in line of vision; “doll’s-eye” movement of eyes on turning of the body
Reflex: Moro response active; stepping and placing reflexes; grasp reflex active
Social: Visual preference for human face
AT 1 MO
Prone: Legs more extended; holds chin up; turns head; head lifted momentarily to plane of body on ventral
suspension
Supine: Tonic neck posture predominates; supple and relaxed; head lags when pulled to sitting position
Visual: Watches person; follows moving object
Social: Body movements in cadence with voice of other in social contact; beginning to smile
AT 2 MO
Prone: Raises head slightly farther; head sustained in plane of body on ventral suspension
Supine: Tonic neck posture predominates; head lags when pulled to sitting position
Visual: Follows moving object 180 degrees
Social: Smiles on social contact; listens to voice and coos
AT 3 MO
Prone: Lifts head and chest with arms extended; head above plane of body on ventral suspension
Supine: Tonic neck posture predominates; reaches toward and misses objects; waves at toy
Sitting: Head lag partially compensated when pulled to sitting position; early head control with bobbing motion; back
rounded
Reflex: Typical Moro response has not persisted; makes defensive movements or selective withdrawal reactions
Social: Sustained social contact; listens to music; says “aah, ngah”
AT 4 MO
Prone: Lifts head and chest, with head in approximately vertical axis; legs extended
Supine: Symmetric posture predominates, hands in midline; reaches and grasps objects and brings them to mouth
Sitting: No head lag when pulled to sitting position; head steady, tipped forward; enjoys sitting with full truncal support
Standing: When held erect, pushes with feet
Adaptive: Sees raisin, but makes no move to reach for it
Social: Laughs out loud; may show displeasure if social contact is broken; excited at sight of food
AT 7 MO
Prone: Rolls over; pivots; crawls or creep-crawls (Knobloch)
Supine: Lifts head; rolls over; squirms
Sitting: Sits briefly, with support of pelvis; leans forward on hands; back rounded
Standing: May support most of weight; bounces actively
Adaptive: Reaches out for and grasps large object; transfers objects from hand to hand; grasp uses radial palm; rakes
at raisin
Language: Forms polysyllabic vowel sounds
Social: Prefers mother; babbles; enjoys mirror; responds to changes in emotional content of social contact
AT 10 MO
Sitting: Sits up alone and indefinitely without support, with back straight
Standing: Pulls to standing position; “cruises” or walks holding on to furniture
Motor: Creeps or crawls
Adaptive: Grasps objects with thumb and forefinger; pokes at things with forefinger; picks up pellet with assisted
pincer movement;
uncovers hidden toy; attempts to retrieve dropped object; releases object grasped by other person
Language: Repetitive consonant sounds (“mama,” “dada”)
Social: Responds to sound of name; plays peek-a-boo or pat-a-cake; waves bye-bye
AT 1 YR
Motor: Walks with one hand held; rises independently, takes several steps (Knobloch)
Adaptive: Picks up raisin with unassisted pincer movement of forefinger and thumb; releases object to other person
on request or gesture
Language: Says a few words besides “mama,” “dada”
Social: Plays simple ball game; makes postural adjustment to dressing

58. Case of 12 years old boy on skate downstairs he felt and had perineum trauma, with bruises over the
scrotum, perineum, lower abdomen. Retrograde cystourethrogram show extravasation of the dye. Where
is the injury?
A. Penile urethra
B. Urinary bladder
C. Prostatic urethra
D. Ureters.

Answer: A
Note : Anterior urethral (composed of the penile and bulbar urethra) injuries. This type of injury is seen most
commonly in blunt trauma, but is not usually associated with pelvic fractures. It results from a strong blow to the
perineum that causes the bulbar urethra to be crushed against the inferior border of the pubic symphysis. This
typically occurs in a fall astride, a straddle injury from a vehicle accident, an assault, or from bicycle handlebars.

59. 2 year old child got otitis media after Upper Respiratory Tract Infection .what is the Treatment ? (
common Q )
A. Observe.
B. High dose ibuprofen.
C. Amoxicillin 45 mg/kg/day for 5 days.
D. Amoxicillin 90 mg/ kg/ day for 10 days.

Answer: d
Note : high dose amoxicillin is DOC for O.M in pediatrics
60. absence of moro reflex in right side of infant is due to ?
A. Intracranial hemorrhage.
B. Neonatal hypoglycemia.
C. Neonatal sepsis.
D. Erbs palsy.

Answer: d

Note : Erbs palsy causes asymmetrical moro reflex. Intracranial hemorrhage causes poor moro reflex.

Reference : Https://www.msdmanuals.com/professional/pediatrics/perinatal-problems/birth-injuries

61. 13 years old with enteric fever. Allergic or resistant to chloramphenicol (i forgot). Treatment is ?
A. Double chloramphenicol.
B. Add ciprofloxacin.
C. Ciprofloxacin alone (orally)
D. Im ceftriaxone
Answer: D
Note : double or triple or quadrable chloramphenicol will not affect the resistant so it still not effective , ciprofloxacine
not given for any one less than 18 years due to its effect on growth cartilage , so the answer is D

62. Blood film for girls came abdominal pain cough splenomegaly ( LONG CASE ) dx ?
A. P.malaria
B. P.falcifom
C. P. Oval
D. Mp. Something

Answer: Depends on blood film


Note : see them in Google image if u confuse choose B

63. Newborn in endemic area village of cretinism suspect to have many cretinism baby what's your first
action ?
(one of the most common complicated Q )
A. add Iodine supplements in water
B. Repeat thyroid function after 1 or 2 month
C. give the babies thyroxine
d. do nothing

Answer: C most likely


Note : if ask about the baby only , the answer for sure C , if the Q about all babies in village or about the tt of enemic
condition the answer will be between A and C

64. Neonate with physiological jaundice her parent brought the baby because the color changed from
yellow to greenish what you suspect disease: (i’m not sure about q)
A. ABO incompatability
B. G6PD def
C. Due to oxidation of bilirubin
D. The treatment start to work

Answer, C
For more details read about bilirubin metabolism

65. Treatment of EBV infectious mononeucliosis (in scenario there patient with tonsillar exudates,
lymphadenopathy, splenomegaly) ?
A. Oral acyclovir
B. Oral antibiotic
C. Iv acyclovir
D. Supportive tt
Answer: D
Note : supportive means : hydration , Paracetamol , NSAID

66. Most common cause of precocious puberty in girls ?


A. Idiopathic
B. Adrenal tumor
C. Ovarian tumor
D. Functional ovarian cyst"

Answer: A
Note : Idiopathic —central precocious puberty is idiopathic in 80 to 90 percent of cases of girls, but in only 25 to 60
percent of boys (uptodate)

67. Child sit and support his head, laughing and cooing :
A. 4 weeks
B. 6 weeks
C. 8 weeks
D. 16 weeks

Answer: D

Note : Baby can support head steadily and lough at 4 months .

68. Child his height and weight below normal besides growth hormone what you will order ?
A. Somatomedin c
B. Aldosterone
C. Insulin
D. Testosterone

Answer: A
Note :If case is due to growth hormone deficiency which more likely here so the answer is a and the the height should
be fallen more than 2.5 sd below mean for chronological age sex and ethnicity. (below 25% in growth chart)
Somatomedin c is another name to insulin like growth factor (igf-1)
(uptodate)

69. Thalassemia causing stillbirth, which type is it?


A. Two normal beta chains and 2 normal alpha chains
B. Two normal beta chains and 4 abnormal alpha chains
C. 2 abnormal beta chains and 4 normal alpha chains
D. 4 abnormal beta chains and 2 normal alpha chains

Answer: B
Note : This is alpha thalassemia major this will cause loss of all alpha chain and it will lead to hydrop fetalis and
consequently cause stillbirth. (uptodate/medscape)

70. which of the following is cyanotic heart disease ? ( common Q )


A. TOF
B. ASD
C. VSD
D. PDA

Answer: A
Note : cyanotic heart disease 5 Ts : Tetralogy of Fallot , Total anomoulas pulmonary venous connection ,
Tricuspid atresia , Truncus arteriosus , Transposition of great artery

71. Baby sit briefly, crawl , move object from hand to hand , but can't do pincer grasp , age ?
A. 4 month
B. 6 month
C. 7 month
D. 9 month

Answer : C
See nelson table above
72. Oropharyngeal maculopapular rash .. Also rash in palm
of hand and plantar of foot , microoganism ?
A. CMV
B. EBV
C. Coxsackie virus
D. Varicella

Answer : C
Note : hand mouth foot disase

74. Child with vomiting and diarrhea. On exam .. Sunken eyes.. Skin turgor.. Depressed
fontanelle.. The degree of dehydration is ?
A. 5%
B. 10%
C. 15%
D. 20%

Answer: B

Degree of dehydration :
●mild dehydration (3 to 5 percent volume loss) – a history of fluid losses may be the sole finding, as clinical signs may
be absent or minimal.
●moderate dehydration (6 to 10 percent volume loss) – signs and symptoms are now apparent and can include the
following: tachycardia, orthostatic falls in blood pressure, decreased skin turgor, dry mucous membranes, irritability,
decreased peripheral perfusion with a delay in capillary refill between two and three seconds, and deep respirations
with or without an increase in respiratory rate. There may be a history of reduction in urine output and decreased
tearing, and, in infants, an open fontanelle will be sunken on physical examination.
●severe dehydration (>10 percent volume loss) – such children typically have a near-shock presentation as
manifested by hypotension, decreased peripheral perfusion with a capillary refill of greater than three seconds, cool
and mottled extremities, lethargy, and deep respirations with an increase in rate. Severe hypovolemia requires
immediate aggressive isotonic fluid resuscitation to restore the effective circulating volume (ecv) and prevent
ischemic tissue injury.

75. Child presented with petechiae rash and his platelets is 15,000 , otherwise healthy. What will you do
for him?
A. Bone marrow aspiration
B. Splenectomy
C. IVIG
D. Steroid

Answer: C ( more accurate supportive , so if its in choices choose it )


Note : this is case of ITP , the tt in pediatrics is supportive except in these situations :
1. If Platelet less than 10,000 some sources said 15,000 due to risk of major bleeding
2. If there is bleeding other than skin e.g epistaxis or hematuria

Start with IVIG if above any indication present . Steroid takes time and may cause Bone marrow changes and mask
the malignancies in bone marrow so its not first tt in emergency

76. Child can run , say short stories , what is the age:
A. 2 years.
B. 3 years.
C. 4 years
D. 5 years.
Answer: c
Note : A child starts to run at 18 months, and tell a story at 4 years.

77. Baby came to the emergency with abnormal movement and teeth delay.
Glucose is 5 (normal range from (5-10)
Calcium is low
Magnesium is low
Phosphate is high what to give ?
A. Calcium
B. Glucose
C. Magnesium
D. Ringer lactate

Answer: A , according to pediatric resident ( I am not sure 100% )

78. Baby came with barking coughing ( croup case ) what you will hear on pulmonary auscultation? ( common
Q)
A. Bronchial breathing
B. Decreased breath sounds
C. Prominent increase inspiratory sound due to subglottic narrowing
D. Increase exploratory wheezing

Answer: C , its inspiratory stridor

79. Child with septic arthritis came to ER with knees pain , swelling . Management ? ( common Q )
A. Oral antibiotic for 14 days
B. Broad spectrum iv antibiotic
C. Surgical drainage and iv antibiotic
D. Antipyretic till the result of aspiration culture

Answer: C

Refernce : Toronto note

80. Kwashiorkor:
A. Low protein and low carbohydrate. (marasmus)
B. High protein and low carbohydrate.
C. Low protein and high carbohydrate.
D. High protein and high carbohydrate.

Answer: C
Reference : Illustrated textbook

81. 7years girl presented with bilateral symmetrical lower limb weakness, EMG showed ( i can't
remember ) ,knee reflexes showed absolute bilateral knee reflex absence , she had hx one week ago of
gi symptoms , what is the best treatment ?
A. Plasmapheresis
B. Immunoglobulin
C. Steroids
D. Methotrexate

Answer: B
Note : it is a case of guillain - barre syndrome the treatment is iv immunoglobulin if the patient has significant
weakness , plasmapheresis if progression continue both will hasten the recovery.

82. Which of the following is a cyanotic heart disease?


A. Asd
B. Vsd
C. Pda
D. Ductus arteriosus

Answer: D

83. Child with epilepsy on anticonvulsant what you will change in his vaccines ?
A. Change opv to ipv
B. DTP
C. Remove all vaccines
D. Remove all live vaccines

Answer: B
Refernce : http://www.vaccines.gov/basics/safety/should/

84. Baby said hi when he entered the clinic, imitates his mother, feeds his doll, refers to himself “me” and
say “eye”:
A. 12 months
B. 15 months
C. 18 months
D. 24 months

Answer: C or D ( mostly D )

Illustrated textbook and Neslson

85. Baby thirsty with tachycardia, sunken eye. Volume loss ?


A. 1%
B. <3%
C. 5-9%
D. 9%

Answer: C

86. (case of intussusception) child came with colicky abdominal pain, vomiting, bloody stool. Us
showed doughnut sign. What is the most important step in management of this case ? ( common Q )
A. Urgent surgery referral
B. Nasogastric decompression
C. Iv fluid resuscitation
D. Barium enema

Answer: C

Note : stabilize the pt first


Iv fluid resuscitation, then decompression, afterward, air enema or barium if air in available
( Reference : . Master the board udlme step 2 pediatric chapter)

87. Which vaccine is contraindicated in HIV patient ?


A. Opv
B. Varicella
C. Mmr
D. Hbv

Answer: A
Note : all contraindication but OPV most serious one and contraindicated regardless viral load

88. 12 Year old girl to diagnose HTN ?


A. 120/80
B. 40/99
C. Above 90 percentile
D. Above 95 percentile

Answer: D

89. Young baby complains of vesicle , pastule on back the lesions like band ?
A. Shingles
B. Chicken box
C. Herpes
D. Coxsackievirus

Answer: A
90. Child present to er with fever and sore throat for one week. Now he has paroxysms` cough and cyanosis at end
of cough ?
A. Epiglotitis
B. Sinusitis
C. Croup
D. Bronchitis

Answer : C
Note : paroxysmal cough presents more in ( Pertusis ) so if it is in choices choose it

91. What is the effect of polio (IPV & OPV ) vaccines on body ?
A. All lead to the formation ag in the anterior horn
B. All lead to the formation of the ab in the serum which fight the virus
C. They all enter the intestinal mucosa where the entry of the virus is
D. They all lead to the formation of interferon gamma

Answer : B , If ask about OPV only the answer will be C

92. Baby hypotension sever vomiting and watery diarrhea what is the electrolyte abnormality
A. Low Na
B. High Na
C. High K
D. Hyperglycemia

Answer: A
note : vomiting lead to : hypokalemia and hyponatremia , diarrhea leads to : hyponatremia , so the answer is : A

93 . A 6 weeks old baby pale, jaundice on examination there is palpable spleen 2 cm below the costal margin.
Lab shows total bilirubin =205 mg\dl, direct bilirubin = 60 mg/dl, positive direct & indirect combs test,
Peripheral blood smear(attached photo shows spherocytosis) dx ?
A. hereditary spherocytosis
B. Gilbert disease
C. ABO incompatibility
D. Automimmune anemia

Answer : D ( positive
coombs )

94 . Which of the following congenital heart disease is secondary to failure of spiral rotation of the heart
septum?
A. Transposition of great artery
B. Asd
C. Vsd
D. Pda
Answer: A
Note : Misalignment of the sepatient um can cause the congenital heart conditions tetralogy of fallot, persistent
truncus arteriosus, dextro-transposition of the great arteries, tricuspid atresia, and anomalous pulmonary venous
connection

95. ( long scenario for cerebral palsy ). On examination there is crossing of lower limb when child suspend
by the axilla. Which type of cerebral palsy does the patient have ?
A. Hemiplagia
B. Diplagia
C. Quadriplagia
D. Athetoid

Answer: B
Scissoring is seen in spastic cerebral palsy ( diaplegic & quadriplegic) , Diplegia :affect lower limb more than
upper
96 . Child admitted with sore throat and bilateral knee pain?
A. Juvenile rheumatoid arthritis
B. Rhumatoid arthritis
C. Osgood shatler disease
D. Rheumatoid arthritis

Answer: A ( we need more details and we have to check all choices in exam )
Note : from these choices A , but if rhematic fever in choices and the knee pain migratory choose it ,

97 . Child eating a lot of milk but he does not eat meat, MCV hypochromic microcytic anemia , how will you manage
this child ?
A. Oral vitamins + iron
B. Trial of iron then then observe
C. Folic acid
D. Iron

Answer : B ( not sure 100% )

98. Child complains of unilateral scrotal swelling , does not transilluminate , what is next step ?
A. Discharge the patient
B. Give antibiotic
C. Do laparotomy
D. U/s and think about surgery

Answer: D
Note : to exclude testicular torsion and malignancy

99 . Newborn with congenital adrenal hyperplasia present with ?


A. Hirsutism
B. Infantile acne
C. Abdominal striae
D. Dehydration

Answer: D

Refernce : Http://emedicine.medscape.com/article/919218-overview

100. Baby with 5 min after birth assessing ( hr 120 ,breath irregular and grasping , acrocynotic , cough and
grimace , flexing of limb ) apgar score ?
A. 6
B. 7
C. 8
D. 9

Answer: B
Explanation: apgar score is assessed at 1 and 5 minutes and scored based on the table.

Reference: toronto notes 2015, page p66, pediatrics


101. 8 years old girl, parent complains that she looks older than her class mates , W.t and height above
the 95th percentile , otherwise normal whats next ?
A. Reevaluate after 12 months
B. Obesity medications
C. Life style modification
D. Surgical intervention .

Answer: A or C
Explanation: this is most likely due to obesity. Familial tall stature also known as constitutional tall stature is the most
common cause of tall stature. The second most common cause is nutritional. The height as well as the weight are at
higher percentile.

102. Failure of obliteration of pharyngeal arch 2 ,3 , 4 leads to?


A. Cervical cyst
B. Branchial fistula
C. Ectopic thymus
D. Parathyroid

Answer: B
Explanation: 2nd through 4th clefts form temporary cervical sinuses, which are obliterated by proliferation of 2nd arch
mesenchyme. Failure of obliteration leads to branchial cleft cyst within lateral neck.
Reference: first aid 2015, page 564

103. 4 month child witch developmental milestone?


A. Try to crawling
B. Roll over from side to side
C. Sit without support
D. Complete fixation of the head

Answer : D

104. Calculate glasgow coma scale: child crying and confused + respond when calling his name + withdraw to
pain ?
A. 9
B. 10
C. 11
D. 12
Answer: C
Note : in Glasgow coma scale we take higher scale for example in this Q cry and confused we calculate it as confused
which equal = 4 , respond to speech = 3 , withdrawal to pain = 4 , in pediatrics consolable crying = 4 , non consolable
crying = 3
105. Child football player on short acting β-agonist 5 times a week and use zafirlukast but still sx what to add
?
A. Long β agonist
B. Inhaled steroid
C. Theophylline
D. It’s okay no thing is needed

Answer: B

106. Baby can say a few words or 3 words! Whats the age ?
A. 1 Yr
B. 2 Yr
C. 3 Yr
D. 4Yr

Answer: A

107. Rheumatic heart dx prophylactic for 10 yrs old boy with no cardic involvement?
A. 3m
B. 6m
C. 6 yrs
D. 10 yrs

Answer: D

108. 16 yr from aferica (ginia) with painless neck mass for 5 weeks developed cough , fever , URT symptoms
dx ?
A. Burkitt lymphoma
B. Infectious mononucleosis
C. Hugging lymphoma
D. Lym dx

Answer: A

109 . Neonate take immunoglobulin from his mother whats the type of immunological ? ( Very common Q )
A. Active artificial immunity
B. Passive artificial immunity
C. Active neutral immunity
D. Passive neutral immunity
Answer: D

Naturally acquired passive immunity can be provided during pregnancy, and through breast-feeding
Naturally acquired active immunity occurs when the person is exposed to a live pathogen, develops the disease,
and becomes immune as a result of the primary immune response. Artificially acquired active immunity can be
induced by a vaccine, a substance that contains the antigen.
Artificially acquired passive immunity is a short-term immunization by the injection of antibodies, such as gamma
globulin, that are not produced by the recipient's cells. Naturally acquired passive immunity occurs during
pregnancy, in which certain antibodies are passed from the maternal into the fetal bloodstream.

110 . What measurement a you should take to relieve an infant abdominal colic ?
A. Antispasmodic drugs
B. Increase bottle feeding
C. Warm baths
D. Prevent child abuse

Answer: C
Reassure the parents and drug treatment generally has no place in management of colic unless gerd appears likely;
although the anticholinergic agent dicyclomine hydrochloride is effective against colic, it has rare but serious adverse
effects and cannot be recommended.
111. Child less than 50 percentile, polyuria, constipation, low na, k, cl, where is the primary defect?
A. Na, cl channel
B. K chanel
C. H+ reabsorption
D. H+ secretion

Answer: A
Note : this most likely Gitelman syndrome and the defect in Na / cl channel , read about Gitelman and Bartter
syndrome and comparsion between them .

112. Child was diagnosed with dm type 1 presented with frequent hypoglycemic attack at different time
during the day, he was diagnosed 6 month back, and he is compliant to his diet and treatment , the most
likely cause of his symptoms ?
A. Brittle diabetes
B. Dawn phenomena
C. Smogyi phenomena
D. Honey moon period

Answer: D

Brittle diabetes is severe instability of blood glucose levels with frequent and unpredictable episodes of hypoglycemia
and/or hyperglycemia that disrupt quality of life.
Honey moon period begins early after dx of DM1 , easly to control DM but u have to re-adjust insulin doses to avoid
hypoglycemia.

113.which of the following is an earliest sign of puberty in boy?


A. Pubic hair
B. Penis enlargement
C. Scrotal enlargement
D. Testicular enlargement

Answer: D
Note : Many references mention that testicular enlargement are the first sign of puberty in males.
First sign of puberty in female is : breast budding

114. case of Tonsilitis( from case u know that ) tt ?


A. Amoxicillin / clavilonic
B. Vancomycin
C. Ciprofloxacillin
D. Trimethoprime / sulpha

Answer: A
To treat tonsillitis penicillin v or amoxicillin or erythromycin (if penicillin allergy) x 10 d are
used Reference: toronto notes

115 . 10 years old child with difficulty to eat for 2 years , multiple time aspiration in the past two weeks , on total
parenteral nutrition 1000 calorie and protein daily with lab result attached (low albumin and anaemia ) what is
your action ?
A. Continue same tpn dose
B. Continue tpn with increase dose to 2000
C. Insert gastrostomy tube, same calorie
D. Insert gastrostomy tube decrease calorie to 80

Answer: B ( not sure 100% )

Since the patient is complaining of aspiration; gastronomy can cause aspiration as a complication. Since the patient is
anemic and has hypoalbunimia continuing the same dose is wrong.
Reference: toronto notes
References : kaplan usmle step 2 pediatric

116 .long case of Child with red urine and constipation. Urine analysis increase rbcs, wbcs, protein
A. Uti
B. Hsp
C. Hemolytic uremic
D. Post streptococcus glomurenephritis

Answer: B or D

117. Baby with vomiting and diarrhea , looks ill , cries with tears , capillary refill 3 sec , vital signs were normal
almost tt ?
A. Moderate dehydration give oral rehydration
B. Moderate dehydration give IV fluid
C. Sever dehydration give bolus fluid
D. Sever dehydration give another something (couldn't remember the exact sentences but these were the choices
)

Answer: B
Note : patient most likely has moderate dehydration which is usually treated using oral rehydration solution but due to
vomiting we tt it by IV.

118. Case: mother come to doctor because worry about her child my be had dehydration what is most tool
exam use to determine degree of child dehydration ?
A. History
B. Clinical
C. C.t
D. Ultrasound

Answer : B
Note : the best way to determine degree of dehydration is : w.t of baby before illness and his W.t now , this difficult
sometimes so we depend on clinical assessment .

119. 5 years unwell with lower limb ecchymosis, positive cd10 (calla) ur diagnosis ?
A. Aml
B. All
C. Hodgkin
D. Ebv
Answer: B
Note : Cd 10 also called common acute lymphoblastic leukemia antigen (calla), one of first markers to identify
leukemic cells in children (hence its name)

120. Child with stridor, slightly relieved by epinephrine and symptoms presents what will you do next?
A. Steroids
B. Antihistamines
C. Antibiotics
D. Give Epinephrine again

Answer: D
Read about croup tt

121. 10 months, set without support, can't walk or crawl, say mom to hold him , grip pen what is the delay:
A. Socially
B. Fine motor
C. Gross motor
D. Speech

Answer: C
Note : at 10 months at least crawl , here he cant even crawl

122. Child who can understand few commands, points to what he needs, walks without support, crawl
upstairs, his age in months?
A. 6 M
B. 9 M
C. 15 M
D. 24 M

Answer: C
Note : see Nelson Table

123 . Patient increase foot size 39 >> 41.5 and increase size of hand and joint which hormone responsible ?
A. Thyroxine
B. Prolactin
C. Acth
D. Somatotropin hormone “ known as growth hormone”

Answer: D
Note : : all other answers are not applicable for above description.

124. A one month old child with total bilirubin of 200 and direct bilirubin of 80. What's the cause?
A. Gilbert syndrome
B. Crigler-najjar syndrome
C. Choledocal cyst
D. Abo incompatibility
Answer: C
Note :All options are examples of unconjugated hyperbilirubinemia except for choledocal cyst ( conjugated
hyperbilirubinemia) It presents as jaundice, acholic stools in early infancy and palpable mass in right upper quadrant
with hepatomegaly.

125 . Child with eczema use topical steroid not affected what we add ?
A. Oral antibiotic
B. Topical antibiotic
C. Sulfa something
D. Tacrolimus
Answer: D
Reference: http://emedicine.medscape.com/article/1049085-treatment

126 . Minimal probability of a baby getting beta thalassemia from carrier parents?
A. 0%
B. 25%
C. 50%
D. 75%

Answer: B

127. A child complains of bilateral knee swelling, fever and pharyngitis, “all at the same time I think”. His labs
show high ESR with no other abnormality. What is the most likely diagnosis?
A. Juvenile R.A
B. Septic arthritis
C. Acute rheumatic fever
D. Infectious mono

Answer: C
Note : Acute rheumatic fever is diagnosed according to modified jones criteria: 2 major or 1 major 2 minor plus
evidence of preceding step infection (history of scarlet fever, group a strep pharyngitis culture, positive rapid antigen
detection test, anti-streptolysin o titers)
Major criteria Minor criteria
Sydenham chorea Prolonged pr interval
Transient migratory arthritis Elevated esr
Rheumatic subcutaneous nodules Arthralgia
Erythema marginatum Elvated crp
Panacrditis Fever
Toronto notes 2015

128. Regarding PALS, which is true in pediatric resuscitation?


A. 30 compression 2 breath
B. 15 compression 1 breath
C. 10 compression 2 breath
D. 5 compression 1 breath

Answer: A
Note : The ratio is 30:2 if single rescuer and 15:2 if two rescuers.

129. Four years old child c/o fever and maculopapular rash associated with auricular and occipital lymph
node, he only received his birth vaccines. Dx ?
A. Mumps
B. Measles
C. Rubella
D. Chicken pox

Answer: C
Note : Rubella clinical picture is asymptomatic infection, mild coryza, erythematous discrete skin rashes usually fade
after 3 days, lymphadenopathy (most common) sub-occipital, post-auricular, and cervical. +\- splenomegaly.
Reference: manual of clinical pediatrics.
130. 15 years old male come for routine check up, labs hb: 10 , MCV: 69 MCH: 20 wbc: normal. Diagnosis?
A. Iron deficiency Anemia
B. Beta thalasemia trait
C. Anemia of chronic disease
D. Sickle cell anemia

Answer: B
Note : the hints in Q , the age 15 and MCV , in IDA usually MCV low but not less than 70 , also the age more with
Thalassemia trait which usually diagnosed incidentally .

131. Neonate + prolonged bleeding after circumcision, aptt high, pt, bleeding time and platelet are normal,
condition is most likely due to deficiency in
A. V
B. Vii
C. Viii
D. X

Answer: C
Note : bleeding after circumcision classical in Hemophilia A

132. Which one of these disease likely to exhibit cyanosis in later life?
A. 6 year with coartcation of aorta (lower limbs only)
B. VSD (reversal of the shunt occurs early in childhood or adulthood)
C. Trancus arteriosis (early cyanotic)
D. ASD (reversal of the shunt occurs late in
adulthood)

Answer: D
( not sure 100% , because VSD also comes late )

133. Clear case of DKA ( DM1 baby comes with vomiting and abdominal pain and change in odor , glucose
high , ) in a child what's the initial management?
A. Electrolytes replacement
B. Fluid replacement
C. Bicharb
D. Insulin infusion

Answer: B

134. Child with symptoms of DKA abg ph 7.24 , PCO2 lower than normal , HCO3 lower than normal. What is
the abnormality ?
A. Compensated metabolic acidosis
B. Compensated metabolic alkalosis
C. Uncompensated metabolic acidosis
D. Uncompensated metabolic
alkalosis

Answer: A
135. Cow milk over breast milk more in what?
A. Fat.
B. Calorie.
C. Protein.
D. Iron

Answer: C

136 . 4 weeks old infant, mother happy he never cries (i.e. He is lethargic). On examination: jaundice +
umbilical hernia + distended abdomen + coarse face features + bulging frontal fontanel. Diagnosis?
A. Congenetal hypthyroidism
B. Gilbert's syndrome
C. Cerebral palsy
D. Rickets

Answer: A

Congenital hypothyroidism:
The vast majority (more than 95 percent) of infants with congenital hypothyroidism have few if any clinical
manifestations of hypothyroidism at birth. This is because some maternal thyroxine (t4) crosses the placenta, so that
even in infants who cannot make any thyroid hormone, umbilical cord serum t4 concentrations are about 25 to 50
percent of those of normal infants. In addition, many infants with congenital hypothyroidism have some, albeit
inadequate, functioning thyroid tissue.

137. Baby diagnosed with cystic fibrosis. ..he has + sweat chloride test his brother is normal, to confirm
diagnosis of cystic fibrosis?
A. Ctfr gene in parent
B. Ctfr in sibling
C. Chloride test. .parent
D. Chloride test in
sibling

Answer: A

138. Child with dehydration, depressed anterior fontanel, and decreased skin turgor. What is the percentage
of dehydration?
A. 5
B. 10
C. 15
D. 20

Answer : B

139. The most common drug used to treat juveniles rheumatoid arthritis is:
A. Paracetamol
B. Penicillamine
C. Systemic steroid
D. Aspirin

Answer: D
Note : The answer should be NSAID . Aspirin used to be the most common but aspirin is no longer the drug of first
choice because of the increased frequency of gastric toxicity and hepatotoxicity when compared to other NSAID
medications , so check the choices in exam .

140. Pneumococcal 13 vaccine whats type ?


A. Toxoid
B. Conjogoid
C. Inactivated
D. Live Attenuated
Answer: B
Reference: kaplan pediatrics

141. 4m old baby for vaccine she said at 2m he had fever and erythema and swelling
A. Give 4m same dose
B. Allergic test
C. Not give DTP
D. Antipyretic and antihistamine prophylactic

Answer: A
Note : give the vaccine of 4m as it is , after vaccine for local reaction give cool compressor with acetaminophen or
nsaids , antipyretic if developed fever.

142. Child who's lethargic and losing his concentration, Hgb is 10.5, what to give?
A.IM iron.
B. Oral ferrous sulphate.
C. Forti cereal.
D. Vit,b12
Answer: B
Note : most lijely IDA The patient is symptomatic, and the initial treatment for
IDA is oral Ferrous Sulfate

143. 8 y/o girl came with her father on wheel chair with pain in her knee. On examination there was limited
flexion and extension, they didn't mention any thing about knee effusion.
BP 105/58
HR: 101
Temp: 39.6
How to diagnose?

A. Rheumatic Factor
B. ASO
C. Arthrocentesis
D. ECHO

Answer: B
Note : not clear Q , Anti-streptolysin O (ASO) titre and throat swab should be considered if rheumatic fever is
possible (Jones criteria). Otherwise, arthrocentesis should be considered if septic arthritis is possible.

144. A child can roll over, sit triploid, attempt to take object. Which month?
A. 6
B. 9
C. 2
D. 10

Answer : A

145. Angular chelosis is due to deficiency in what vitamin?


A. B12
B. B6
C. B2
D. B1

Answer : C
Note : B2 deficiency manifests as angular chelosis, stomatitis and glossitis. B12 deficiency manifests as
megaloblastic anemia and peripheral neuropathy. B6 deficiency also manifests as peripheral neuropathy.
Reference Master the board

146. Treatment of post-streptococcal glomerulonephritis in children with edema and HTN?


A. High dose of antibiotic
B. Diuretic for edema
C. Diuretic for
HTN
D. Steroid
Answer : B or C ( mostly B )
Note : The mainstay of treatment of Poststreptococcal Glomerulonephritis is symptomatic treatment starting with salt
and water restriction then loop diuretics to control both the edema and the HTN.
Reference: http://emedicine.medscape.com/article/240337-treatment

147. Surgeon want to treat child diagnosed with PDA. During procedure, the surgeon is at risk to injure ?
A. Left Phrenic Nerve
B. Vagus Nerve
C. Left Recurrent Laryngeal Nerve
D. R.t recurrent Laryngeal nerve

Answer: C
Reference : Https://www.openanesthesia.org/pda_repair_recurrent_laryng_n_injury/

148. Fluid required to give a 10 Kg neonate, loss 5% of his body weight ?


A. 1000 ML
B. 1500ML
C. 2500ML
D. 3000ML

Answer: B
Deficit: 5% X 10 x 10 = 500 ml
Maintenance: (100ml / kg / 24h for the first 10 kg) = 1000 m
Total: 1500 ml

149. 6 years old with blood pressure above 95th percentile - +ve family hx of HTN - radial pulse intact, femoral
pulse is absent , what sur diagnosis ?( common Q )
A. Essential hypertension
B. Renal artery stenosis
C. Coarcotation of the aorta
D. Polycystic kidney disease

Answer: C
Note : Radio-Femoral pulse delayed and Rib notching important hints to answer ( coarctation of aorta ) any any Q

150 . A young boy presented with diarrhea sometimes bloody , Wight loss , arthritis , anemia the diagnosis
is :
A. Crohn’s
B. UC
C. Celiac
D. Gastritis

Answer: B
Note : the same Q I had im my exam but without U.C in choices the answer was Crohns
Reference : Step Up to Medicine, 4E, p164 / davidson's medicine ,20 E, P462

151. 1 month Baby come with abdominal distension and constipation since birth what would you do next :
A. X-ray
B. Biobsy
C. Rectal manometrey
D. Barrium enema

Answer: A
Note : best initial step ( Rectal exam ) if not inchoices then plain abdominal x ray , then Barium enema and
the
Best test is ( Full thickness rectal Biopsy )

152. Pediatric patient known case of Asthma since age 2 years. He is now asymptomatic with rare uses of
albuterol inhalers, he came for counseling, he had contac t sport participation, what is the best question
to ask to know his response ?
A. “Are keeping up with your friends”
B. “How frequent do use inhaler”
C. “Presence night symptoms (cough)”
D. ‘’ Day time symptoms ‘’

Answer: C

153. 12-hour newborn developed jaundice. Which investigation will you order?
A. Hb electrophoresis
B. Osmotic fragility test
C. G6PD screening
D. Enzyme assay

Answer: C

154. 8 year child come with 30 BMI what can you do for him ?
A. Surgery for obesity
B. Give him drug for obesity
C. Ressure and Live style
D. No thing

Answer : C

Note : Life style modification and multidisciplinary approach is most important step in treating obesity in pediatric
population
155. Peadiatric case croup , from where take swab ?
A. Naophangeal swab
B. Nasal swab
C. Pharyngeal
swab.
D.Esophageal

Answer: A

156. Child in the hospital play and come to his parents say stories , draw head and hands and legs what is
the age of this child ; ? ( common Q )
A. 3yrs
B. 4yrs
C. 5yrs
D.2yrs

Answer: B
Note : Nelson Table

157. Neonate with sign of sepsis which empirical antibiotic ?


A. Ampicillin
B. Gentamicin
C.
Cefotaxime
D. Vancomycin
Answer: A

158. Child is complaining of severe throbbing unilateral headache, aggravated by light. What is the most
likely diagnosis ?
A. Migraine
B. Cluster headache
C. Stress headache
D. Tension headache

Answer: A

159. An adolescent boy came to the clinic with unilateral gynecomastia. No other complaints. Everything
was normal. How will you manage ?
A. Reassure
B. Give hormonal therapy
C. Breast us
D. Surgery

Answer: A
Note : Gynecomastia (transient development of breast tissue) is a common self-limited condition seen in 50% of
male during puberty (but any discharge from nipple or fixed mass should be investigated).
Reference: toronto notes.

160. Months old infant his parents were not able to bring him for his 4 months vaccination. What will you
do?
A. Arrange for 4 months vaccination
B. Give missed vaccination and next appointment
C. Give vaccine together during next appointment
D. Give all vaccines at 6 month appointment

Answer: B

161. 14 years old boy comes to your clinic with swollen lips. He has similar episodes since 3 or 5 years.
Deficiency of which of the following caused his presentation?
A. Factor d
B. Anaphylatoxin inhibitor
C. C1 esterase inhibitor
D. Elastase
Answer: C
Note : Hereditary angioedema is an autosomal dominant disease caused by low levels of the plasma proteins c1
inhibitor
(c1-inh).
Reference: medscape. Nelson 7th

162. 2 weeks neonate passed unformed stool. What will you do?
A. Prescribe formula milk.
B. Give oral rehydration solution.
C. Prescribe lactose-free milk.
D. Give Lacose free formula

Answer: B
Note : to prevent dehydration

163. Pediatric patient with classical symptoms of DKA + elevated blood glucose . What will you do next?
A. Urine dipstick
B. Genetic testing
C. Hba1c
D. KFT

Answer: A
Note : Urine dipstick to pick up DKA early and to prevent further complications , We see ( acetoacetate ketones in
dipstick )

164. Baby with tachypnea, cough, hemoptysis and bilateral lung infiltrates. What is the treatment ?
A. Steroid
B. Antibiotic
C. Surgery
D. Beta blocker

Answer: B
Note : most likely infectios process

165. Which of the following is a cyanotic heart disease in children?


Asd
Vsd
Tetralogy of fallot
Answer: c

166 . A 6 years child of positive HBV mother not taken any vaccine except BCG after delivery. What will you
give him ?
A. MMR,OPV,HBV
B. HIB, MMR,OPV,HBV,PCV
C. HIB, MMR,OPV,HBV
D. DTP , OPV , Varicella , MMR

Answer : D
Note : the most important presence of ( DTP , OPV , Varicella ,
MMR )

167. A 2 years old child can run and play ball. Socially active but he refused to share his toys with other
children. What you will say to his parents?
A. Delay motor
B. Social impairment
C. Fine motor
D. Well developed and normal response

Answer: D
168. 2 month boy present with 2 cm of hemangioma in the back .. What is the tt ?
A. Close follow up
B. Excision
C. Beta blocker
D. Antibiotics

Answer: A
Note : the vast majority of infantile hemangiomas do not require any medical or surgical intervention.]

169. 20 days infant diagnosis as meningitis, his culture show gram negative bacilli. Which of following
could be the organism?
A. Hemophiles influenza
B, E.coli
C. Neisseria
meningitides
D. S.pneumonia

Answer: B
Note : N.Meningitides it is gram negative diplococcus , H . Influenza it is gram negative coccobacili

170 . Patient with ostium secundum atrial septal defect. What you will see in his ECG ?
A. Prolonged PR interval
B. Left axis deviation
C. Right axis
deviation
D. LBBB

Answer : C

171. Best way to take urine sample ?


A. Midstream
B. Foley catheter
C. Bag sample
D. Aspiration

Answer: A
Note : Mid-stream urine sample means that you don’t collect the first or last part of urine that comes out. This
reduces the risk of the sample being contaminated with bacteria

172. 7 month baby with you discover that the baby has VSD and asymptomatic otherwise healthy. What are
you going to do?
A. Close observation
B. Surgery
C. Follow up after 6 months
D. Cathetarization

Answer : C
Note : no intervention is usually required for patients with small defects. These patients are typically asymptomatic
and have a reasonable expectation of spontaneous closure or decrease in the size of the defect over time.
-patients who continue to have a murmur, but are otherwise asymptomatic and growing well at the 8- to 10-week
visit, are seen again by the pediatric cardiologist at approximately 12 months of age.\
-if the murmur persists at the 12-month and the patient remains asymptomatic and clinically stable, no further
intervention is required. Echo follow-up is typically performed at three years of age for patients with membranous
defects. In those with a muscular defect, no echo is required if the patient remains asymptomatic.
-asymptomatic patients with residual small defects are usually followed every two to five years for overall
173. Infant paraumbilical hernia, what you will do?
A. Put plastic in mid abdomen
B. Reassurance it will disappear after 4 weeks
C. Do hernia repair before start school
D. Hernioplasty

Answer: C
Note : usually self disappear within 2 years , if not disappear or Large in size surgery done at age 2 yr

174 . Neonate with deafness, rash .. What the cause ?


A. Rubella
B. Measles
C. Cmv
D. HBV

Answer: A
Note : congenital rubella sx : deafness , cataract , congenital heart disease

175 . What is characterized about x Fragile syndrome ?


A. Obesity
B. Macrogonadisim
C. Small face
D. high IQ

Answer: B
Note : 1 in 200 in frequency , Most common cause inherited mental retardation
Features:
Characteristic craniofacial finding (large head, prominent forehead, jaw, ear)
Characteristic neurobehavioral profile including (mental retardation, autism spectrum disorder, pervasive
developmental disorder)
Macro-orchidism
Mild connective tissue disorder including :
( joint laxity, patulous eustachian tubes, mitral valve prolapse)

reference : essential nelson of pediatrics

176. What come with turner syndrome ?


A. Hypothyroidism
B. Dm
C. Addison's
D. Hyperthyoidism

Answer: A

Reference: nelson essential of pediatric

177. A baby 6 month show regurgitation after every meal he esophagus PH is low he is normally developing
what is the tt ?
A. Close follow up
B. Surgical fundal
C. Esophageal manometry
D. give him PPI

Answer: A
Note : no treatment is required because this is a case of normal physiological gastroesophageal reflux which happen
in baby younger than 8months & presented with effortless regurgitation but otherwise the baby is normal , more than
50% will their sx will disappear at 1 year .
178. What is the most common congenital abnormally cause infective endocarditis ?
A. Asd
B. Vsd
C. Tetralogy of fallot
D. Pulmonary Atresia

Answer: C
Note : most of sources said : C , but some of sources said : VSD

179. Patient can't take BCG vaccine because he has degeciency in ?


A. IL6
B. TNF gamma
C. INF gamma
D. C1 elastase

Answer: C

180. 9-month old baby cannot sit by himself he is fisting his hand and crossing his leg most likely his
presentation ?
A. Normal child
B. Cerebral Palsy
C. Down syndrome
D. Turner syndrome

Answer: B
Note : Signs of hypertonia include keeping the hands fisted, keeping the legs extended and crossing the legs
or ankles , down syndrome comes with : Hypotonia

181. Child can support his head when sit and loving when stare to him or cooing
A. 4wk
B. 8wk
C. 12 weeks
D. 16 weeks

Answer: D , all done in 2 months except support the head with sit its at 4 months

182. Developmental milestones q: 5 words, hop on one leg ?


A. 48 m
B. 36 m
C. 24 m
D. 60 m

Answer: A
Note : Nelson Table

183. The most part in impaction of foreign body in lung ?


A. Left bronchus
B. Right bronchus
C. Bifurcation
D. Carina

Answer: B
Note : the right main Bronchus is wider, shorter, and more vertical than the left main bronchus
184. Boy with glomerulonephritis ( hematuria ), after week he developed hemoptysis what is the dx ?
A. Henoch schenolein Purpra
B. Goodpasture syndrome
C. Rapid deterotion
D. Ig A Nephropathy

Answer: B
Note : this classical presentation of Goodpasure , also Wegner Granulomatosis come with same presentation

185. Child history of hip pain , x-ray of hip shows effusion what is the next step of management?
A. Aspiration
B. Antibiotic
C. Us
D. repeat the x ray

Answer: A
Note : Ultra-sound guided hip aspiration to rule out other causes, since transient synovitis is a diagnosis of
exclusion.

186. 3 years old his parents has TB as a pediatrician you did PPD test after 72 hr you find a10 mm
enduration in the child this suggest ?
A. Inconclusive result
B. Weak positive result
C. Strong positive
result
D. negative test

Answer : C
Note : if high risk pt like ( HIV pt or pt exposed to relative with TB like this case ) 5 mm induration enough to say +
test
If moderate risk pt ( all pt with immune deficit and other risks ) 10 mm induration is enough
If pt without risk we need at least 15 mm to say its +ve

187. Child with high-grade fever for 5 days and sore throat ,on examination there was tonsillitis and
white patches on the gingiva. No Lymph node enlargement, ASO is negative. The most likely
causative organism is ? ( common Q )
A. Coxsackie virus.
B. Herpes simplex virus.
C. Ebv.
D. CMV

Answer : B
Note : Acute herpetic gingivostomatitis
This is a manifestation of primary HSV1 infection that occurs in children aged 6 months to 5 years. Adults may
also develop acute gingivostomatitis, but it is less severe and is associated more often with a posterior
pharyngitis.[5]
Infected saliva from an adult or another child is the mode of infection. The incubation period is 3-6 days.clinical
features include the following:
Abrupt onset
High temperature (102-104°f)
Anorexia and listlessness
Gingivitis (this is the most striking feature, with markedly swollen, erythematous, friable gums.)
Vesicular lesions (these develop on the oral mucosa, tongue, and lips and later rupture and coalesce, leaving
ulcerated plaques.)
Tender regional lymphadenopathy
Perioral skin involvement due to contamination with infected saliva
Course: acute herpetic gingivostomatitis lasts 5-7 days, and the symptoms subside in 2 weeks.

188. A case of a child with delayed developmental milestones flat nasal bridge wide forehead short hand
and feet what is the cause ?
A. Genetic
B. Dietry insufeciency
C. Metabolism
disorder
D. infectious

Answer : A
Note : most likely it is : Down syndrome

189. 15 month old child brought by the parent with peripheral and central cyanosis, He was diagnosed
with cardiac problem but the parent doesn't know what exactly, most likely diagnosis?
A, VSD
B. Tetralogy of fallot
C. PDA
D. Coaratacation of the aorta

Answer: B
Note : this is a baby with a cyanotic heart disease which includes most commonly tetralogy of fallot, tricuspid
atresia, pulmonary atresia, and severe forms of ebstein's anomaly of the tricuspid valve. Acyanotic heart diseases
include asd, vsd, and pda.
Source: http://pediatricheartspecialists.com/articles/detail/cyanotic_heart_defects

190. Child abused sexually, in vaginal examination hymen rupture in which of the following indicate rape? (
very common Q )
A. 6 o'clock
B. 8 o'clock
C, 3 o'clock
D. 9 oclock

Answer: A
Source: forensic book.

191. Medical director discovered cretinism in 90% of children in his village, when he analyzed the water
he found that it is deficient in iodine. The director wants to prevent and manage (the question was
asking about how to treat them) cretinism. What he is going to do initially? ( very common Q and
difficult to find source )
A. Iodine supplementation in water
B. Thyroxin supplement (levothyroxine).
C. TSH and t4 in 2 weeks.
D. No need to tt

Answer: B or A ( mostly B , not sure 100% )


Note : initial management should be supplementation of thyroxin, if the question was asking about a definitive
solution then we may choose to supplement water sources with iodine, however they are asking about initial
management.

192. What is the most common cause of hearing loss in children?


A. Eustachian tube dysfunction
B. Prenatal maternal infection
C. recurrent Otitis media with secretion
d. Rubella infection

Answer : C
Note : most common cause in newborn is Genetic 1 out of 2 , in older babies the most common cause is Recurrent
otitis media
Reffernce : Http://www.merckmanuals.com/home/children's-health-issues/ear,-nose,-and-throat-disorders-in-
children/hearing-impairment-in-children

193. Baby with rash in diaper area was going to different private hospitals which they gave him 3
different steroid with no improvement, when you examine him you noticed that he has sattallite spots
in the thigh and buttucks .what you will give ?

A. Antifungal
B. Topical steroid
C. Topical
antibiotics
D. oral antibiotic

Answer : A
Note : this case of ( diaper dermatitis ) if not improve with steroid we suspect candidiasis and also satellite lesions
give us hint .

194. 2 weeks newborn came with history of SOB , sweating in forehead, what cardiac anomaly he has?
A. VSD
B. ASD or TOF (one of them not sure) → ( asd, asymptomatic, tof : immediately after birth, severe
cyanosis is seen in patients with tetralogy of fallot and pulmonary atresia or severe pulmonary
stenosis)
C. Trans-position of great vessels → usually born at term, with cyanosis apparent within hours of birth.
D. PDA → 3 week to 6-week-old infants can present with tachypnea, diaphoresis, inability or difficulty with
feeding, and weight loss or no weight gain.

Answer : Not clear , may be Large VSD or TOF

195. treatment of neonate with apgar score 3 ?


A. Chest expansion
B. Ventilation
C. Iv fluids
D. no need intervention

Answer: B
Note : Ventilation and sometimes need chest compression if less than 3

196. Child with eye itching for one month (no other symptoms in scenario), and have history of asthma, what's
the diagnosis ?
A. Bacteria conjunctivitis
B. Viral conjunctivitis
C. Venral conjunctivitis
D. Blephritis

Answer: C
Note : asthma important hint for allergy , venral = allergy

197. Child had bee sting , No symptoms other than edema and erythema at the site of the bee sting.
Management will be ?
A. Oral steroid
B. Antihistamine
C. Admission and
observation
D. Epinephrine

Answer: A
Note : Antihistamine use if itching in sx , Epinephrine use if Anaphylactic shock . so the best answer from above A

198. Which of the following used in sickle cell anemia to increase production of HbF ?
A. Folic acid
B. Hydroxycarbamide
C. Vit.b12
D. Fresh fruit

Answer: B
Note : Hydroxycarbamide (hydroxyurea) , Decitabine , Butyrate all increase HbF .

199. Boy came for routine checkup, there was a murmur, ECHO done with only 2mm vsd what is your
management ?

A. Surgical repair
B. Watchful waiting
C. Give Medication
D. Cathetarization

Answer : A

Note :
Small <4 mm
Moderate 4 to 6 mm
Large >6 mm

200. Long history (case of meningitis) child receive IV penicillin G , LP showed gram –ve diplococcal , family
concerned about his young brother:
A. Admitted him to the hospital + observation
B. Give him iv penicillin
C. Oral Rifampicin
D. No thing

Answer: C
Note : Rifampin 600 mg (for children > 1 mo, 10 mg/kg; for children < 1 mo, 5 mg/kg) po q 12 h for 4 doses
Ceftriaxone 250 mg (for children < 15 yr, 125 mg) im for 1 dose
In adults, a fluoroquinolone (ciprofloxacin or levofloxacin 500 mg or ofloxacin 400 mg) po for 1 dose

201. Newborn totally healthy with left thigh bruise all examination normal.
Prolonged pt, ptt , ur diagnosis ?
A. Hemophilia
B. Factor 10 deficiency
C. Idiopathic thrombocytopenic
purpura
D. TTP

Answer: B

202. Child with dental caries, what u will advise him?


A. Oral antibiotic
B. Diet modification
C. Antiseptic mouth
wash
D. Diet high
carbohydrate

Answer : B ( not sure 100% ) may be C

203. Pediatric patient is always fatigued, he sleeps well at night with no orthopnea or PND . What is deficient
enzyme?
A. Pyruvate kinase
B. Pyruvate dehydrogenase complex
C. Pyruvate decarboxylase
D. Pyruvate hydroxylase

Answer: A
Note :

204. Asthmatic patient on inhaled corticosteroid and short acting beta 2 agonist which he use 3 times daily.
What is next step in the management?
A, oral steroid
B. Long acting beta 2 agonist
C. Interleukin inhibitors
D. Mast cell
stabilizer

Answer: B

205. Child had symptoms of mild infection ( I thing URTI ) 4 days ago. And taking antibiotics. Now he’s ok.
No fever good feeding and active. Came for routine vaccination. What will you do?
A. Give him the vaccine
B. Wait until he finish the antibiotics course
C. Give the vaccine except DTP
D. Posrpond all vaccines

Answer: A
Note : mild illness not contraindication for vaccination

206. Sign of fetal distress in CTG ?


A. less contraction
B. Early deceleration
C. Late
deceleration
D. good contraction

Answer: C

Note : A late deceleration is a reflex fetal response to transient hypoxemia during a


uterine , you have to see CTG ( late deceleration ) in Google image because They
give u the CTG and ask whats in this CTG .

207. 45 days old child k/c of G6PD deficiency. Presented with jaundice and bilirubin=25 and his weight at 10 th
percentile. What is the cause of jaundice ?
A. Physiological
B. Hemolysis
C. Low birth weight (birth weight was 2.51
k)
D. Metabolic

Answer: B
Note : Gucose-6-phosphate dehydrogenase (g6pd) deficiency is an x-linked enzymatic defect common in
blacks that can result in hemolysis after acute illnesses or intake of oxidant drugs (including salicylates and
sulfonamides).

208. Child with vomiting + diarrhea after meal... Had similar attack one month ago after he came from
camping with school. What is the most important thing help you in diagnosis?
A. Pus in stool.
B. History of dairy products.
C. Lactose breath test.
D. something not related

Answer: A ( most likely its chronic Giardiasis , so if stool exam for cysts in choices choose it )

209. Child 5 months, nurse said he has developmental delay in milestones, what would you do to reassure
the mother ?
A. Pincer grasp
B. wave bye bye
C. Reach things
C. Sit without support

Answer: C
210. 2 weekS old baby with strong cough and 2 episodes of him losing consciousness, on exam there is
intercostal retractions.
O2 sat was 90 , What do you do?
A. Inhaled steroids
B. Ventilator support
C. Antibiotics
D. B2 gonist

Answer: B or O2 mask

211. Neonate 2 week old with umbilical hernia and jaundice, what will you do?
A. Check newborn metabolic screening results
B. Baby’s TSH and t4 levels
C. Mother’s TSH and t4 levels
D. US

Answer: B (congenital hypothyroidism)


Note : Newborn screening detects the majority of infants with congenital primary
hypothyroidism infants with abnormal screening results are recalled for confirmatory
serum tsh and free t4

212. Child presented with erythematous pharynx, with cervical lymph nodes and rapid strplysin test negative
and low grade fever with positive EBV . It next step ?
A. Give antibiotics and anti-pyretic
B. Give antipyretic and fluids
C. Do culture and sensitivity
D. Give anti viral

Answer: B
Note : its case of infectious mononeoclusis and its tt :
Supportive: rest, hydration, saline gargle, analgesic for sore
throat
2-avoid contact sports if splenic enlargement for 6-8 wks.
If there is airway obstruction (due to tonsillar enlargement): admit + steroid
Acyclovir doesn’t reduce duration of sx and nor result in earlier return to school/work

213. About an adolescent who missed or didn't receive vaccine ?


A. 3 times 2 weeks apart
B. 3 times 4 weeks apart
C. 3 times 6 months apart
D. 3 times 4 months apart

Answer:
Example: between 1st and 2nd doses it’s often a minimum of 4 weeks apart except for hepatitis a (6 months
apart) and meningococcal (8 weeks apart) and varicella (3 months apart in younger than 13 years old) Catch
up immunization schedule according to vaccine link for:

214. Commonest sexual anomaly in boys ?


A. Hypospadias
B. Micropenis
C. Cryptorchidism
D. Epispadias

Answer: C
Note : Cryptorchidism is the most common congenital abnormality of the
genitourinary tract ( uptodate )

215. A case of absence seizure. Epidural fentanyl causes ? ( common Q )


A. Demyelination.
B. Neurotoxic metabolite.
C. increase stimulation of gamma receptors
D. decrease alpha receptor
Answer: C , both B and C happens but many sources talk about gamma receptor stimulation
216. A mother took her child to the well baby clinic, he was laughing with his mother. When the doctor came
he cried. How old is he?
A. 2 mo
B. 4 mo
C. 6 mo
D. 8 mo

Answer : C
Note : Starting laughing loudly spontaneously to familiar voice at 4m but development of stranger anxiety at 6m
(milestone: nilsson)

217. most common Side effect of DTP vaccine ?


A. Fever
B. Generalised body pain
C. Erythema at injection site
D. Shock

Answer: C
Note : most common C and pain at the site of injection then low grade fever .

218. Child with generalized tonic clonic 5min, fever 38 , what is best management ?
A.Control fever
B. Administer diazepam
C. Phenytoin
D. Phenobarbital

Answer: B
Note : this is case of Febrile seizure , the best in ER Lorazepam then Diazepam given till 3 times if the seizure not
stop then start Phenytoin 1 once then if not stop give phenobarbital then if not stop anesthetic and intubation ICU

219. Central treatment of child with kawasaki is ?


A. Aspirin
B. Iv immunoglobulin
C. Nsaid
D. Antibiotics

Answer: B
Note : IVIG to prevent coronary artery aneurysm , Aspirin use to tt Fever .

220. Child develop glove and stock numbness in foot and hand . Which is the best management:
A. Plasma exchange
B. Immunoglobulin
C. Steroid
D. some thing

Answer: B (if it is guillain-barré syndrome)

221. Child presented with runny nose.. Sore throat. . Feel of fullness of ears with no fever.. On
examination nose congested and erythematous tonsils.. Diagnosis ?
A. URTI
B. Croup
C. Otitis media
D. Epiglotitts

Answer: a
Note : Otitis media there is usually triad of otalgia, fever, and conductive hearing loss
croup presented with characteristic barking cough and fever
URTI could present with different non specific signs and symptoms and according to etiology it could be viral
with low grade fever or no fever.
221. Child presenting with a picture of intussception( usually between 3 to 12 months come with
abdominal pain and bloody stool and vomiting ) , which of the following is the most appropriate step
in management?
A. Barium enema,
B. Surgery,
C.
Observatio
n
D. US

Answer: D
Note : if dehydrated or unstable the most important step to give fluid , if not in choices or if the pt stable next step
US easy and fast and sensitivity reach 99% , but if ask about best test or management choose Barium enema

222. Child with ASD which finding on examination:


A. S3 with galob
B. Ejection systolic murmur
C. Fixed Split wide S2
D.S4

Answer: C, (fixed split s2 specific for asd and found almost in all patients with large left-to-right shunt)
Note : also ejection systolic murmur , in ECG : right axis deviation and RBBB

223. Fever, abdominal pain, watery diarrhea, vomiting , 12 y child Whats the microorganism ?
A. Rotavirus
B. Adenovirus
C. Norovirus
D. RSV

Answer: A , is the most common cause in pediatrics , in adult : Norovirus

224. Stiping up immedate after meal ,2 months old baby , all normal what will you do?
A. CT Abdomen
B. Rssurance
C. PPI
D. Surgery

Answer : B , only observation

225. Which congenital heart disease is the most common in down syndrome ?
A. Vsd.
B. Atrioventricular septal defect.
C. Coarctation of the
aorta.
D. TOF

Answer: B
Note : According to medscape the most common congenital cardiac defect is endocardial cushion defect
(43%), which results in atrioventricular septal defect (avsd)/av canal defect

226. 12-old boy came to the clinic with his parents , they are complaining that he is obese and has
strong appetite, all his milestones were 2-3 months later than normal , PE / small hands and feed
compared to rest of body , the cause of his problem is :
A. Genetic
B. Metabolic
C. Natural
D. Nutrition

Answer: A , case of Prader willi syndrome


227. 2weeks baby had UTI and sensitivity was +ve for amoxicillen,what investigation you should order
before starting tt ?
A. Renal us
B. Abdominal x ra
C. Urethral
culture
D. LFT

Answer: A
Note : All children under age of 5 and all male children should have a renal us if complain of UTI to
identify anatomical abnormalities including hydronephrosis , dilatation of the distal ureter , or bladder
hypertrophy and to rule out pyelonephritis .
VCUG indications :
Female <5 y with uti
Female > 5y presenting with second uti
All males
Febrile uti

228. Child fall on his abdomen and felt a severe right abdominal pain and tenderness, what is the
diagnosis ?
A. Liver contusion
B. Acute cholecystitis
C. Acute
pancreatitis
D. Gatritis

Answer: A
Note : ( Check the case in exam and the choices ) , The mechanism indicates a liver injury which happens
commonly after blunt injuries (spleen injury is more common with blunt trauma than liver injury). And the
severe pain might have resulted after bleeding into the peritoneal space causing peritonism.

229. Child present with signs of meningitis, what complication will suspected if patient not treated ?
A. Seizures
B. Meningioma
C. Vision loss
D. esophagus hemorrhage

Answer : A

230. Possible causes of child with distress, in bronchoscope take biopsy ?


A. Cystic fibrosis
B. Deficiency of anti trypsin
C. Loss of ciliary
D. Something
else

Answer : not clear


at all

231. How to prevent infection for small baby during examination ?


A. Wear gloves
B. Gown
C. Hand
wash
D. Mask

Answer: C

232. A neonate has eye infection gm -ve diplococci ?


A. Iv ciprofloxacin
B. Oral cephalosporin
C. Local erythromycin
D. IM 3rd generation Cephalosporine

Answer: D

233. A few months old baby came to receive his vaccination. The baby was delivered at 34 weeks’
gestation. The baby’s weight is low with a 90 g weight gain every month. Regarding vaccination
of this case ?
A. Delay 2 months from usual
B. As usual

C. Half dose
D. Give it now and delayed the other 


Answer: B
Note : The data indicate that baby should follow the same vaccination schedule as that generally used for full-
term infants, without correcting for prematurity and regardless of birthweight.

234. Patient with “pertussis” best swab from ?


A. Nasal swab
B. Nasopharyngeal
C. Tracheal
D. larynx

Answer B
The results of blood culture are uniformly negative because b pertussis grows solely in the respiratory
epithelium.
The culture specimen should be obtained by using deep nasopharyngeal aspiration or by holding a
flexible swab (dacron or calcium alginate) in the patient's posterior nasopharynx for 15-30 seconds or until
a cough is produced.

235. Q about truncus arteriosus & bulbus cordis , cause ??!!


A. Vsd
B. Asd
C. Tetralogy of fallot
D. Transposition of great artery

Answer: D , not clear but I think D

236. Child with nephrotic syndrome on steroid for 3-6wks or 6 months has vaccine ?
A. Give the vaccine
B. Stop the steroids
C. 3 month and give
appointment
D. No need to treat

Answer: C , check
choices in exam
Note : Routine childhood vaccines with live virus strains are contraindicated during steroid therapy and for a
minimum of 1 month afterward. But other than that it is not contradicted. It is recommended to give in
addition to the routine dead influenza yearly and pneumococcus vaccine every 5 years.

237. The most common complication of mump in pediatrics ?


A. Hearing loss
B. Encephalitis
C. Sterility
D. Meningitis

Answer: D , its meningitis not Encephalitis


Note : The most common complication in adult is orchitis.
238. The most common organism in whooping cough ?
A. Bortenella
B. Bordetella Pertusis
C. H. Infeluanza
D. Parainfluenza virus

Answer: B
239. Boy has a cat developed itching for a month with red eye and watery with discharge no
lymphadenopathy and general exam normal ?
A. Cat scratch
B. Dermtitis allergic
C.
Conjunctiviti
s
D.
Seborreic

Answer: B

240. Baby with vomiting and abdominal distension examination gush of stool history of decrease
feeding and constipation, history of barium enema what next ? ( check the case and choices in
exam may be hirschsprung )
A. Colectomy
B. Hydrostatic reduction
C. Leveling
colostomy
D.
Something

Answer: C

241. Asthmatic boy on muntelukast present to ER , he has symptoms everyday excacerbated


by exercise what to give for maintenance ?
A. Oral steroid daily with long acting when needed
B. Inhaled steroid twice with short acting when needed
C. Muntelukast with long acting
D. Theophyline

Answer: B

242. Child, obese, had left hip pain and limping with x-ray showing slept femoral hip, no hx of trauma, dx ?
A. Slipped capital femoral head
B. Fracture
C. Osteomyelitis
D. DDH

Answer: A
Note : SCFE : risk factors: male, obese
243. Six days infant irritable , poor feeding , twitching of muscles bulging fontanile what is next ?
A. Blood culture,
B. LP
C. IV antibiotic
D.oral antibiotic

Answer : C , ( not sure 100% )


Note : any change in baby behavior or symptoms during neonatal period its ( sepsis ) until proven otherwise so we
start empirical antibiotics

244. Child came complaining of pallor & fatigue. He has short stature & multiple cafe au lait spots over his
skin. Lab shows (WBC = normal, RBC = low, HB= low, Reticulocye = 9)
A. Iron deficiency anemia
B. Sca
C. Fanconi anemia
D.Thalassemia

Answer : C
Note : Fanconi anemia is aplastic anemia + congenital anomalies like short stature
Café au lait seen in fanconi anemia and neurofibromatosis

245. Parent bring their child after mid night (3 a.m.) Child complains of barking cough, dyspnea, fever &
inspiratory stridor,
o2 =92% in room air. Which of the following symptoms are of concern?
A. Flaring of nasal
B. Expiratory stridor
C. Blue color of lips
D. costal retraction

Answer
D. : C

246. A 18 month old child present to outpatient clinic having diarrhea & flatulence. Looking pale, below 25th
percentile for weight. Baby was completely normal at age 12 month before he weaned from breast feeding,
dx ?
A. Celiac disease
B. Cystic fibrosis.
C. Hirschsprung
disease
D. crhons

Answer: A

247. Lactating mother with active hepatitis B Virus ?


A. Encourage breast feeding
B. Stop breast feeding
C. See level of hbs ag then decide
D. shifted to formula

Answer: A
Note : All women with hepatitis b are encouraged to breastfeed their babies since the benefits of breastfeeding
outweigh the potential risk of transmitting the virus through breast milk. In addition, since all newborns should receive
the hepatitis b vaccine at birth, the risk of transmission is reduced even further.

248. Child can walk without support , crawling ,build 3 cubes point to something he interested in , so
what is the age of the child ?
A. 9 month
B. 15 month
C. 2 years
D. 3 years
Answer: B

249. Parents brought their baby to your clinic , you noticed testicular a symmetry and was tender on
palpation , the testes where palpable in scrotum bilaterally , next step :
A. Transillumination
B. Surgery
C. Watch it while the baby is craying and coughing
D. something

Answer: B ( depends on D , but mostly testicular torsion and we have to think about surgery )
Note : in the pediatric population, there is a higher likelihood for testicular torsion if the testis is high riding
compared with the other side. While abnormal lie can help in diagnosis, fewer than 50% of cases demonstrate
horizontal lie. In an older patient, a physical examination may reveal a swollen, tender, high-riding testis with
abnormal transverse lie and loss of the cremasteric reflex. Surgical exploration is mandatory unless torsion can
be excluded.

Reference: illustrated textbook of paediatrics 4th edition,


page 350

250. Baby on antibiotic developed watery diarrhea, what is most likely organism ?
A. Perferingas
B. C. Difficle
C. Rota virus
D. E.coli

Answer: B
Note : the diagnosis of pseudomembranous colitis caused by C. difficile colitis should be suspected in any
patient with diarrhea who has received antibiotics within the previous 3 months, has been recently hospitalized,
and/or has an occurrence of diarrhea 48 hours or more after hospitalization. Symptoms include: mild to moderate
watery diarrhea that is rarely bloody, cramping abdominal pain, anorexia, and malaise.
Reference: http://emedicine.medscape.com/article/186458-overview

251. 18 months old girl delivered premature her W.t was 2.6 kg .she is healthy but last 2 days c/o irritability
and fatigue .mother shift feeding from breast feed to cow milk feeding at age of 9 month , she has
hypochromic microcytic anemia, this patient complaint due to ?
A. Prematurity
B. Cow milk feeding
C. Bone marrow defect
D. the age

Answer: B
Note :: cow milk fed infants suffer from iron deficiency anemia due to low iron content in cow's milk.
Reference: nelson essentials of pediatrics 7th edition, page 97, table 31-1

252. Neonate with gradual cyanosis , in the beginning there is parasternal murmur without cyanosis, there
is right ventricular hypertrophy , right axis deviation, on x-ray there is small heart with pulmonary
vessel (something i didn’t recall) what is the diagnosis ?
A. Transposition of great vessels
B. Tetralogy of fault
C. ASD
D. VSA

Answer : A ( not sure 100% )


Transposition of great vessels Tetralogy of fallot
Most common cyanotic heart disease Most common cyanotic disease
presenting at birth
Single and loud s2 Paroxysmal hypercyanotic attack (tet spell)
Egg on string appearance in cxr Systolic thrill along left sternal border, harsh systolic murmur
along upper sternal border, boot shaped heart in cxr, right
axis
deviation on ecg
Reference kaplan step 2ck pediatric
2014

253. Child with bronchiolitis what is the treatment?


A. Ribavirin
B. Acyclovir
C. Other antiviral
D. Something

Answer: A ( the answer is supportive tt like O2 and fluid ) if not in choices choose : A
Explanation: the initial management of bronchiolitis is by means of supportive therapy. This is achieved by
maintaining
oxygenation, hydration, and ventilation in case apnea developed. Pharmacologic therapy has limited role in
management however ribavirin aerosol is indicated in following conditions to minimize or prevent need of
ventilation: congenital heart and pulmonary disease, immunosuppressed patients & infants less than 6 weeks.

254. A mother with hepatitis C , infant comes to your clinic asking about breastfeeding. How will you
advise her ?
A. Treat the Mother then breastfeed.
B. Continue breast feeding.
C. Stop breastfeeding.
D. breast feeding after drug 1 hour

Answer: B
Reference: http://www.cdc.gov/breastfeeding/disease/hepatitis.htm

255. 10 years old child with diarrhea. What is the correct about oral rehydration solution ?
A. Start with 50 in first 4 hour then maintenance 100 per day
B. Start with 50 in first 4hours then maintenance 50 per day
C. Start with 100 in first 4 hours then maintenance 100 per day
D. start with 100 in first 8 hours then maintenance 100 per day

Answer:
A

256. Pediatric with holosystolic murmur in left 3rd intercostal space, with hyper dynamic heart and loud S2
what’s the dx ?
A. VSD
B. ASD
C. PDA
D. Coarctation of aorta

Answer: A

257. What are the most common cause of failure to thrive ?


A. Psychosocial
B. Cystic fibrosis
C. Asthma
D. Bronchiolitis

Answer: A
Note : Child (usually infant) not fed adequate calories.emotional or maternal deprivation concurrent with nutritional
deprivation. Leads to neglect of infant; psychosocial deprivation most common reason in all age groups.for diagnosis
feed under supervision (may need hospitalization) for 1 week the baby should gain >2 oz/24 hours over the week.
258. Child with anemia a picture showed spherocytosis , what investigation you will do ?
A. Electrophoresis
B. Osmotic fragility test
C. G6PD levels
D. Enzyme assay

Answer : B
Note : Spherocytes are present in : hereditary spherocytosis, immune hemolytic anemia, post-transfusion. This is a
case of hereditary spherocytosis and to confirm the diagnosis osmotic fragility test should be preformed.

259. Child with arthritis , fever, epistaxis, gingival bleeding, results : platelets : is low , Hb is : low , whays
the investigation ?
A. Bone marrow aspiration
B. Electrophoresis
C. Anti dsDNA
D. ANA

Answer: A
Note : The causes of pancytopenia are: aplastic anemia, , myelofibrosis ,leukemia, TB , amyloidosis, sarcoidosis or
drugs e.g.
(Chemotherapy).

260. Child come to elective oral surgery ,all lab within normal, on examination there is murmur 2/6 best
hear in left sternum or ( suprasternal ) not sure , this murmur appear in sitting disappear in supine .
Otherwise normal: what the appropriate initial step:
A. Give antibiotics before surgery
B. Postpone surgery
C. Reassurance
D. IV antibiotic

Answer: C
Note : changing the child’s position from supine to sitting, then to standing, and finally to squatting during the
examination will change the flow and is useful in helping to define innocent murmurs, treatment is to
reassurance

261. Like mump senario :"can't remember " ?


A. Sajigran
B. Hypersensitvity vasculitis
C. Autoimmune....
D. stone

Answer: A ( not clear )

262. Mother changes her baby diaper many times a day. Labs all within the normal except (Na) was low. What is
diagnosis?
A. Acute proximal renal tubules acidosis
B. Acute distal renal tubules acidosis
C. Congenital chloride diarrhea
D. Renal failure

Answer: C ( not sure 100% )


note : Congenital chloride diarrhea: serum electrolyte levels may be within the reference range, especially in neonates
and treated patients. However, typical findings include low concentrations of serum chloride, sodium, and
potassium.
Renal tubular acidosis: serum sodium expect to be within normal ranges

263. 10 years old baby boy diagnosed with type 1 diabetes , presented to emergency department with 3 days
history of not feeling well and losing weight.
Investigations showed high blood sugar and ketone in the urine. What is best initial thing to do?
A. Electrolyte replacement
B. Fluid replacement
C. Insulin injection
D. Glucagone

Answer: B
Note : tt of DKA :
Step (1): always start with iv fluid then insulin
Step (2): correct potassium and bicarbonate
Step (3): monitoring therapy: should continue until resolution of dka. Criteria for resolution are:

Plasma glucose <11.1 mmol/l (<200 mg/dl) (at this point, insulin can be decreased by 50%)
Serum bicarbonate >18 mmol/l (>18 meq/l)
Venous ph >7.3
Anion gap <10.

Reference : Http://bestpractice.bmj.com/best-practice/monograph/162/treatment/details.html

264. Girl after eating seafood developed rash with severe itching and diarrhea, the mechanism of reaction:
A. Complement
B. Cell mediated
C. Immediate reaction
D. Delayed hypersensitivity

Answer: C
Note : Food-induced anaphylaxis: type i hypersensitivity reaction is mediated by food proteins binding to food-
specific ige, leading to mast-cell degranulation. Mast cells release mediators such as tryptase and
histamine, which give rise to the symptoms of anaphylaxis.
Typical symptoms include urticaria, angio-oedema, throat tightness, hoarseness, pruritus, vomiting, nausea,
diarrhoea, wheezing, respiratory distress, and hypotension, usually within 2 hours of ingestion of the inciting
food.
The most common food allergens in adults are peanuts, tree nuts, shellfish, and fish. In children, milk, eggs,
soya, wheat, peanuts, and tree nuts

265. Baby can walk when he held by one hand and good pincer grips but he cannot put things in the bottle. What
is his age?
A. 9 m
B. 12 m
C. 15 m
D. 24 m

Answer: A

266. child developed generalized edema with fever and dark urine, all labs normal except low calcium and
low albumin, what is dx?
A. Minimal change disease
B. Mesangial Disease
C. Nephropathy
D. May be Glomerulonephritis

Answer: D
267. Child with RTA had head injury, admitted to ICU. After 12hrs he give urine 100ml\h.
Lab show :
Sereum osmolarity : high
Serum Na : high
Urine osmolarity : low
What is the cause of this condition :
A. Central diabetes inspidus
B. Renal diabetes inspidus
C. Fluid overload in the icu.
D. SIADH

Answer: A
Note : this case of diabetes insipidus , central because of trauma in RTA , how we suspect Diabetes insipidus from
history and Lab the pt has polyuria , urine low osmolality and high Na in serum due to loss of fluid in urine

268. 2 month old with ida what is the route of treatment?


A. Oral ferrous sulfate
B. Parenatal iron
C.Transfusion of packed erythrocyte
D. IM iron

Answer: A

269. Case senario about child fall on his toys and twisted his ankle , they mention that he couldn't walk and
want to be held by his parents:
A. Soft tissue swelling of the ankle
B. Spiral fracture of tibia
C. Dislocation of fibula
D. Comminuted fracture

Answer : B

270. 13Years old girl has type 1 dm , her weight 40 kg ( below 50th percentile ) and his hight 150 cm ( below
95th percentile ) , she has no signs of secondary sexual characteristics of puberty, you want to perform
annual sceening in clinic for ? (common Q )
A. Ophthalmology
B. Growth hormone
C. Celiac screening
D. DM

Answer : A ( not sure 100% )

271. Boy fall on his mandibular, which muscle forceful close the jaw ? ( very common Q )
A. Masster
B. Medial Pterygoid
C. Temporalis
D. Lateral Pterygoid

Answer : A
Note : The masseter elevates the jaw which closes the mouth. The temporalis elevates and retracts the jaw. The
lateral pterygoid is the only muscle that actively opens the jaw. The medial pterygoid elevates and closes the jaw

272. Child plays with his father’s watch , once father check it, the battery were missing his child looks
normal but drooling , what is your action?
A. Endoscopy
B. Activated charcoal
C. X-ray chest ,abdomen
D. CT scan

Answer: A
Note : When a foreign body is strongly suspected on clinical grounds, visualization by endoscopy, which has the
added advantage of allowing removal of the object, may be the most efficient method of management.
as result, esophageal button batteries have emerged as the most critical indication for emergent endoscopy in
children"

273. 6 year old with duchene muscular dystrophy what to see ?


A. Atrophied muscles of lower limbs
B. Fasiculation
C. Gower sign’climb on him self
D. complete paralysis

Answer: C
Note : The gower sign is a classic physical examination finding in md and results from weakness in the child's
proximal hip muscles. Although the gower sign is a classic physical examination finding in duchenne md, it is by no
means pathognomonic; other types of md and disorders with proximal weakness may also cause this sign.

274. Pediatric case of broncheactitis whats the most important in tt ?


A. Physiotherapy
B. Home oxygen
C. Steroid
D. Antibiotics

Answer : D
Note : Antibiotics and chest physiotherapy are the mainstay modalities ( if antibiotics not in choices choose A )
Reference : https://www.medscape.com/answers/296961-7073/what-are-the-treatment-options-for-bronchiectasis

275. 12 yo with bilateral lower lung infiltration what is the treatment ? ( common Q )
A. Ciprofloxacin
B. Azithromycin
C. Penicillin
D. Cephosporine

Answer: B
Note : this case of a typical pneumonia ( age : between 5 and 15 and bilateral infiltration ) most common due to (
mycoplasma )
The tt is ( Macrolides like : Azythromycine , Clarythromycine , Erythromycine )

276. Child with paroxysmal cough, "whooping ‘’ what is the diagnosis ?


A. Pertussis
B. Epiglotittis
C. Croup
D. Pharyngitis

Answer: A paroxysmal cough (whooping cough) is characteristic of pertussis.

277. Child hemoglobin is low , RBC show spherocyte , what type of anemia?
A. Microcytic hypochromic
B. Microcytic hyperchromic
C. Macrocytic anemia
D. Normocytic Normochromic

Answer: D
Note : : RBC show spherocytes = hereditary spherocytosis (type of hemolytic anemia)
278. Radiosensitive testicular tumor ?
A. Seminomas
B. Yolk sac
C. Embryonal carcinoma
D. Non germinal

Answer: A
Note : radiosensitive tumours are tumours which respond well to radiotherapy.

279. 14 years old child known case of sickle cell disease , present to the ER due to abdominal pain , pale he
had a URTI few days back , vital :
Hb = 34 (normal 110-120)
Wbcs : high
Fever
Reticulocyte =8
Splenomegaly
what is the treatment ?
A. Splenectomy
B. Packed rbcs transfusion
C.
D. Analgesic and hydration

Answer : D
Note : most likely this ( Acute chest syndrome and the tt of it is ( supportive

280. A case if croup child is tachypnic , tachycardic , retrosternal retraction we started on racemic
epinephrine inhaled it is improving the symptoms initially what your next step ?
A. Antibiotic
B. Steroid
C. Oxygen
D. intubation

Answer : B
Note : if not improve or the symptoms return back we give Epinephrine again , but if symptoms improve we can give
steroid to decrease time of hospitilization

281. What is the reliable method to diagnose ALL ( acute lymphocytic leukemia ) ?
A. Lymph node involvement
B. Anemia and thrombocytopenia
C. Bone marrow blast cell
D. PET scan

Answer: C, bone marrow aspiration and biopsy (definitive for confirming leukemia)

282. 4 y/o boy thumb sucking what you will advise the mother ?
A. Observation
B. Palatal crib
C. Reward
D. Hits

Answer: C. Reward and positive reinforcement. Identify triggers. Offer gentle reminders.

283. Child can roll over, sit triploid, attempt to take object. Which month?
A. 6
B. 9
C. 2
D.
8

Answer : A
284. A baby setting in his mother’s lap unsupported. When the doctor spoke, the baby turned around,
laughed and babbled to the doctor. How old is the baby?
A. 2 months
B. 4 months
C. 6 months
D. 8 months

Answer: mostly B
Note : see Nelson Table

285. Fetal demise in 30 week. What's the cause?


A. Abnormal 2 Beta and normal 2 Alpha
B. Abnormal 4 Beta and normal 4 Alpha
C. Normal 4 Beta abnormal 4 Alpha
D. normal 4 Beta and 4 Alpha
Answer: C ( hydrops fetalis )

286 . 18 month-old female child presents with anemia. What is the most likely diagnosis ?
A. Homozygous β-thalassemia
B. Homozygous α-thalassemia
C. Carrier α-thalassemia
D. Carrier b thalassemia
Answer: A
Note :Β-thalassemia major manifests by age 1-2 years old with symptoms of severe anemia.

287. 6 years old child, came for pre school checkup, on exam he looks normal except for grade 3 heart
murmur along the sternal border “no specific location or timing of the murmur provided”, no thrill, the murmur
accentuated with supine position, dx ?
A. Still’s murmur
B. VSD
C. ASD
D. TOF

Answer: A
Note : The most common innocent murmur is a still murmur, which is characteristically loudest at the lower left sternal
border and has a musical or vibratory quality that is thought to represent vibrations of the left outflow tract.
Characteristics that are more likely to be associated with an innocent murmur include a systolic (rather than diastolic)
murmur; soft sound; short duration; musical or low pitch; varying intensity with phases of respiration and posture
(louder in supine position); and murmurs that become louder with exercise, anxiety, or fear.
Reference: http://www.aafp.org/afp/2011/1001/p793.html

288. What type of insulin is used in DKA ?


A. Nph
B. Regular( rapid )
C. Glarigine
D. Long acting

Answer: B
289. 11 years old child had severe diarrhea 3 weeks ago, now the child presented with bilateral lower
limbs weakness and numbness, dx?
A. Poliomylitis
B. Gullaine Barre syndrome
C. Muscular
dystrophy
D.Cerebral palsy

Answer: B
Note : The typical patient with gbs presents 2-4 weeks following a relatively benign respiratory or
gastrointestinal illness with complaints of finger dysesthesias and proximal muscle weakness of the lower
extremities.

290 . Child c/o fever, bloody stool, and tenesmus, abdominal exam
showed abdominal distention, Dx?
A. Ascaris
B. Amebiasis
C. Geirdiasis
D. Rota virus

Answer: B
Note : the hint in ( bloody diarrhea ) and ( abdominal distension )

291. A child with gum bleeding, erythema papules in mouth. Swab showed ( multinucleated giant cell ) ,
Which organism ?
A. Coxsackie virus
B. Staphylococcus
C.EBV
D. Herpes simplex

Answer: D
Note : multinucleated giant cell seen in Tzanck smear ( HSV and Herpes Zoster ,Varicella )

292. 5 years old girl presenting with hepato spleenomegally, pale, decreased level of consciousness , V\S
indicating shock, labs:
HB: 5 , platelets: 65,000 , what's your important next investigation?
A. Abdomen US
B. Reticulocyte count
C. BM
biopsy
D.LFT

Answer: B
Note : The patient has splenic sequestration. Splenic sequestration occurs with highest frequency during the first
5 years of life in children with sickle cell anemia. It is a medical emergency. This complication is characterized
by the onset of life-threatening anemia with rapid enlargement of the spleen and high reticulocyte count.

293. Case of thalassemia with splenomegaly + frontal protrusion


Labs: hb electrophoresis, HB A2 and HB F : high what is the diagnosis ?
A. B thalassemia major
B. B thalassemia minor
C. Alpha
thalassemia
D. Carrier
thalassemia

Answer: A
Note : Normally : HB A about 97% and other 3% is HB A2 and HB F, in Thalassemia Major : HB A = 0 and HBF =
more than 90% and HB A2 = more than 3.5 , While Thalassemia Minor : HBA = 80 – 95 % . HB F = 1 - 5% and HB
A2 = 3.5 – 7.5 %
394. Neonate developed cyanosis (2nd or 3rd week after delivery) and there is finding on on auscultation I
cannot remember exactly but they didn’t mention about machinery murmur the qs about the
management ?
A. NSAID
B. Steriod
C. Prostaglandin E1
D. Surgery

Answer: A , ( I think the case about PDA , during first 3 weeks starts with NSAIDS like Indomethacine ) to
close the opening without surgery . double check in exam

295. Child complains of abdominal pain for 2 weeks with diarrhea that is occasionally bloody and
malaise. What is the most appropriate test for diagnosis?
A. Abdominal ct
B. Barium enema
C. Abdominal us
D. Colonoscopy

Answer: B or C
Note : most likely its ( Intussception ) if unstable give fluid , for dx start with US and the best for dx and tt is :
Barruim

296. Cyanotic newborn with scaphoid abdomen. Heart sounds identified in the right side only.
What's the most likely diagnosis?
A. Dextrocardia with situs inversus
B. Transposition of great vessels
C. Diaphragmatic hernia
D. Hiatal hernia

Answer: C
Note : newborns frequently exhibit a scaphoid abdomen, barrel-shaped chest, and signs of respiratory distress
(retractions, cyanosis, grunting respirations). In left-sided posterolateral hernia, auscultation of the lungs reveals
poor air entry on the left, with a shift of cardiac sounds over the right chest.
Reference : Http://emedicine.medscape.com/article/978118-overview

297. Baby can sit roll from prone to supine and back play handle object but can't pick things by 2 fingers ,
age ?
A. 4 months
B. 6 months
C. 8 months
D. 10 months

Answer: B

298. Child with painful ear and runny nose and mild cough what You will give him ?
A. Antibiotic
B. Antihistamine and decongestant
C. Paracetamol
D. Steroid

Answer: A
Note : mostly signs of O.M or URTI

299. Child with cola colored urine, which test should you perform first?
A. Urinary microscopy.
B. Renal function test.
C. Renal biopsy.
D. Urine culture

Answer: A

300. Neonate was on breast feeding after 2 weeks has irritability and decrease feeding diagnosed with
meningitis, What is the organism ?
A. Niesseria Meningitidis
B. Listeria Monocytogenes
C. Strepococcal pneumonia
D. Gonorrhea

Answer : B
Note : sepsis and meningitis in newborn cased by : GEL ( GBS , E.coli . Listeria )

301. A woman has +ve Hep B surface antigen delivered a baby in a hospital and received
immunoglobulin and Hepatitis B vaccine within the first 12 hour ,, what are the recommendation for
breast feeding ?
A. No breast feeding
B. Breast feeding immediate before vaccines
C. Breast feeding after
12
D. After 24 hours

Answer: C
Note: mothers with hbv infection are free to breast feed their
Infants after the neonate has received the appropriate recommended vaccination
Reference: kaplan pediatrics

402. Treatment of meningitis in newborn ?


A. Ampicillin
B. Doxycycline
C. Ceftriaxone
D.Vancomycine

Answer: A

303. Child with URTI then complaining of bleeding from nose, gum and bruising the treatment is ?
A. Prednisolone
B. IVIG
C. Antibiotics
D. Supportive
Answer : B
Note : this is case of ITP ( come also with thrombocytopenia ) the treatment usually supportive but here there is
bleeding from nose and Gum other than skin , so in ER we give : IVIG

304. Most common cause of epistaxis in children ?


A. Self-induced picking ( trauma )
B. Polyps
C. Idiopathic
D. Bleeding disorder

Answer: A , if Trauma in choice and picking in choice choose nasal picking


Reference: Medscape

305. Late complication of meningitis in children?


A. Seizure
B. Cranial nerve palsy
C. Hemorrage
D. Hearing loss

Answer: A and D , I am not sure if D in choices but if both present A more likely to be the answer
Note :Seizures are a common complication of bacterial meningitis, affecting almost one third of the patients.
Persistent seizures, seizures late in the course of disease, and focal seizures are more likely to be associated
with neurologic sequelae.
Reference : Http://emedicine.medscape.com/article/961497-clinical#b3

306. Patient with lower limb weakness and sensation also angular stomatitis?
A. Vitamin B1 (thiamin)
B. Vitamin B3 (niacin)
C. Vitamin B9
D. Vitamin B12

Answer: D
Note : Vitamin B12 deficiency can present with lower limb weakness and loss of sensation with angular stomatitis.
Vitamin B2 and B3 can present with angular stomatitis but usually not with lower limb weakness. Vitamin B1
deficiency causes beriberi which causes mainly lower limb weakness and paresthesia or cardiac diseases (wet
beriberi).

307. Child develops petechia in the legs, he has history of URTI. Labs showed: normal CBC except platelet
count was : low. What is the next step in management?
A. Platelet transfusion
B. Immunoglobulin
C. Steroid
D. Supportive

Answer: D
Note : case of ITP tt is supportive there is no
bleeding Except the skin bleeding , no need to
IVIG

308. 6 years old boy with fever, sore throat, developed pink maculopapular rash all over his body and
pericarditis, what's the Dx?
A. Still' disease
B. Kawasaki Disease
C. measles
D. Rubella

Answer: B
Note : Kawasaki disease produces prolonged fever (often abrupt in onset and preceded by several days of
nonspecific symptoms) along with a constellation of clinical features that includes the following: Nonexudative
bilateral conjunctivitis (90%), Anterior uveitis (70%), Erythema and edema on the hands and feet; the latter
impedes ambulation, Strawberry tongue and lip fissures, Myocarditis and pericarditis
Diagnostic criteria established by the American Heart Association (AHA) are fever lasting longer than 5 days
and 4 of the 5 following main clinical features:
Changes in the peripheral extremities: Initial reddening or edema of the palms and soles, followed by
membranous desquamation of the finger and toe tips or transverse grooves across the fingernails and
toenails (Beau lines) Polymorphous rash (not vesicular): Usually generalized but may be limited to the groin
or lower extremities Oropharyngeal changes: Erythema, fissuring, and crusting of the lips; strawberry
tongue; diffuse mucosal injection of the oropharynx
Bilateral, nonexudative, painless bulbar conjunctival injection
Acute nonpurulent cervical lymphadenopathy with lymph node diameter greater than 1.5 cm,
usually unilateral

309 . 2 Years child with neck mass, has has ptosis , miosis , anhidrosis and
Heterochromia . What is your diagnosis (case of Horner's syndrome) ?
A.Wilms
B. Neuroblastoma
C. Ewing sarcoma
D. Osteosarcoma

Answer: B
Reference : Http://emedicine.medscape.com/article/988284-clinical

310. Child brought with diarrhea and abdominal pain. Distended abdomen lab otherwise normal. What is
the diagnosis?
A. IBD
B. Carbohydrates intolerance
C. IBS
D. Celiac

Answer: C
Note : IBS pt normal sometimes come with diarrhea or constipation and abdominal pain

311. Milestone: A child says baba and walks holding furniture, and a lot of other features, how old is this
child?
A. 12M
B. 10M
C. 15M
D. 18M

Answer: B
Note : Holding furniture 10M , walking 12M

312. Child with pain when chewing and headache, he complains loss of weight and decrease appetite ,
What is the complaint ?
A. Hearing loss
B. Facial palsy

Answer: Temporomandibular Joint Syndrome ? not clear plz read it well in exam
Reference: http://emedicine.medscape.com/article/809598-clinical

313. Child come with case of immune diffcancy with presented with lump in the groin and lap was given
what is the diagnosis ? Theres CH50 in Q ?
A. Severe combined immune diffciancy
B, Chronic granulamtae disise

Answer : may be B , Not clear and the choices not complete


Note : the presentation of lump make chronic granulamatouis disease more possible.
314. 2 y/o child had mass in flank lead to displace the collection system , Dx ?
A. Wilms
B.
Nuroblastoma
C.
Osteosracoma
D. Lymphoma

Answer: A
Note : check the scenario in the exam

315. Diet causes Kawashioker?


A. Low protein
B. High
Carbs
C. High fat
D. High
protein

Answer: A

316. Baby has sckle cell anemia and recive blood transfusion , what about vaccintion ?
A. Take vaccine
B. Dont give vaccie
B. wait one week
D. No need to vaccinate

Answer : A except MMR , precaution if blood transfusion within 11 months

317. 4 weeks old boy with acute onset forceful non bilious vomiting after feeding. On abdominal
examination: there is olive mass at epigastric area. What is the 1st investigation should you do? (
very common Q )
A. Ph monitoring
B. Abdominal us
C. X ray
D. Endoscopy

Answer: B , (the diagnosis of choice is US and the most accurate test is an upper GI series)
Note : Hypertrophic pyloric stenosis: when the vomiting persists, other clinical and biochemical findings may occur
such as dehydration, hypochloremic alkalosis and unconjugated jaundice. Late clinical manifestations include
weight loss and visible gastric peristaltic activity with a palpable pyloric ‘olive’. Prompt us diagnosis is important as
these late findings make the infants suboptimal candidates for surgery.

318. Child came with wheezing and cough , diagnosed to have Asthma and his Dr. Prescribed
beclomethasone space inhaler Twice daily. You will be worried about ?
A. Growth retardation
B. Extraocular problem
D. bleeding
D. O.M

Answer: A
Note : Corticosteroids inhalers can lead to Growth delayed and oral thrush (yeast infection of the mouth).

319. Child presented with Asthma exacerbation. The patient did not respond to beta 2 agonist. What is your
next step?
A. Aminophylline
B. Systemic steroid
C. Inhaled steroid
D. long acting B2 agonist
Answer: B
Note : in Asthma exacerbation in ER starts with O2 and short acting B2 gonist nebulizer then Systemic steroid

320. A child presented with fever and coryza, then watery diarrhea.
A.Adenovirus
B. Rotavirus
C. Noroway
D. EBV

Answer: A , Adenovirus cause URTI and GE

321. 8 years old boy with petechiae all over his body. Lab results: low platelets and high creatinine level.
What is the diagnosis ?
A. ITP
B. TTP
C. HUS
D. UTI

Answer : C
Note : HUS usually child come after GE the pt complains of bloody diarrhea then hematuria + low platelet and high
creatinine
TTP same presentation of HUS but in adult usually + neurological symptoms like seiure or headache

322. A child came to you with café au lait spots in face and neck. Which of the following features can
strengthen your diagnosis ? ( very common Q )
A. Port-wine stain.
B. Axillary freckling.
C. hair loss
D. Erythema nodosum

Answer: B
Note : Clinical diagnosis requires the presence of at least 2 of 7 criteria to confirm the presence of
neurofibromatosis 1 . The 7 clinical criteria used to diagnose nf1 are as follows, in the absence of alternative
diagnoses:
Six or more café-au-lait spots or hyperpigmented macules =5 mm in diameter in prepubertal children
and 15 mm postpubertal
Axillary or inguinal freckles (>2 freckles)
Two or more typical neurofibromas or one plexiform neurofibroma
Optic nerve glioma
Two or more iris hamartomas (lisch nodules), often identified only through slit-lamp
examination by an ophthalmologist
Sphenoid dysplasia or typical long-bone abnormalities such as pseudarthrosis
First-degree relative (eg, mother, father, sister, brother) with nf1

323. (long scenario) child brought by his father in wheel chair complaining of knee swelling and history
of falling on his knee. What is the best investigation ?
A. Joint aspiration.
B. X-ray.
C. ASO
D. MRI

Answer: B
Note : A plain radiograph of the affected joint should be performed to rule out fractures, periostitis, avascular
necrosis, bone tumors, and bone dysplasias.

324. What is the best initial investigation in a child presented with croup ?
A. X-ray
B. Pharyngeal swab
C. blood culture
D. clinical exam

Answer: D , the best to dx is clinical , if not in choices choose neck X ray ( Steeple sign )

325. Malnourished baby with fair coiled hair and abdominal distension. What is the most likely diagnosis?
A. Kwashiorkor
B. Marasmus
C. Celiac
D. Ulcerative colitis

Answer: A
Note :

326. Child suspect meningitis what to do immediately ?


A. Antibiotics
B. Lumbar
puncture
C. blood culture
D. CT scan

Answer: B
Note : do LP after rule out high intracranial pressure by fontanelles and fundocopy , then give Antibiotis

327. A child with flat buttocks. What investigation will you do ? ( suspect Celiac )
A. Anti TTG antibody
B. Intestine biopsy
C. Immunoassay
D. CT scan

Answer: A
Note : Antibody testing, especially immunoglobulin a anti-tissue transglutaminase antibody (iga ttg), is the best first
test, but the best in general : Biopsy

328. Child with chronic diarrhea , endoscopy showed sickle shaped parasite adherent to the bowl wall ,
what is it ?
A. giardia
B:entemebea histolytica
C. Rota virus
D. Shigella

Answer: A

329. Breastfeeding mother known history of seizure with phenytoin ask about breast feed ?
A. Reassuranse
B. Feeding after 8
hrs
C. contraindication of breast feeding
D. stop the drug

Answer : A

330. Young patient complaind of scrotal pain, examination is normal, US normal, urine analysis show
pyuria . What is the next step?
A. Refer to surgery
B. Give him azithromycine and
cefxime
C. KFT
D. Biopsy

Answer: B
Note : This is a case of epididydimitis
This patient has epididymitis. In males 14-35 years of age, the most common causes are neisseria gonorrhoeae
and chlamydia trachomatis. The recommended treatment in this age group is ceftriaxone, 250 mg intramuscularly,
and doxycycline, 100 mg twice daily for 10 days. A single 1-g dose of azithromycin may be substituted for
doxycycline. In those under age 14 or over age 35, the infection is usually caused by one of the common urinary
tract pathogens, and levofloxacin, 500 mg once daily for 10 days, would be the appropriate treatment.
But testicular torsion cause severe pain without urinary symptoms diagnosis by color doppler ultrasonography will
show a normal-appearing testis with decreased blood flow. And it need ergent surgical intervention

331. Picture of growth chart all parameters was low what is the dx ?
A. Genetic
B. GH deficiency
C. Celiac disease
D. Acondroplasia
Answer: depends on the chart
Note : If the weight and hight proportionally small >> chromosomal “genetic”
If the wight fall more that the hight “ftt” >> due to chronic illness , lack of intake
If hight fall more than the wight “shrt stature” >> endocrine .

332. Patient with barking cough .. what is the organism ? ( CROUP )


A. Parainflunza Virus
B. H.infelunza
C. RSV
D. EBV

Answer: A

333. 7 y child and an Upper & Lower respiratory tract infection since birth with generalized joint pain
aches his uncle and brother have the same condition labs show high creatinine and BUN and +ve
urine protein ? =====================
Idiopathic fibrosis
Autoimmune
Answer: ?

334. 12 year-old mild jaundice, splenomegaly , echogenic shadow of gall bladder Stone , dx ?
A. Sickle cell anemia
B. Thalassemia
C. Hereditary spherocytosis
D. Iron defeceincy Anemia

Answer: C
Note : why not A , SCA pt usually has autosplenectomy at 5 year old
335. Child ingested a caustic material he present to ER crying and drooling what to do 1st ?
A. Maintain airway
B. Activated charcoal
C. Bronchoscopy
D. X ray

Answer: A , always start with ABCDE

336. Adolescent male with swollen parotid and salivary gland, with dry eye and dry mouth, labs HLA,
ANA and RF are positive which of the following is appropriate treatment ?
A. Physostigmine
B. Artificial eye and saliva drops
C. anticiotics
D. Steroid

Answer: B , this is case of sjogren’s syndrome

Note : sjögren’s syndrome (ss) is a chronic autoimmune inflammatory disorder characterized by diminished
lacrimal and salivary gland function. The diminished exocrine gland function leads to the “sicca complex,” a
combination of dry eyes and dry mouth.
-ss is most common in women in their 50s and 60s but can affect adolescents and young adults, as
well as men. -prominent parotid and lacrimal glandular enlargement, may result from ss.
-anti-ro/ssa and/or anti-la/ssb antibodies, “rheumatoid factor and ana might be +ve but not might be -ve not
specific”.
-treatment of dry mouth due to salivary gland hypofunction aims to alleviate symptoms and prevent complications
with artificial tears and saliva.

337. 3 year old child with UTI admitted what investigation to be done ?
A. Us
B. Cystoscope
C. CT scan
D. DMSA

Answer: A
Note : any baby less than 5 years with UTI did : US

338 . Child with septic arthritis how to manage ?


A. Drainage and Abx
B. Broad spectrum IV
Abx
C. oral Abx
D. Steroid

Answer: A

339. Which of the is mostly associated sign with croup:


A.wheezing
B. Dysphonia
C. Cyanosis
D. Loss of conciocsness

Answer: B ( Hoarsness of voice )

340. Child ate a number of iron tablets presented with severe symptoms including constipation and
bloody stool , nausea and vomiting and drowsiness how would you treat him ?
A. IV desferoxamine.
B. Dialysis
C. induce vomiting
D. Active charcoal

Answer: A
Note : desfroxamine which is an antidote to iron (chelating agent) used to bring down iron levels, it is also used as
treatment of hemochromatosis (disease of iron over-absorption in the gut), another example to chelating agents is
penicilliamine used for the copper overload in wilson’s disease.

341. What is the commonest cause of HTN in adolescent ? ( common )


A. Idiopathic ( essential )
B. Renal cause
C. Renal artery stenosis
D. Polycystic kidney disease

Answer: A
Note : disease or renal artery stenosi (secondary to renal involvement) , Most childhood hypertension,
particularly in preadolescents, is secondary to an underlying disorder Renal parenchymal disease is the most
common (60 to 70 percent) cause of hypertension.
Adolescents usually have primary or essential hypertension, making up 85 to 95 percent of cases.23 table 323–25
shows causes of childhood hypertension according to age.

Source: http://www.aafp.org/afp/2006/0501/p1558.html

342. Girl came with upper arm bone pain investigation was given with anemia ( low HB ) and high bilirubin
and high reticulocyte count what most appropriate next step ?
A. Hb electrophoresis
B. Arm x-ray
C. Analgesic and hydration
D. Splenectomy

Answer: C , if not in choices choose A


Note : we are suspecting sickle cell anemia in this patient due to anemia (from sickle shaped rbc rupture), high
bilirubin (from rbc hemolysis) and high reticulocytes (young rbc trying to compensate rbc loss), and most importantly
bone pains that can be explained by the occurrence of a vaso-occlusive crisis known to happen to sicklers.

343. An infant came with cyanosis during feeding and crying (there was many thing in scenario) what is the
management:
A. Prostaglandin
B. Surgical repair

Answer : depend on case and dx


Note :
10% of all chd, most common cyanotic heart defect diagnosed beyond infancy
embryologically, a single defect with hypoplasia of the conus causing:
vsd + right ventricle (rv) outflow tract obstruction (rvoto) (e.g. Pulmonary stenosis) + overriding aorta + rvh
infants may initially have a l ~ r shunt and therefore are not cyanotic but the rvoto is progressive, resulting in
increasing r ~ l shunting with hypoxemia and cyanosis
history: hypoxic "tet" spells
primary pathophysiology is hypoxia, leading to increased pulmonary vascular resistance
(pvr) and decreased systemic resistance, occurring in exertional states (e.g. Crying, exercise)
paroxysm of rapid and deep breathing, irritability and crying
hyperpnea, increasing cyanosis often leading to deep sleep and decreased
intensity of murmur (decreased flow across rvoto)
peak incidence at 2-4 months of age
if severe may lead to seizures, loss of consciousness, death (rare)
management: 0 2, knee-chest position, fluid bolus, morphine sulfate, propanolol
physical exam: single loud s2 due to severe pulmonary stenosis (i.e. Rvoto)
investigations
ecg: rad, rvh
cxr: boot shaped heart (small pa, rvh), decreased pulmonary vasculature,
right aortic arch (in 20%)
treatment: surgical repair within first two years of life, or earlier if marked cyanosis, "tet" spells, or severe rv
outflow tract obstruction
ebstein's anomaly
congenital defect of the tricuspid valve in which the septal and posterior leaflets are malformed and displaced
into the rv leading to variable degrees of rv dysfunction, ts, tr or functional pulmonary atresia ifrv unable to
open pulmonic valves
ra massively enlarged, interatrial communication and patent foramen ovale (pfo) often exists allowing r ~ l
shunting
tr and accessory conduction pathways (wpw) are often present - often associated with arrhythmia
cause: unknown, associated with maternal lithium and benzodiazepine use in 1st trimester
treatment: in newborns, consider closure of tricuspid valve + aortopulmonary shunt, or transplantation • in
older children, tricuspid valve repair or valve replacement + asd closure
transposition of the great arteries (tga)
3-5% of all congenital cardiac lesions, most common cyanotic chd in neonate
parallel pulmonary and systemic circulations
systemic: body ~ ra ~ rv ~ aorta ~ body
pulmonary: lungs ~ la ~ lv ~ pulmonary artery ~ lungs • physical exam
no murmur if no vsd
newborn presents with progressive cyanosis unresponsive to oxygen therapy as the ductus
arteriosus closes and mixing between the two circulations diminishes; severe
hypoxemia, acidosis, and death can occur rapidly
if vsd present, cyanosis is not prominent and infant presents with chf after a few
weeks of life
investigations
ecg: rad, rvh
cxr: egg-shaped heart with narrow mediastinum ("egg on a string") • treatment

prostaglandin e1 (prostin vr'") infusion to keep ductus open until septostomy or surgery (arterial switch procedure)
infants without vsd must be repaired within 2 wks to avoid weak lv muscle

344. Baby was delivered 30 weeks and has respiratory symptoms what will be the main cause ?
A. Asthma
B. Decreased pulmonary
surfactant
C. Pneumonia
D. Amniotic aspiration

Answer: B
Note :In premature infant surfactant production is often inadequate to prevent alveolar collapse and
atelectasis, which result in respiratory distress syndrome

345. Baby 2 weeks old present with jaundice started 7 days ago , High indirect bilirubin 19.3... How to
manage?
A. Phototherapy
B. blood transfusion
C. Spleenectomy
D. Iron supplement

Answer: A

346. 4 months on breastfeeding, This is her first baby came with 2 days hx of lethargy constipation, fever,
response weak when light directed to his eyes (long scenario), cause ?
A. Hypothyroidism
B. Infantile
botulism
C. Gullain barre
D. Poliomylitis

Answer: B ,
Note : Infants typically present with constipation and poor feeding. This presentation is followed by progressive
hypotonia, and weakness. Loss of deep tendon reflexes appears to occur more commonly in type b infection [37].
Cranial nerve dysfunction is manifested by decreased gag and suck, diminished range of eye movement,
pupillary paralysis, and ptosis. Autonomic signs include decreased tearing and salivation, fluctuating heart rate
and blood pressure, and flushed skin , sometimes mention that the baby eat ( Honey ) , the hint here the baby is
the the 1st baby , so a lot of mothers give their babies honey without knowing that the honey is contraindicated in
1st year .

347. Developmental milestone expected in a 3 year old?


A. Climbs stairs
B. Catches ball with foot
C. hoping in one foot
D. Tell story

Answer: A

348. A case of pediatric nephrotic syndrome with edema ( Face edema ) and proteinuria without hematuria ,
what will you give him ?
A. Steroid trial
B. Diuretic
C. Antibiotics
D. Biopsy

Answer : A
Most common cause of Nephrotic syndrome in Pediatrics is ( Minimal change disease )

349. Mother with Group B Sterptococcus and had a baby who has irritability and agitation and fever. What
will you do ?
A. Give antibiotics
B. Do cultures
C. chest x ray
D. Urine analysis

Answer: A or B ( mostly A )
Note : the newborn has risk for sepsis we start with empirical ABx

350. When do Gilbert disease patients have jaundice?


A. At birth
B. With concurrent disease
C. While feeding
D. after eating

Answer: B
The most common inherited disorder of bilirubin glucuronidation is gilbert syndrome (also known as meulengracht
disease). Gilbert syndrome is a benign condition that has also been called "constitutional hepatic dysfunction" and
"familial nonhemolytic jaundice" . Although many patients present as isolated cases, the condition can also run in families.
It is characterized by recurrent episodes of jaundice and may be triggered by, among other things, dehydration, fasting,
intercurrent disease.

351. Baby, 3rd day after delivery got a purulent eye discharge what is the organism ?
A. Chlamydia
B. Gonorrhea
C. GBS
D. Listeria

Answer: : B (ophthalmia neonatorum)

Note : Gonorrhea ( presents during first 5 days of birth , purulent discharge )


Chlamydia ( presents after 5 days of birth , mucopurulent discharge )

352. 7 year girl with pubic hair growing, what is best investigation ?
A. Fsh
B. Ct scan
C. LH
D. US for pelvic

Answer: B , ( not sure 100 % )

353. Physician in the clinic tell the child to bend forward and hang his both hands freely. This test is used
in detection of ?
A. Rectal prolapse
B. Scoliosis
C. child abuse
D. BPH

Answer : B , Adams test forward bend test ( first step to dx Scoliosis )

354. Supra renal mass with mets to lung in child ?


A. Neuroblastoma
B. Wilms
C.
Lymphoma
D. Hepatoma

Answer: B
Note : Neuroblastoma : rarely metastasize to the lung , I think there is mistake in senario

355. Boy medically free , complaints of hematuria on urine analysis next step ?
A. Repeat urine analysis.
B. Bladder cystoscope
C. US
D. Biopsy

Answer : A
Note : This is a case of microscopic hematuria which mean : > 3 rRBC on urinalysis of at least
two separate sample Limited to these two choices bladder scop is not indicated in the
investigation for hematuria.
The most common cause of hematuria in children
is uti A source for the choice a could not be found

356. Patient with features of turner syndrome, they usually have:


A. Cardiovascular abnormality
B. High cholesterol
C. Hypertension
D. SLE

Answer: A ( most common Bicuspid aortic valve then Coarctation of Aorta )

357. Newborn with Red lump on left shoulder , what is the dx ?


A. Lipoma
B. Hemangioma

C. Meningioma
D. Cyst

Answer: B

358. Child complain of petechial rash all over the body . On examination there was a palpable spleen .
There is a history of urti, what’s the diagnosis?
A.ITP
B.HSP
C. Thromboembolism
D. Something

Answer: A ( depend on 4th choice , ITP not common to present with splenomegaly )

359. Preterm baby complain of sob, x-ray showed gross ground appearance + air bronchogram, this is due
to ?
A. Pneumonia.
B. Low surfactant. (ARDS)
C. Aspiration
D. Pneumothorax

Answer: B
n
Note from Fairst Aid : ARDS is the most common cause of respiratory failure in preterm infants.
It’s caused by surfactant deficiency which leads to: poor lung compliance, alveolar collapse, atelectasis

360. Initial management of juvenile Rheumatoid Arthritis ?


A. Intra articular corticosteroid
B. Paracetamol
C. NSAID
D. Oral steroid

Answer: C , if not in choices choose Aspirin if not in choices choose A

361. Baby 8 can sit without support, he can grasp, says mama, wave bye bye to the dr. Which
developmental milestone he has defect in?
A. Gross motor
B. Fine motor
C. Language
D. normal development

Answer : D

362. Child with long history of enuresis, what is the most important investigation to be done:
A. VCUG
B. Urinalysis
C. KFT
D. Urine culture

Answer: B
Note : Urinalysis is the most important screening test in a child with enuresis. Blood tests usually are not needed.
No imaging is needed if primary enuresis (PE) is suspected; however, radiologic evaluation might be warranted if
other conditions are being considered. This note from Medescape

363. Child present with crying and lobulated breathing + retraction of intercostal muscle what to do ?
A. Prepare for intubation
B. Clam the baby down

Answer : B but we have to know other choices , may be O2 one of them so it will be the answer
364. Sign of duodenal atresia in x-ray ?
A. Bird peak appearance
B. Double bubble appearance
C. one bubble appearance
D. Screw appearance

Answer: B

365. Child with hair loss spot, mother noted she was pulling her hair when stressed, what to give her?
A. Lithium
B. Lorazepam
C. SSRI
D. diazepam

Answer: C , SSRI like : citalopram, fluvoxamine, escitalopram, paroxetine, sertraline, fluoxetine but the
best is : Behavioral Intervention , so plz if it was in choices choose it
Note : this case of trichotillomania , Currently available evidence suggests that the first line of treatment for
trichotillomania is behavioral treatment and intervention

366. Child with itchy scalp and scales, other classmates are affected, what is the most likely diagnosis ?
A. Tinea capitis
B. Scabies
C. Tinea corporis
D. Pediculosis

Answer: A
Note : Tinea capitis is a highly contagious disease caused by superficial fungal infection of the skin of the
scalp scaling , eyebrows and eyelashes. Pruritus usually is minimal but may be intense at times. Alopecia
is common in infected areas , if only itching without scaling choose ( Pediculosis ‫ )القمل‬, if itching and
scaling also in arm and leg choose
( Tinea corporis )

367. Baby ride tricycle , draw circle but can’t draw square ?
A. 2 years
B. 3 years
C. 4 years
D. 5 years

Answer: A

368. Baby brought by his parents to er complaining of SOB & drooling saliva , Diagnosis:
A. Croups
B.
Epiglottitis
C.
Tracheitis
D.
Bronchiolitis

Answer : B

Note : The presentation of SOB with (drooling this is the hint ) points towards epiglottis which is an infection of
epiglottis in toddler and young school age. However croup usually presents with stridor and barking cough in
preschoolers and in winter usually. Epiglotittis come with toxic appearance and high grade fever while Croup low
grade fever , the signs in neck X ray
Epiglotittis : Thumb sign , Croup : Steeple sign

369. 2 year child didn't complete his vaccination ( 10 months is the last one ) present with fever
bilateral swelling periauricular , unable to swallow , dysphasia , enlarged tonsils and spleen
and lymph nodes...( no infectious mononucliosis nor mump in the choices) , your dx ?
A. Diphtheria
B. Streptococcus
pharyngitis
C. Coxsackie
D. Adenovirus

Answer: A
Note : Diphtheria pharyngitis will cause all the prementioned signs and symptoms in addition to a pathognomic
grayish membrane.

370. 15 Y/o amnorrhea ,short stature ,HTN ,broad neck ,also parent short stature , Dx ?
A. Familial
B. Turner syndrome
C. Down syndrome
D. Endocrine cause

Answer: B
Note : Turner features : Amenorrhea , short
stature , broad neck , coarctation of aorta ,
hypothroidism

371. 1 month old baby , has constipation science birth , best diagnostic investigation ?
A. Manometry
B. Full thickness intestinal biopsy
C. Full thickness Rectal biopsy ‘
D. Brrium enema

Answer: C
Note : hirschsprung disease should be considered in any newborn with delayed passage of meconium or in any
child with a history of chronic constipation since birth. The definitive diagnosis of hirschsprung disease is confirmed
by a full-thickness rectal biopsy demonstrating absence of ganglion cells.

372. Baby with blood jelly stool ( was case of intussusption ) what will u do ?
A. Us
B. Barium enema
C. CT scan
D. colonoscopy

Answer: A or B
Note : if the baby unstable start with fluid if stable and want initial step go with US ( Donghut sign) , if want the
best of definitive go with Barrium Enema , currant jelly stool is associated with intussusception. The traditional
and most reliable way to make the diagnosis of intussusception in children is to obtain a contrast enema (either
barium or air). Contrast enema is quick and reliable and has the potential to be therapeutic.
373. 5 years old baby presented with sever lower limb pain , growth parameters under the 5th percentile ,
low HB , dx ?
A. Osteomyelitis
B. Vasocclusive crisis
C. Acute chest syndrome
D. Splenic sequestration

Answer: B
Note : the lower limb pain (vaso-occlusive crisis), growth retardation, and low hemoglobin all are symptoms of
sickle cell disease. The most common clinical manifestation of Sickle cell disease is vaso-occlusive crisis but most
common cause of death in Sickle cell disease is Acute chest syndrome . During childhood and adolescence, scd is
associated with growth retardation, delayed sexual maturation, and being underweight.

374. Modified dukes criteria for diagnosis of Endocaritis is ?


A. 1 major 2 minor
B. 1major 3 minor
C. 3 Majors
D. 4 Minors

Answer: B
Note :
2 major criteria and 0 minor criteria or
1 major criteria and 3 minor criteria or
0 major criteria and 5 minor criteria

375. What is the type of murmur in atrial septal defect ( ASD ) ?


A. Fixed split in s2
B. Ejection systolic click
C. Holosystolic murmur
D. Fixed split in S1

Answer: A
Note :
Auscultation in septal defects
Atrial septal defect Fixed wide splitting of s2
Ventricular septal defect Harsh holosystolic murmur loudest at tricuspid area

Ejection systolic clicks are associated with congenital aortic or pulmonary


stenosis.

376. Asystole in child , the first treatment ?


A. Epinephrine
B. Atropine.
C. Lidocaine
D. Something

Answer: A , if D is ( chest compression or CPR ) choose it


Note : epinephrine remains the drug of choice for asystole in children. Atropine is not indicated.)

377. A three weeks old boy presented with scrotal asymmetry. In examination both testicles were palpable
in the scrotum.
(that’s it, with no more information). What are you going to do next in examination ?
A. Trans-illumination test
B. Let the patient to cry/cough to see if the is any bulging
C. examine the neulogical sx
D. Percussion

Answer: A
Note : to exclude the Hydrocele
378. Say few words at which age ?
A. 24m
B. 12m
C. 34 m
D. 48 m

Answer: B

379. Neonate with bilious vomiting , seed stool diarrhea he passed meconium in day 2 what is the cause ?
A. Hirschsprung disease
B. Allergy to formula milk
C. pyloric stenosis
D. Duedenal atresia

Answer: B ( if the pt complains of diarrhea and pass stool in 2nd day this go with B ) ( if constipation then
it’s A )

380. Newborn girl ( i did not remember the complain but there is enlargment of clitoris ( I think this is
congenital adrenal hyperplasia CAH ) ask about treatment ?
A. Fluid
B. Hydrocortisone
C. Estrogen
D. Surgery

Answer: B ( for treatment of CAH , but if complaint is dehydration, answer is A ) read it well in the exam

381. Long history of child with history of infectious mononucleosis ( upper respiratory
tract infection with hepatosplenomegaly and tonsil enlatgement and
lymphadenopathy ) what is your next step ?
A. CBC
B. Monospot
test
C. sputum
culture
D. CT scan

Answer : A , if ask about best test or give u CBC result and ask what is next the answer will be : B

Note : Heterophile test antibodies (e.g:monospot test)are sensitive and specific for ebv heterophile antibodies,
they are present in peak levels 2-6 weeks after primary ebv infection, and they may remain positive in low
levels for up to a year.
Reference: http://emedicine.medscape.com/article/222040-overview

382. Child presented with convulsion after Gastroenteritis ?


A. Salmonella
B. Shigella
C. E.Coli
D. Rota

Answer: B
Note : Generalized seizures have been reported occasionally among young children with shigellosis, and
usually resolve without treatment. Children who experience seizures while infected with shigella typically
have a high fever or abnormal blood electrolytes (salts), but it is not well understood why the seizures
occurs.
Reference : Http://www.cdc.gov/shigella/general-information.html#definitions-symptoms
383. A boy draws his brother .till a story. Play a role as father how old is he in years ?
A. 4
B.
5
C.
6
D.
2

Answer : A

384. Mouth ulcer can caused by ?


A. HSV1
B. HSV2
C. EBV
D. Asthma

Answer: A
Note : HSV type 1 is usually associated with orofacial disease, and HSV type 2 is usually associated with genital
infection

385. 7 days baby weight 3.02 Kg was 3.5 Kg when born , mom is concern about her baby weight, although
she feed him well , what you will do ? ( common Q )
A-reassure the mom it's normal
B-routine test and reassure
C. give him formula feeding
D. Lactose tolerance test

Answer: A
Note during first week the baby may lost up to 10% of his weight due to loss of fluid and other , by 10 days will gain
it again

386. 11 months old infant , dark foul smell stool mixed with blood and mucus , what will u do investigation
? ( I think case of intussuption ) we talked about it a lot , see Qs above
A. Technicium scan
B. Ultrasound
C. Fluid
D. Barrium enema

Answer : C , start with fluid esp. if there is signs of dehyrayion , if no choose B as initial and D as the best

387. Cyanotic heart disease?


A. Coarctation of the aorta
B. Truncus arteriosus
C. VSD
D. PDA

Answer: B

388. Milestone : child crawl , hand to hand but no pincer grip , what is the age ?
A. 7 m
B. 9 m
C. 10 m
D. 12 m

Answer : A
389. Child comes to vaccination , his Sister recently kidney
transplant, which vaccine should he Avoid ?
A. OPV
B. Measles
C. MMR
D. IPV

Answer : A
Note : Administration of OPV is associated with a low incidence of paralytic poliomyelitis in vaccines. Also,
individuals in close contact with recently inoculated vaccines may be at a small risk of developing paralytic
poliomyelitis because poliovirus can be shed in the feces (and possibly from the pharynx) for 6-8 weeks after OPV
administration.

390. Child with mild intermittent asthma, he is using short acting B2 agonist 1 time\ week. Nowadays, he is
using it 4 times \ week without any benefit, what will you add ?
A. Long acting b agonist
B. Low dose inhaled steroid
C. Oral steroid
D. Ipratropium bromide

Answer: B
Note : in Asthma starts with Short acting B2 agonist then Add low dose inhaled steroid then add Long acting
B2 agonist or increase the dose of steroid to medium inhaled then to High dose inhaled steroid then we can
add others , last step : Oral steroid .

391. Neonate + Bilateral white eye reflex. Posterior segment information not available ? ( common Q )
A. Congenital cataract
B. Congenital
glucoma
C. Retinoblastoma
D. optic neuroma

Answer: C
Note : white eye reflex seen in : Retinoblastoma , Absence of eye reflex seen in : Congenital cataract , if
Retinoblastoma not in choices choose A
392. Child with OM take amoxicillin and follow up now with outpatient Clinic on examination there is fluid
behind tympanic membrane ?
A. Amoxicillin
B. Wait and watch
C. Tympanoplasty
D. grommet insertion

Answer: B

393. Young child present with his family (a picture of baby, his age seems to be in months, his head is tilted
to one side). Baby was normal at birth with normal non-complicated delivery, when you try to stretch
sternocleidomastoid the baby begins crying. All reflexes and movements are normal. What is the
diagnosis?

A. Cervical rib
B. Infant torticollis
C. Clavicular fracture
D. Erbs Palsy

Answer: B
Note : Cervical rib is mostly asymptomatic in individuals who have it, and it is almost always an incidental
finding. Symptoms rise in case of trauma or injury leading to thoracic outlet syndrome.
The cause of congenital torticollis in infants is not known. Intrauterine and perinatal events that have been
proposed to contribute to its development include malposition of the head in utero and injury to the
sternocleidomastoid muscle (scm) that results in fibrosis.
Torticollis typically develops by two to four weeks of age, after which the child prefers to hold the head tilted
to one side.

394. 2 days old baby with urine smelling like burned sugar. What’s the diagnosis? ( Very common Q )
A. Maple syrup disease
B. Phenylketonuria
C. Cystic disorder
D. Galactossemia

Answer: A

395. Child presented to er with hx of testicular pain, what is next step?


A. U/S
B. Surgery counseling
C. Analgesic and discharge
D. Antibiotic and culture

Answer : B
Note : Testicular torsion is a clinical diagnosis. If the history and physical examination strongly suggest testicular
torsion, the patient should go directly to surgery without delaying to perform imaging studies.
When a low suspicion of testicular torsion exists, color doppler and power doppler ultrasonography can be used to
demonstrate arterial blood flow to the testicle while providing information about scrotal anatomy and other testicular
disorders.

396. Baby 18 month old with delay speech other exam normal what next ?
A. Hearing
B. Development Exam
C. Language test
D. something

Answer : A

397. Case of 2 months baby , his parent feeds him with bottle of milk while he sleeping , developed
with plaque and discoloration of his teeth , what is the cause ?
A. Nursing bottle carries
B. Dental deformating
C. Something
D. something

Answer : A ( check the choices )

398. Baby was pale with petechial rash , side effect of which drug ?
A) chlormphenicol
B) vancomycin
C. Amoxicillin
D. Clindamycin

Answer : A ( it causes Aplastic Anemia )

399. Child with earache and fever ?


A. URTI
B. Otitis media with effusion
C. Otitis media
D. Otitis externa

Answer : C

400. Child with arthritis, fever, epistaxis, gingival bleeding, results


Platelet is : low , HB : Is low.. Appropriate investigation ?
++++++++++++++++++++++++++++++++++++
A. Bone marrow aspiration
B. Electrophoresis
C. Anti dsdna
D. Something

Answer :
Answer: most important is blood smear not mentioned in choices
Explnanation: http://bestpractice.bmj.com/best-practice/monograph/795/diagnosis/differential-diagnosis.html

401. African teen with jaw mass histopathology showed starry sky appearance (case of burkitt's
lymphoma) what's that pathophysiology?
A. Overexpression of c-myc
B. Overexpression of n-myc
C. Overexpression of Fab
D. Overexpression of k-ras

Answer: A
Note : All types of burkitt lymphoma are characterized by dysregulation of the c-myc gene by one of three
chromosomal translocations t(8:14), the most common. Or less commonly t(8:2) and t(8:22). Overproduction of the c-
myc product may also change the lymphocytes into cancer cells, other Q : Starry sky appearance and asl what is
your dx ? its Burkitts Lymphoma

402. ( long scenario), child with pansystolic (holosystolic) murmur. What is the cause?
A. ASD
B. VSD
C. Pulmonary Atresia
D. Aortic stenosis

Answer: B
Note : causes of Pansystolic ( holosystolic ) murmur : VSD , Mitral Regurge , Tricuspid Reguge .

403. Child come with watery diarrhea during the stool microscope you found a flagellated parasite. What is
the mechanism of the diarrhea ?
A. secretions a lot of fluid
B. decrease the absorption of fluid
C. increase th absorption
D. Decrease the secretion of fluid

Answer: B
Note : marked or moderate partial villous atrophy in the duodenum and jejunum can be observed in histologic sections
from asymptomatic individuals who are infected. In addition to disrupting the mucosal epithelium, effects in the intestinal
lumen may contribute to malabsorption and the production of diarrhea. [5, 19] nevertheless, diarrhea can occur in
individuals in the absence of obvious light microscopic changes in small intestinal structure.
Varying degrees of malabsorption of sugars (eg, xylose, disaccharides), fats, and fat-soluble vitamins (eg, vitamins
a and e) may contribute to substantial weight loss. The histopathologic response to giardiasis varies and
imperfectly correlates with the clinical symptoms

404. Child present with abdominal mass and some urinary symptoms but i can not remember after ct the
report say that there is a large mass involved the renal collecting system what your next step ?
A. 24 hr collection of catecholamine
B. MRI
C. Biopsy
D. MCUG

Answer : C
Note : ( not sure 100% if it was in choices , if case go with Wilms and biopsy in choices choose it if not in choices
choose MRI )

405. Safe vaccine you can give it to immunocompromised patient:


A. BCG
B. HBV
C. OPV
D. MMR

Answer: B , safe Vccines : pneumococcal, meningococcal, Hib and any killed vaccines
Note : Contraindicated: all live vaccines. Oral polio, Rota , MMR, smallpox , BCG , varicella , OPV

406. Child with vomiting presented dehydrated and a small mass was felt in the epigastric area, what is his
acid-balance ?
A. Hypochloremic Hypokalemic metabolic alkalosis
B. Metabolic acidosis
C. Hyperkalemic metabolic alkalosis
D. Hypokalemic metabiloc acidosis

Answer: A
Note : The patient is having (pyloric stenosis), also known as infantile hypertrophic pyloric stenosis (IHPS), is the
most common cause of intestinal obstruction in infancy causing functional gastric outlet obstruction. Hypochloremic
hypokalemic metabolic alkalosis is the classic electrolyte and acid-base imbalance of pyloric stenosis
Classically, the infant present with non-bilious vomiting or regurgitation, which may become projectile.

407. 18 months baby says baba mama only , what you will do for him ?
A. Developmental assessment
B. Bone age
C. hearing assessment
D. Something not related

Answer: C ( not sure 100% )


Note : Baby at 18 months should say several single words. Saying mama baba should be at 12 months , I am not
sure if hearing assessment in choices but if it was in choices may be it is the answer , I go with it , if not in choices
choose A
408 . Pediatric patient is complaining of Red eye and fever, later he develops pink rash on the face which
spreads to upper and lower limbs, there is also white papule in the mouth, what is the diagnosis?
A. Rubella
B. Meningococcal
rash
C. Measles
D. HSV1

Answer: C
Note : most likely Measles , the hint is : white papule ( kopliks spots ) in mouth .

409. Child presented to clinic, very tall and very thin ( not the exact wording of the question ) .exam
showed cardiac abnormality and pectus excavatum, don’t remember other details. Dx ?
A. Marfan
B. Charg
syndrome
C. Turner
syndrome C.
Prader willi

Answer : A
Note : Marfan syndrome hints : Tall , Thin , and cardiac anomaly

410. 7-year girl developed pubic hair and axillary hair and complain from acne and breast develop not
enlargement of clitoris what dx ?
A. Central cause of puberty (something like that )
B. Gonadotropin realizing tumor
C. Something
D. something

Answer : A !!! I am not sure depends on other choices

Note : usually idiopathic in females (90%), more suggestive of pathology in males (50%)
central (GNRH dependent)
peripheral (GNRH independent)
Hypogonadotropic hypergonadism
Differential diagnosis: adrenal disorders (cah, adrenal neoplasm), testicular/ovarian tumor, gonadotropin/hcg
secreting tumor (hepatoblastoma, intracranial teratoma, germinoma) exogenous steroid administration, mccune-
albright syndrome, aromatase excess syndrome, rarely hypothyroidism (van wyk-grumbach syndrome).

411. 16 y/o female presented with history of irregular period , that came every 5 weeks then, 6 weeks ?
A. Normal changes or physiological changes
B. Uterian fibroids
C. Endometriosis
D. turner syndrome

Answer : A , I think who deliver the Q confuse in number of weeks

412. Child with severe rheumatic fever &cardiac involvement what to give for short period ? ( common Q )
A. Im penicillin monthly
B. Large does aspirin + Orally steroid
C. Aspirin
D. Penicillin daily

Answer: B
Note : The primary goal of treating an arf attack is to eradicate streptococcal organisms and bacterial antigens
from the pharyngeal region. Penicillin is the drug of choice in persons who are not at risk of allergic reaction. A
single parenteral injection of benzathine benzylpenicillin can ensure compliance. Oral cephalosporins, rather
than erythromycin, are recommended as an alternative in patients who are allergic to penicillin.
Corticosteroids should be reserved for the treatment of severe carditis. After 2-3 weeks, the dosage may be tapered,
reduced by 25% each week , Aspirin use for arthritis

413. 4 days baby present with bilious vomiting, he had abdominal distention, poorly feed on
examination abdominal distention (no other findings in examination in the q)
A. Midgut volvulus
B. Allergic to
formula
C. Duedenal
atresia
D. Pyloric stenosis

Answer : A
Note : Midgut volvulus onset b/w 3 to 7 days after birth , Duedenal atresia usually within hrs of birth

414. Child with recurrent URTI with pseudomonas, and atypical organism. Whats the cause ?
A. Cystic fibrosis
B. Low CD4
C. Something
D. something

Answer: A
Note : Typical respiratory manifestations of cf include a persistent, productive cough, hyperinflation of the lung
fields on chest radiograph, and pulmonary function tests that are consistent with obstructive airway disease. The
onset of clinical symptoms varies widely, due to differences in cftr genotype and other individual factors, but
pulmonary function abnormalities often are detectable even in the absence of symptoms. As an example, in a
cohort of infants largely identified by newborn screening, 35 percent had respiratory symptoms (cough,
Wheezing, or any breathing difficulty); mean pulmonary function scores were abnormal by six weeks of age and
declined during the subsequent two years.
Transient infection of the airway with pathogenic bacteria often occurs early in life. Eventually, over years and
varying widely among individuals, chronic airway infection with either staphylococcus aureus or gram negative
bacteria is established, often with radiographic evidence of bronchiectasis. S. Aureus and nontypeable haemophilus
influenzae are common pathogens during early childhood, but pseudomonas aeruginosa is ultimately isolated from
the respiratory secretions of most patients.

415. Child with mild trauma develop hemarthrosis, in past history has of similar episode , your dx ?
A. Platelets dysfunction
B. Clotting factor
def
C. Platelet
deficiency
D. Leukemia

Answer : B ,
Note : Hemophilia two types : A deficiency in factor 8 , B : deficiency in factor 9 , most common symptom is
Hemarthrosis

416. CPR in child according to American heart association in presence of 02 rescuer is ?


A. 30 compression 2 venilation
B. 30 compression 1 venilation
C. 15 compression 2 ventilation
D. 15 compression 1 ventilation
Answer : C
Note : if 1 rescuer the answer will be : A
417. A child who ate honey develops progressive paralysis symptoms , what is your dx ?
A. C. Botulism
B. C. Perfringens
C. C. Difficile
D. Gulaine barre syndrome

Answer: A

418. Child take overdose of isoniazid and toxicity what is the symptoms ?
A. seizure or status epilepticus
B. meningitis
C. respiratory hyperventilation
D. diarrhea

Answer : A

419. A child with sickle cell anemia and bilateral hip pain. What is the possible diagnosis? ( common Q )
A. Avascular necrosis of femoral head
B. Ostomylitis
C. Slipped femoral
D. DDH

Answer: A

420. Intranasal influenza vaccine , the name of the vaccine is ?


A. Live attenuated influenza vaccine (LAIV)
B. Killed
C. Toxoid
D. Polysaccharides

Answer : A , trade name : flue Mist

421. Patient with thumb sign on X-ray what is your treatment ?


A. Ceftriaxone and clindamycin
B. Penicillin
C. Doxycycline
D. Ciprofloxacine and Gentamicine

Answer: A
Note : the first priority in treatment of Epiglotittis in ER is secure the airway usually by intubation , after that give
antibiotics for 7 to 10 days to cover Group A streptococcus , staphylococcus pyogenes and H.ifluenza , the most
importand antibiotic is 3rd generation Cephalosporin ( Cefriaxone ) and ampicillin and Clindamycin .

422. Mother brings her first & only infant was fatigued, not move or gaze from light direct on his eye
what the Dx?
A. Polio
B. Polymyelitis
C. Gulaine barre
D. Infantile botulism
Answer : D
Note : the hint is ( only infant ) so the mother doesn’t know the complication of honey

423. Child presented with cyanosis and murmur in newborn , what is your dx ?
A. TGA
B. VSA
C. ASD
D. TOF
Answer : A

424. TOF present with which of the following defects ? ( very very common Q )
A. ASD
B. VSD
C. Aortic stenosis
D. Coarctation of aorta

Answer : B
Note : TOF components : VSD , Pulmonary stonsis , Overriding aorta , Right ventricle hypertrophy
Very verty very common Q , u have to know all above defects in TOF

425. Child 9 years old with severe avascular necrosis , what is the treatment ?
A. medical treatment
B. Observation
C. Decompression
D. Physiotherapy

Answer : C ( not sure of choices and answer )

426. Pedia case neonate has jaundice admitted due to physiolgical jundice then disharge , still appear
jundice what is the cause ?
A. Breast milk jaundice
B. ABO incompatability
C. RH isoimmunization
D. Duednal atresia

Answer : A (not sure of choices )


Note : most common cause of prolonged jaundice is : breast milk jaundice , other causes : hypothyroidism ,
Galactossemia , sepsis , Cliglar najjar .

427. The most common parotid tumor in pediatrics ?


A. Pleomorphic adenoma
B. Hemangioma
C. Lymphoma
D. Warthins tumour

Answer : A
Note : Mucoepidermoid the answer if they asked about the most common malignant tumor.

428. Baby with greasy looking rash on face , what is most likely diagnosis ?
A. Seborrheic dermatitis
B. Acne
C. kertodermatitis
D. Something

Answer: A ( usually in newborn and on forehead )

429. What is the most common site for mump ?


A. Parotid
B. submandibular
C. submental
D. testes

Answer: A

430. Typical case of post streptococcal glomeruloNephritis (hematuria after 1 to 2 weeks of URTI ). What will
you do to confirm diagnosis?
There is no choices but the answer must be : Biopsy if not in choices choose ( high ASOT ) if not in choices choose
low C3

431. The most common chromosomal abnormality in a new infant is ?


A. Pateu syndrome
B. Turner
C. Marfan
D. Down syndrome ( trisomy 21 )

Answer: D
Note : Most common abnormality of autosomal chromosomes.

432. Child was having rhinorrhea and then developed episodes of cough followed by vomiting. Which of
these vaccination may prevent him from having this disease ?
A. Dtap
B. OPV
C. Hib
D. Influenza vaccine

Answer: A ( Case of Pertusis prevented by pertussis vaccine )

433. (a case of pertussis) how to diagnose of pertussis?


A. Nasopharyngeal swab
B. Oropharyngeal
C. Trachea
D. Lung

Answer: A
Note : Gold standard for dx of whooping cough ( Pertusis ) is culture from Nasopharyngeal

434. Tetralogy of fallot finding defects ( very very common as we said ) :


Ventricular septal defect (vsd) + overriding of the aorta + pulmonary stenosis + right ventricular hypertrophy (rvh)

435. A case of TOF . How does it appear on x-ray and echo ?


ECG : right axis deviation, rvh
CXR : boot shaped heart, decreased pulmonary vasculature, right aortic arch (in 20%)
436. Turner feature is :
A. Thick skin neck
B. Tall stature
C. Hyperthyroidism
D. Long arms

Answer: A
Note : Short stature, low set mildly deformed ear, triangular face, flat nasal bridge, epicanthal fold, web neck with or
without cystic hygroma, shield like chest with wide internipple space, puffiness of hands & feet, internal malformation
mainly coarctation of aorta in heart, horseshoe kidney.

437. A child was on clindamycin developed abdominal pain and watery diarrhea , what is the cause ? ( very
common Q )
A. C. Perfenges
B. C. Difficile
C. Streptococcus
D. Staphylococcus

Answer: B
Note : case of Pseudomembranous colitis caused by C. Difficile after taking ABX usually after course of : Peniciilin or
clindamycin or other ABXs .

438. Child came with right abdominal pain, jaundice, palpable tender liver, what is your dx? ( common Q )
A. HAB
B. HBV
C. HCV
D. Gastritis
Answer: A
Note : acute presentation and jaundice its Hepatitis A

439. Turner syndrome chromosomes defects is ?


A. 45xo
B. 47XXX
C. 46XY
D. 47XXY

Answer: A
Note : the defects in Turner syndrome is ( deletion ) of X , so its deletion

440. Child with fever, malaise, Lymph nodes enlargement & mouth ulcers. What is the diagnosis ?
A. HSV2
B. HSV1
C. EBV
D. CMV

Answer: B

441. 10 years old girl presented with fatigability, diarrhea and glossitis. What is the diagnosis ?
A. Vit.B6
B. Vit.B3
C. Vit.B1
D. Vit. B12

Answer: D

442. 6 years old boy presented with gingivitis, petechiae and rash. What is the diagnosis ?( very very common Q )
A. Vitamin A deficiency
B. Vitamin B12 deficiency
C. Vitamin A deficiency
D. Vitamin d deficiency

Answer: A

443. 15 years old has DM with dehydration signs , what is ur DX ?


A. DKA
B. hyperosmolar acidosis
C. Lactic acidosis
D. hypoglycemia

Answer: A

444. A boy with rickets ( picture was provided X- RAY of Lower Limb ). What is the deficient vitamin?
A. Vit.D
B. Vit,C
C. Vit.B12
D. Vit.A

Answer: A

445. Which of the following diseases has mendelian mode of inheritance?


A. Alpha thalassemia

Answer: not clear . they answered it A


Note : Autosomal recessive

446. Child presented with bronchiolitis. What is your management?


A. Give oxygen
B. Antibiotics
C. Steroid
D. Slabutamol

Answer: A
Note : as we said the Treatment of bronchiolitis is supportive ( O2 , fluid ) if not in choices choose ( Ribavirin )

447. Mode of inheritance of neurofibromatosis ?


A. Autosomal recessive
B. Autosomal dominant
C. X linked recessive
D. X linked dominant

Answer : B
Note : ffeatures of NF : axillary frecking , café late spots

448. 7 years old boy developed flu after receiving flu vaccine. His father asked you about the reason. How will
you reply?
A. its bacterial indection and need ABX
B. Live attenuated vaccine has small risk of infection
C. its due to toxicity of vaccine
D. the baby has immunedeficiency

Answer: B

449. Boy presented with unilateral nasal obstruction and foul smelling. What is the diagnosis ?
A. Polyps
B. Foreign body
C. Rhinitis
D. Fibroma

Answer : B
Note : age is very important. Infant+ unilateral nasal obstruction + foul smelling = foreign body. Progressive
nasal obstruction concerning for growing mass. Anosomia + sinusitis= nasal polyps

450 . What is the inheritance mode of fanconi anaemia ( they will give u features not dx , short stature ,
anemia and café latte spots these important hints ) ? ( very common Q )
A. Autosomal dominant
B. Autosomal recessive
C. X-Linked recessive
D. X- Linked dominant

Answer: B

451. Bilateral parotid swelling , what is your dx :


A. Parotid tumor
B. Parotid caliculi
C. Mumps
D. Parotitis

Answer : C

452 . Child presented with recurrent nasal congestion, rhinorrhea, sneezing, tearing eyes . what is the
diagnosis?
A. Allergic conjunctivitis
B. Allergic rhinitis
C. Dermatitis
D. Ashma

Answer: B
Note : the hallmarks of allergic rhinitis are clear: thin rhinorrhea; nasal congestion; paroxysms of sneezing; and
pruritus of the eyes, nose, ears, and palate. Postnasal drip may result in frequent attempts to clear the throat,
nocturnal cough, and hoarseness. It is important to correlate the onset, duration, and severity of symptoms with
seasonal or perennial exposures, changes in the home or school environment, and exposure to nonspecific irritants,
such as tobacco smoke.

453. Rheumatic fever prophylaxis is ?


A. IM Ceftriaxone monthly
B. Oral Rifampcin
C. IM Penicillin monthly
D. IM Penicillin weekly

Answer: C
Note : An injection of 0.6-1.2 million units of benzathine penicillin g intramuscularly every 4 weeks is the
recommended regimen for secondary prophylaxis for most us patients. Administer the same dosage every 3 weeks
in areas where rheumatic fever is endemic, in patients with residual carditis, and in high-risk patients.
Management of acute rheumatic fever consists of benzathine penicillin to eradicate the beta-hemolytic streptococcus,
anti-inflammatory therapy with salicylates, and bed rest. Additional supportive therapy for heart failure or chorea may
be necessary. Long-term penicillin prophylaxis, preferably with intramuscular benzathine penicillin g, 1.2 million u
every 28 days, is required. Oral regimens for prophylaxis generally are not as effective. The prognosis of acute
rheumatic fever depends on the degree of
permanent cardiac damage.
454. ( long scenario ) baby with cavernous hemangioma and have pleural effusion. ( he have other findings).
What you will find in this baby? ( not clear Q )
A.Pulmonary hemangioma
B. Liver hemangioma
C. something
D. something

Answer: B ( not sure if this choice presents ) check it more

455. A child is always using abnormal sitting habits (W-setting), what is the effect on the bones?
A. Tibial eversion
B. Tibial anteversion
C. Femoral anteversion
D. Femoral eversion

Answer : C
Note : Internal femoral torsion (femoral anteversion), it's the most common cause of intoeing > 2 years of age.
Management: observation, takes 1-3 years to resolve. Surgery only if significant at > 10 years of age.

456. What is the gene responsible for neurofibromatosis 1 ( may be give u sx : axillary freckling and café latte
) ( very very common Q ) ?
A. chromosome 11 q11
B. chromosome 17 q 11
C. chromosome 22 q 12
D. chromosome 21 q 21

Answer : B
Note : NF1 long arm q of chromosome 17 , chr17q11.2 , NF2 long arm q of chromosome 22, chr22 q12.2
Both automsomal dominant , NF2 the problem in hearing

457. A case of newborn with ambiguous genitalia. Which hormone you would like to check?

No choices , most likely it is case of Congenital Adrenal Hyperplasia ( READ THE NOTE )
Note : 17-hydroxyprogesterone (hormone) should be measured promptly in all infants with nonpalpable gonads
presenting with genital ambiguity to exclude congenital adrenal hyperplasia (cah) due to 21-hydroxylase deficiency.
This is the most common cause of genital ambiguity and can lead to life-threatening adrenal insufficiency within the
first weeks of life.

458. Epiglottitis case ( high fever , leaning forward tripoid position , toxic appearance , acute presentation in
ER ) what is your action ? ( very commn Q )
A. Refer to ENT to intubate
B. Give antibiotis
C. Epinephrine
D. Do clinical exam

Answer: A
Note : epiglottitis in ER should do intubation first to secure the airways , after intubation give ABx
459. Pediatric blunt trauma with duodenal coiled spring sign. What will you do ?
A. Endoscopy
B. Surgery
C. Gastric decompression and NPO
D. Hematectomy

Answer : C ( Supportive , case of duodenal hematoma )


Note : case of deudenal hematoma , In the pediatric age group, duodenal injury from a blunt abdominal trauma
resulting in an intramural hematoma is rare. This case illustrates the characteristic delayed presentation of a duodenal
hematoma following a blunt injury from a motor vehicle accident and the associated sequelae of delayed pancreatitis.
Total parenteral nutrition and gastric decompression provide an effective conservative treatment of the gastric outlet
obstruction associated with this injury. Surgical intervention is only reserved for those patients who continue to show
the clinical and radiological signs of complete high obstruction despite conservative management for three weeks.

460. Child with bad smell and tooth is good , what is your dx ?
A. Tonsillits
B. Tonsilithis ( Tonsil crept stones )
C. Dental caries
D. Tonsills abscess
Answer: B

461. baby with high blood pressure in upper limb and low blood pressure in lower limb , what is ur
dx ?
A. Renal artery stenosis
B. VSD
C. Coarctation of aorta
D. heart failure

Answer : C

462. Mother came to you that her child complains of spells for seconds , what is your dx and treatment ?
A. Tonic seizure
B. Clonic seizure
C. sleeping disorder
D. Absence seizure

Answer : D ( the treatment is Ethusuximide )


Note : absence (petit mal): usually only seen in children, unresponsive for 5-10 s with arrest of activity, staring,
blinking or eye-rolling, no post-ictal confusion; 3 hz spike and slow wave activity on EEG .

463. Young patient swallowed pins, what are you going to do ?


A. Endoscopy
B. Surgery
C. Serial x ray
D. serial CT

Answer : C
Note : if it was sharp and we afraid of injury in stomach we will do Endoscopy , if not sharp we follow up only
464. 2-year-old complains of papule on the foot , no itching , pink pale not respond for antifungal ,
what is the dx ?
A. Granuloma

Answer : not complete , may be Granuloma Annulare

465. Down syndrome have bilateral ventricular enlargement , what is the dx ?


A. ASD
B. AVSD
C. Coarctation of aorta
D. TOF

Answer: B
Note :Atrioventricular septal defects (avsds) . these are the most common in children with down syndrome.

466. 9 month child Takes all vaccines regularly, what vaccine


should be given now?
A. MMR
B. HBV
C. Measles and Meningococcal
D. OPV and IPV

Answer : C
Note : 9 months vaccines: measles, meningococcal conjugate
quadrivalent (mcv4) according to saudi national vaccination schedule

467. 6-month baby can't sit, hypotonia, crossed lower limb. Which vaccine should be modified ?
A. DTP to DTap
B. HBV
C. IPV to OPV
D. OPV to IPV

Answer D ( not sure 100% , if in choices no change for any vaccines may be it will be the answer )

468. Typical case of turner, what another findings you will find ?
A. Cardiovascular abnormality.
B. Limb anomaly
C. Liver hemangioma
D. Vertebra anomaly

Answer: A
Note :
Turner syndrome features :
a)characteristic facial appearance (low set mildly malformed ears,triangular face,flattened nasal bridge
epicanthal fold )
b)webbing of the neck with or without cystic hygroma
c)shield like chest with widened internipple distance
d)internal malformations may include congenital heart defect (coarctation of the aorta is most
common anomaly followed by bicuspid aortic valve
,post stenotic aortic dilation with aneurysm may develop )and renal anomaly (horseshoe kidney )
e)short stature is cardinal feature
f)hypothyroidism
reference :essential nelson of pediatrics

469. Autosomal recessive disease both parents are carrier and phenotype normal , what the chance
they have a kid with a disease ?
A. 25
B. 50
C. 75
D. 100

Answer: A

470. A baby with bilateral renal agenesis , what is the finding ?


A. polyhydramnios
B. oligohydramnios
C. Kidney cyst
D. Large for gestational age

Answer: B
Note : with bilateral renal agenesis: (oligohydramnios which is a sign for the disease during prenatal diagnosis
step
And impossible for life

471. Child with high fever for 2 weeks and abdominal distention and weight loss , what is next ?
A. BM biopsy

Answer: A , We have to know more details in case and choices , but A could be the answer

472. Baby was playing with his father watch, suddenly his father looks to the watch which not
working , baby become agitated and refuse food what you will do ?
A. X ray
B. upper endoscopy
C. CT scan
D. Nasal exam

Answer: B ( starts with Airway secure first if not in choices , and foreign body slodge in esophagus
the answer is B )

473. Baby wave his hand(bye bye) which


developmental milestone age?
A. 7 months
B. 9 months
C. 12 months
D. 15 months

Answer : B

474. 4 years old brought by his parents, height < 5th percentile, they ask if he will remain short. What
you will do initially ?
A. GH level
B. Bone age
C. parental height
D. Glucose level

Answer : C ( first assessment , to know average height , may be familial )

475. 8 years old girl presented with fever, numerous bruises over the entire body and pain in both
legs. Physical examination reveals pallor and ecchymosis and petechiae on the face,trunk and
extremities. Findings on complete blood count includes a haemoglobin of 6.3 g/dl, white cell count
of 2800/mm3 and platelet count of 29,000/mm3. Which of the following would be the most
appropriate treatment ?

Answer :dx most likely , acute lymphoblastic leukemia…. Treatment : chemotherapy , read more about it.
476. child presented with diaper rash with sattalie lesion he was given local creams and steroid but
didn’t work , what give ?
A. Local antifungal
B. Steroid again
C. antibiotics
D. oral steroid

Answer: A
Note : An antifungal treatment may be prescribed if the child is diagnosed with a yeast infection. This type of
treatment is not recommended without consulting the child's provider first. Antifungal treatments are available
as a cream, ointment, or powder. The treatment is usually applied two or three times per day, beneath a skin
ointment or paste, until the rash is gone.

477. What is the most common cause of facial cellulitis in pediatrics ?


A. Staphylococcus aureus
B . H. influenza
C. Group A strep
D. Strep pneumonia

Answer : C

478. Which of the following is a feature of tetralogy of fallot?( as ew said very common Q )
A. Pulmonary stenosis
B. Aortic stenosis
C. ASD
D. left ventricle hypertrophy

Answer: A
Note : teratology of fallot consists of 4 pillers:
Pulmonary stenosis
Vsd
Over-riding of the aorta
Right ventricular hypertrophy

479. Child came to the clinic with his mother was having ball, the doctor asked him to throw the ball
to him , he through it to the doctor and he went away to take the ball back
what is the developmental milestone of this child ?
A. 15 months
B. 24 months
C. 36 months
D. 48 months

Answer : B

480. Best diagnosis tool for thalassemia is ?


A. Blood smear
B. Genetic study
C. CBC
D. Electrophoresis
Answer: D

481. Diagnosis of Infective Endocarditis by ?

Answer : no choices , depends on duke criteria


The duke diagnostic criteria
A definitive clinical diagnosis can be made based on the following:

2 major criteria
1 major criterion and 3 minor criteria
5 minor criteria

Major blood culture criteria for ie include the following:

Two blood cultures positive for organisms typically found in patients with ie
Blood cultures persistently positive for one of these organisms, from cultures drawn more than 12 hours
apart
Three or more separate blood cultures drawn at least 1 hour apart

Major echocardiographic criteria include the following:

Echocardiogram positive for ie, documented by an oscillating intracardiac mass on a valve or on


supporting structures, in the path of regurgitant jets, or on implanted material, in the absence of an
alternative anatomic explanation
Myocardial abscess
Development of partial dehiscence of a prosthetic valve
New-onset valvular regurgitation

Minor criteria for ie include the following:

Predisposing heart condition or intravenous drug use


Fever of 38°c (100.4°f) or higher
Vascular phenomenon, including major arterial emboli, septic pulmonary infarcts, mycotic aneurysm,
intracranial hemorrhage, conjunctival hemorrhage, or janeway lesions
Immunologic phenomenon such as glomerulonephritis, osler nodes, roth spots, and rheumatoid factor
Positive blood culture results not meeting major criteria or serologic evidence of active infection with an
organism consistent with ie
Echocardiogram results consistent with ie but not meeting major

echocardiographic

482. Child with Egg allergy, what vaccine should avoid? ( very common Q )

Answer: influenza and yellow and Q fever vaccines

483. Baby with diarrhea and he should take vaccine, what u will do?
A. Delay vaccine
B. Give all vaccines
C. delayed DTP
D. give only killed vaccines

Answer: B ( not sure 100% )


Note : In general can be safely administered to children with mild illness, with or without fever (eg, upper
respiratory infection, otitis media, and diarrhea)

484. Which babies should not get rotavirus vaccine or should wait?

Answer :
Contraindications: severe allergic reaction after prev dose , hx of intussusception , severe combined
immune deficiency

485. Boy return from africa and got meningitis. What is the Cause ?
A. strep . pneumonia
B. Meninigococcal
C. H. influenza
D. Viral
Answer : B ( not sure 100% )
Note : The largest burden of meningococcal disease occurs in an area of sub-saharan africa known as
the meningitis belt, which stretches from senegal in the west to ethiopia in the east.

486. Pic of child of history support the diagnosis of henoch schonlein purpura ( abdominal pain , joint
pain , rash and hematuria sometimes . Treatment ?
Supportive therapy.

Note : The vast majority of patients with henoch-schönlein purpura (hsp), also called immunoglobulin a
vasculitis (igav), recover spontaneously. Thus, care is primarily supportive and includes adequate hydration,
rest, and symptomatic relief of pain.

487. 6 month infant turns bluish when feeding. Auscultation shows holosystolic murmur with single
s2. Diagnosis ?
A. TGA
B. VSD
C. TOF
D. ASD

Answer : C
Note : cyanosis go with TOF , also the murmur is murmur present in VSD which is part of TOF , so TOF the
answer , TOF : most common cyanotic heart disease in children, cyanosis usually absent at birth but develop
over first 2 year of life, infant are often asymptomatic until 4-6 month
Auscultation: single s2 with systolic murmur on upper and left sternal border
Tga: most common cyanotic heart in infant cyanosis usually within few hours after birth.
Auscultation: loud single s2. They may not be a murmur if no svd present

488. Case of boy with greasy stool no matter what he eat ; recurrent chest infection; sweat test +ve
(not sure if the test was mentioned in the case ) what is your dx ?
A. asthma
B. Cystic fibrosis
C. immotile cilia
D. Sarcoidosis

Answer : B
Note : CF : clinical features in infants and children:
-meconium ileus (20%) - recurrent respiratory symptoms (45%) -failure to thrive (28%)
others: sinus disease, pancreatic disease: steatorrhea (insufficiency), obstruction of ilium, rectal
prolapse, infertility, musculoskeletal, recurrent dvt, and nephrolithiasis
Diagnostic criteria — both of the following criteria must be met to diagnose cystic fibrosis (cf)
●clinical symptoms consistent with cf in at least one organ system, and
●evidence of cystic fibrosis transmembrane conductance regulator (cftr) dysfunction (any of the following):
•elevated sweat chloride ≥60 mmol/l (on two occasions)
•presence of two disease-causing mutations in cftr, one from each
parental allele •abnormal nasal potential difference

489. Child drink cow milk his hemoglobin low and mcv low which type of anemia he will has ?
A. Iron deficiency anemia
B. Vit. b12 def anemia
C. Folate def anemia
D. Anemia of chronic disease

Answer: A
Note :Iron deficiency anemia in infants and children common causes are
related to nutrition: e.g: -introduction of unmodified cow's milk (non-formula
cow's milk) before 12 months of age -occult blood loss secondary to cow's
milk protein-induced colitis

490. Child has itching and all students in his class got the same infection ?
A. tinea corporos
B. staphylococcus aureus
C. tinea Pedis
D. Sarcoptes scabie

Answer: D
Note :Disease: scabies. Organism: mite sarcoptes scabiei ( if itching betwwen toes its pedis , if in scalp its
tinea capitis , in all body that’s go with scabies , notes about scabies :
Highly contagious. The prominent clinical feature of scabies is itching, it is often severe and usually worse at
night. The pruritus is the result of a delayed type-iv hypersensitivity reaction to the mite, mite feces, and mite
eggs. To diagnose it you need:
-widespread itching that is worse at night
-a pruritic eruption with characteristic lesions and distribution
-other household/ school with similar symptoms (contact)
-diagnosis of scabies can be confirmed by microscopic visualization of the mite, eggs, larvae, or feces in
scrapings of papules or burrows examined under oil immersion.
Organism is mite called sarcoptes scabiei, transmitted by close person to
person contact (uptodate)

491. Child who has repeated infections in chronic granulomatous disease , the microorganism is :
A. strep.pneumonia
B. Legionela
C. Staph.aureus
D. Nisseria.meningitides

Answer : C
Note : in general, the organisms that infect patients with cgd are catalase producing. More
with catalase positive. (mnemonic: space: staphylococcus aureus, pseudomonas,
aspergillus, candida and enterobacter) (uptodate)

492. Long Scenario, single palmar crease large tongue , murmur , hypotonia , flat nose ...etc , what's
the diagnosis?
A. down syndrome
B. Prader willi syndrome
C. Fraxile x syndrome
D. Cystic fibrosis

Answer : A

493. Chronic granulmatous disease child , what is the defect ?


A. defect in catalase enzyme
B. defect in galactose reductase
C. defective phagocyte NADPH oxidase
D. defect in CD4

Answer : C
Note : Chronic granulomatous disease (CGD) is a rare (∼1:250,000 births) disease caused by
mutations in any one of the five components of the nicotinamide adenine dinucleotide phosphate
(nadph) oxidase in phagocytes. This enzyme generates superoxide and is essential for intracellular
killing of pathogens by phagocytes.

494. Most important cause of cerebral palsy is ?


A. Asphyxia
B. intrauterine infection
C. IUGR
D. prematurity

Answer: A if ask about the cuase , if ask about most important risk factor its : Prematurity ( check
more )

495. 11 years child previously normal, presented with cyanosis, echo showed ventricular
hypertrophy i cant remember rt or left, what is the diagnosis :
A. Vsd
B. ASD
C. TOF
D. Pulmonary atresia

Answer: it is important to know which ventricle because the


diagnosis will change RVH : fallots tetralogy but in this case
not TOF due to the age , it goes with VSD I know VSD a
cyanotic but reverse happens of shunt from right to left and
cyanosis .
LVH : tricuspid atresia , pulmonary atresia
According to the patient age it most likely fallots tetralogy

496. Parents had a child with cleft palate and they are asking about recurrence rate of cleft
palate in their children ?
A. 1%
B. 4%
C. 9%
D. 12%

Answer: B ( if 2 babies had cleft palate the risk of 3rd is 9-10%


Note : While if the Q was about spina bifida the chance to have another baby is 1%

497. case About description of scarlet fever rash ?


Answer: no choices , read the notes :
24-48 h after pharyngitis , rash begins in the groin ,axillae , neck ,antecubital fossa:pastias lines may be
accentuated in flexural area *within 24 h , sandpaper rash becomes generalized with perioral sparing , non-
pruritic , non-painful,blanchable. *rash fades after 3-4d may be followed by desquamation

498. Infantile colic character : ( no choices , read the notes )


Note : Episodes of crying for more than three hours a day, for more than three days a week for a three-week
duration in an otherwise healthy child between the ages of two weeks and four months,happen in the evening
and for no obvious reason , associated [2] symptoms may include legs pulled up to the stomach, a flushed face,
clenched hands, and a wrinkled brow. the cry is often high pitched (piercing)

Colic is commonly described as a behavioral syndrome in neonates and infants that is characterized by
excessive, paroxysmal crying. Colic is most likely to occur in the evenings, and it occurs without any
identifiable cause. Compared with regular crying, colicky crying is more turbulent or dysphonic and has a
higher pitch.

499. Picture of 1 month baby with raised and scale what is the cause ?
( depends on pic , if in forehead may be seborrheic dermatitis , if in cheek may be Eczema )

500. Girl with rt knee swelling aspirations results show elevated wbcs and what best management ?

Answer :
If juvenile idiopathic arthritis ( pauciarticullar ) girl> boy ,ass with increase risk of iridocyclitis .it will present
with fever,nodules ,erythematous rashes ,pericarditis ,fatigue
Lab : positive rf in 15% .ana may be positive , increase esr ,wbc ,platelet
Tx : NSAID if not in choices aspirin is the first lines. Methotrexate is second line
If the results of joint aspiration indicated septic arthritis treat with long course of iv antibiotics and
joint aspiration and sometimes surgical drainage if joint is deep like hip or the resolution is not rapid.
501. Child her mother has hep. B surface antigene positive, after 3 months he becomes positive, he
received bcg vaccine, what can give him now?

502. Milestones , baby healthy run to the doctor , play a role model as his father ,can’t complete a
sentence , can’t eat with spoon , age ?
A. 12 M
B. 18 M
C. 24 M
D. 36 M

Answer : B

503. Preventing child from drinking before bed, encourage to go to toilet before bed, all these
measures to help child with ?
A. Enuresis
B. Hematuria
C. UTI
D. diabete insipidus

Answer : A

504. Child above 90th percentile in height, long scenario and cardiac abnormality with fatigue…etc ,
what is the dx ?
A. Ehler danlos
B. Marfan syndrome
C. Prader willi syndrome
D. storage disease

Answer : B

505. Child is doing fine, his brother died while heading to work. What we should investigate for ? (
Common Q )
A. D.M
B. vessel malformation
C. Hypertrophic cardiomyopathy
D. Restrictive cardiomyopathy

Answer: C

506. A child with hydrocephalus progressively increasing in between the 3 rd and 4th ventricle - which
area is blocked ?
A. Interventricular foramin
B. Aqueduct of sylvius
C. median aperture
D. Luschka

Answer: B
Note : one of the most common causes of hydrocephalus is "aqueductal stenosis." in this case,
hydrocephalus results from a narrowing of the aqueduct of sylvius, a small passage between the third and
fourth ventricles in the middle of the brain , between lateral venricle and third ventricle : Interventricular
foramin

507. Cystic fibrosis mode of inheritance ? ( common Q )


A. autosomal recessive
B. autosomal dominant
C. X linked reccive
D. X liked dominant

Answer : A

508 . Vaccine of hepatitis A missed , second dose what to do?


Answer: have it as soon as possible but you don't need to start with the first dose again
Source: family medicine consultant.

509. Boy with hypopigmented lesion in back and extremity becomes lighter with sun exposure , what
is the tt ?
A. Antibiotics
B. steroid
C. Antifungal
D. Zinc

Answer : C
Note : A fungal infction called pityriasis versicolor. The organism is called malassezia furfur. A common,
benign, superficial cutaneous fungal infection usually characterized by hypopigmented or hyperpigmented
macules and patches on the chest and the back. And it usually recur. Treated with topical antifungal

510. Child with symptoms. On examination, there's strong pulse in the upper limbs and absence in
lower limbs. What’s the diagnosis?
A. Coarctation of aorta
B. Hypertrophy cardiomyopathy
C. VSD
D. Cardiac tamponade

Answer: A
Note : classical physical examination finding is a systolic bp that is higher in the upper extremities.
The difference in bp in right and left can indicate the point of coarctation.
511. Child is given fresh frozen plasma and heparin , what's the dx ?
A. hemorrhagic shock
B. DIC
C. ITP
D. HUS

Answer : B
Note : in hemorrhage: replacement of hemostatic elements with platelet transfusion, frozen plasma,
cryoprecipitate ƒ maintain platelets >50,000/mm3 and hemoglobin >80 g/l ƒ 4-5 units of ffp if inr >1.5 or aptt
>38 ƒ 10 units of cryoprecipitate if fibrinogen <100 mg/dl ƒ 1 adult dose of buffy-coat platelets if <10,000
(<20,000 if febrile, <50,000 before invasive procedure)
in thrombotic phase: ufh or lmwh in critically ill, non-bleeding patients

FFP : depletion of multiple coagulation factors (e.g. Sepsis, dic, dilution, ttp/hus, liver disease),
emergency reversal of life-threatening bleeding secondary to warfarin overdose

512. In developing country to prevent dental caries add to water ?


A. Floride
B. Iodine
C. Iron
D. Calcium

Answer: A

513. Baby with oral thrush and ask about diagnosis ?


A. Oral candidiasis
B. Leukoplakia
C. squamous carcinoma
D. abscess

Answer : A , if the thrush or plaque disappear with scratching its ( candidiasis ) if not disappear its (
Leukoplakia )

514. Child present with fever and sore throat which of the following suggest viral cause ? ( common
Q)
A. Rhinorrhea
B. headache
C. lymphadenopathy
D. something

Answer : A
515. Newborn after 2 weeks c/o bilateral conjunctivitis ,chest infiltration , lung crepitation, what is organism ?
A. Chlamydia
B. Gonorrhea
C. Strep.pneumonia
D. Adeno virus

Answer : A
Note : chlamydia trachomatis is the most common cause of sexually transmitted genital infections in the united states.
Infants born vaginally to infected mothers with genital disease are at risk for acquiring c. Trachomatis, which usually
presents as conjunctivitis and/or pneumonia.

516. Preventable cause of gastroenteritis by vaccine ?


A. Hib
B. Measles
C. Diphtheria
D. Rota virus

Answer : D

517. Child with decrease urine output , tea colored urine , generalized swelling next investigation ?
A. CBC
B. Urine analysis
C. Antibody study
D. Kidney biopsy

Answer : B

518. 7 days old baby presented with vomiting and fever ,culture shows catalase [negative] ,gram positive in
chain ,beta hemolytic .mother had hx of infection before delivery .the tt ?
A. Vancomycin
B. Ciprofloxacin
C. sulfoxazole
D. Ampicillin

Answer: D ( its group B strep GBS ) , see diagram of gram + bacteria

519. Which enzyme is deficient in phenylketonuria ?


A. Phenylalanine carboxylase
B. Phenylalaninie oxidase
C. Phenylalanine hydroxylase
D. Alanine oxidase

Answer : C

520. A young child presented with tonsillar ulcer and painful lesion in the back of her mouth and soft palate ,
what the dx ?
A. hand mouth foot disease
B. EBV
C. Herpangia
D. Rubella
Answer : C
Note : caused by coxsackie A , An acute febrile illness associated with small vesicular ( vesicle ) or ulcerative
lesions on the posterior oropharyngeal structures.herpangina is one of many manifestations of enterovirus
infection. Patients present with: fever(may be the first apparent symptom), sore throat and pain upon swallowing ,
headache or backache.
520. Child with fever. Rash in trunk, greyish macule in moth , what is the dx ?
A. Rubella
B. Measles
C. Sixth disease
D. Smallpox

Answer : B ( kopliks in mouth , this is the hint )


Note : the manifestations of the 3-day prodromal period are cough, coryza, conjunctivitis, and the pathognomonic
koplik spots (gray-white, sand grain-sized dots on the buccal mucosa opposite the lower molars

521. Rota virus how to confirm the diagnosis ?


A. stool culture
B. Stool antigen
C. CBC
D. Stool cyst

Answer : B

522. Case of ( young or kid) bilateral knee pain then rash starts on legs , thighs and buttocks , what is the dx
?
A. ITP
B. TTP
C. HSP
D. Juvenile Rheumatoid arthritis

Answer : C

523. Baby presented with abdominal bloating and constipation , investigation shows
increase ca+ ? if ask about the cause , its due to high calcium

524. 15 yo female , no period still , on examination slight breast buds with wide spaced areola , fine pubic hair on
labia majora , if ask about what u will tell the parent ? its normal wait till 16 years

525. Patient with a family history of multiple sudden cardiac death. This patient has marfan's syndrome
features. What might be the cause of death ?
A.Ruptured aortic aneurysm
B. Hypertrophy cardiomyopathy
C. HTN crisis
D. Pulmonary embolism

Answer: A
Note : most worst complication in Marfan ( Aortic aneurysm ) which lead to rupture
526. Newborn with meningitis, what is the most common organism ?
A. E.coli
B. GBS
C. Strep.pneumonia
D. Nisseria. Meningitides

Answer : B , group B strep is the most common followed by E. Coli followed by Listeria.

527. Mature Pincer grasp at which age ?


A. 7 months
B. 9 months
C. 12 months
D. 15 months

Answer : C , pincer grasp starts at 8 months but mature at 12

528. Child eat many tablets of paracetamol , which of the following increase risk of poisoning and liver
damage ? ( common Q
A. increase Glutathione
B. decrease Glutathione
C. level of oxidation
D. no risk on liver

Answer : B , Glutathion depletion main cause of liver damage due to Paracetamol poisoning

529. Child with vasoocclusive crises , last week now came for hepatitis A vaccine , what to do ?
Answer : give him the vaccine

530. Premature newborn come with abdominal distension and air on bowel wall , ur dx ?
A. Intussception
B. Necrotizing entrocolitis
C. Duedenal atresia
D. Pyloric stenosis

Answer : B , the air on bowel is


Pneumatosis intestinalis
531. Child or infant come with abdominal mass compressing collecting system, lung mets , ur dx ?
A. Neuroblastoma
B. Wilms
C. Hepatoblastoma
D. Multiple myeloma
Answer : B ( Nephroblastoma ) compress sollecting system and mets to the lung so its B

532. 2 year old baby only says mama and baba. His brother had history of speech delay until 3 years
old , what is next ?
A. Deveopmental assessment
B. Hearing assessment
C. Speech therapy
D. something

Answer : B

533. Child with swelling inside the leg then developed skin manifestations. What is the side effect of
medication ?

Answer : may be warfarin skin necrosis

534. Child with labored breath what to do? No choices

Answer may be : DKA or respiratory failure

535. Child with blue dot in testis and painful mass in inguinal area ?
A. Testicular torsion
B. Testicular appeandage torsion
C. Testicular tumor
D. Inguinal hernia

Answer : B ( blue dots the hint )

536. Neonate continuously crying , ultrasound showed doughnut sign, how to manage ? case of
intussception

Answer : after stabilization , reduction by hydrostatic ( enema )

537. Child with renal disease, his family are afraid that he become like his father on dialysis. He is
also known to have sensorineural hearing loss , what is the disease ?

A. Wegner granulomatosis
B. Neuroma
C. Neurofibromatosis 2
D. Alports syndrome

Answer : D
538. Patient with cough and vomiting after cough, fever for 3 weeks, what is your dx ?
A. Diphtheria
B. Pertusis
C. Epiglottits
D. Croup

Answer : B

539. case of child with guillain-barre syndrome had viral gastroenteritis 3 weeks ago,
asked about prognosis ?
A. Complete recovery
B. residual weakness
C. Something
D. something
Answer : B ( not sure 100% about other choices )

540. Von will brand disease , what is the deficiet in it ?


A. Factor 7
B. Factor 8
C. Factor 9
D. Factor 10

Answer : B

541. BCL BCR gene associated with what ? ( common Q )


A. ALL
B. CLL
C. CML
D. AML

Answer : C . Chronic myelogenous leukemia (CML)

542. Child with facial swelling and hypoalbuminemia, high cholesterol , what is the dx ?
A. Nephritic syndrome
B. Nephrotic syndrome
C. UTI
D. HSP

Answer : B

543. Newborn presented with vomiting and enlarged clitoris, lab was included showing
hypekalemia and hyponatremia. What is the most likely diagnosis ?

Answer : Congenital Adrenal Hyperplsia

544. Parent came for consuling. Most common mendilne inheritance ?

Answer: thalassemia ( no choices )


Note : The alpha and beta thalassaemias are the most common inherited single-gene disorders in
the world with the highest prevalence in areas where malaria was or still is endemic.

545. Adolescent (12-14) with band like headache. He is stressed in school , what is the dx ?
A. Migraine headache
B. tension headache
C. Cluster headache
D. Chronic daily headache

Answer : B

546. Multiple fractures in child and blue sclera , ur dx ?


A. Galactossemia
B. Osteogenesis imperfecta
C. Phenylkenonuria
D. Ehler danlos

Answer : B
Note :Osteogenesis imperfecta, (oi) is a group of genetic disorders that mainly affect the bones. The term
"osteogenesis imperfecta" means imperfect bone formation. People with this condition have bones that break
easily, often from mild trauma or with no apparent cause. Multiple fractures are common, and in severe
cases, can occur even before birth. Milder cases may involve only a few fractures over a person's lifetime.

547. 1 Month old with wide anterior fontanel, large protruded tongue, ur diagnosis ?
A. Congenital adrenal hyperplasia
B. Congenital hypothyroidism
C. Phenylkenonuria
D. Rickets

Answer : B

548. Child with red bulging tympanic membrane, ear pushed down and forward, what is the dx ?
A. Otitis media
B. Otitis externa
C. Mastoiditis
D. Otitis media with effusion

Answer : C

549. Child with generalized swelling , long scenario of nephrotic syndrome , you are
suspecting minimal change nephropathy, what you will find in the biopsy ?

Answer : in light microscop : normal biopsy , in Electron : effacement of basement membrane

550. Long case about child 2 months with respiratory symptoms , his brother dies from pneumonia
and her sister does not have any medical hx . Investigations : Show T cell 0 low all Ig low , ur dx ?

Answer : may be severe combined immunodeficiency

551. Child with sickle cell, what is the lifelong treatment to prevent infections ?

Answer : penicillin and immunization.

552. School age child who has bad performance in school, the teacher noticed he has
difficulty reading & often rearranges the letters to read , what is the dx ?
A. Dyslexia
B. Language delayed
C. ADHD
D. Mutism

Answer : A
Note : Dyslexia is a common learning difficulty that can cause problems with reading, writing and
spelling. It's a "specific learning difficulty", which means it causes problems with certain abilities used
for learning, such as reading and writing. Unlike a learning disability, intelligence isn't affected.

553. patient Have male child affected with CF, they don’t have it nor their 17 yo female,
possibility of next child ? ( very very common )
A. 1:2
B. 2:4
C. 25:1
D. 1:8

Answer : may be C , Not sure

554. case about (adult came with hematuria and convusion , change in mental status with
thrombocytopenia after infection ) ur Dx ?
A. ITP
B. TTP
C. HUS
D. HSP

Answer : B

555. steroid sensitive age ?( common Q )


A. 6-24 Month
B. 6- 12 Month
C. 96-105 Month
D. 24-96 Month

Answer : D
Note : same age of minimal change disease presentation

556. pic of lymphangitis ( red streaks , see google image ), ask abou treatment ? ( common Q )
A. IV. Antibiotics
B. Oral antibiotics
C. NSAID
D. Steroid

Answer : B
Note : other Q give you the pic or description ( red streaks ) and ask about tt
Lymphangitis caused by : GABH and S.aureus so the tt by : Cloxacillin or Cephalexin

557. child his age 3 years his mother notice that his child become more active and chewing his
toys ,his lab test :normal MCV ,basophils high ,hemoglobin low ,and there is target cell in
peripheral smear?
A. Sideroblatic anemia
B. Thalassemia
C. Iron deficiency anemia
D. G6PD deficiency anemia

Answer : C
Note : if Iron deficiency not in choices choose B , IDA comes sometimes with normal MCV and Target
cell

558. 58 days baby have G6PD his iron level is low and slightly high bilirubin , what is the cause ?
A. Anemia of chronic disese
B. Hemolytic anemia
C. Iron deficiency anemia
D. Mgaloblatic anemia

Answer : B ( G6PD deficiency its hemolytic anemia may lead to IDA due to hemolysis )
559. child is 13 months old complete his vaccination except MMR because itsnot available, but now
the MMR vaccination is available what will you do ?
A. Give him next time
B. Give him now
C. no need MMR vaccine
D. start new doses

Answer : B ( not sure 100% )

560. severe attack of asthma what is the best tool to assessment the severity in ER ?
A. presence of wheeze
B. O2 saturation
C. FEV1
D. use of accessory muscle

Answer : C
Note : FEV1 and Peak expiratory flow rate are the best to assess the severity in ER for baby more
than 5 years .

561. child came to ER in seizure, with hx of ingestion of 16 pill of isonized what is ur


mangment ?
A. Niacin
B. Pyrodixine
C. Thiamine
D. Deferoxamine

Answer : B
Note : Antidote for Isoniazide is Vit.B6 ( Pyrodixine ) so admit the patient to ICU and give him
Pyrodixine

562. you diagnose a child with measles, the rash started 2 days ago, she goes to a daycare 5
days a week. How many sick leave days will u give her ?
A. 1 day
B. 3 days
C. 5 days
D. 6 days

Answer : B
Note : contagious period 3 – 5 days after rash , rash started 2 days ago so remain 3 days

563. child takes sulfa drug then comes with drug reaction , describe skin lesions due to drug
? ( very common Q )
A. nodule and papule
B. vesicle and ulceration
C. Cystic lesions and vesicle and papule
D. ulceration and blisters
Answer : D
Note : choose the choice with Blister always and ulcer

564. which of the following urine features help in preventing UTI ?


A. high ph , high urea , high osmolarity
B . low ph , low urea , low osmolarity
C. low ph , low urea , high osmolarity
D. high ph , low urea , high osmolarity
Answer : D

565. pt with recurrent UTI , the doctor describe Nitrofurantoin as prophylaxis what is the duration ?
A. 50 to 100 mg for 1 months
B. 50 to 100 mg for 3 months
C.50 to 100 mg for 6 months
D.50 to 100 mg for 12 months

Answer : D
Note : its between 6 to 12 months , but many sources said 12 months
Other Q about duration of UTI treatment by Nitrofurantoin the answer : 100 mg for 7 days

566. overdose of digixin symptoms are ?

Answer : At toxic concentrations, digoxin causes ectopic ventricular beats that may result in VT and fibrillation
, hypokalemia
The antidote for Digoxin is : Digfab or immune fab

567. scenario about malaria ( back from sudan with sx of Malaria : cough , malaise , arthralagia and
myalagia usually after weeks of infection , , ask about diagnostic test ?
A. PCR
B. Blood culture
C. Blood film with Giemsa stain
C. Stool culture

Answer : C

568. heberden node definition is ?


A. hard or bony swellings that can develop in the distal interphalangeal joints (DIP)
B. hard or bony swellings that can develop in the Proximal interphalangeal joints (PIP)
C. hard or bony swellings that can develop in the Metatarsal
D. hard or bony swellings that can develop in the Metacarpal

Answer : A
Note if ask about ( bouchard node : hard swelling that can develop in proximal interpharyngeal joints ( PIP ) )
Both signs of Osteoarthritis

569. case of ( newborn with bilious vomiting , abdominal distension in first day , double bouble sign ,
dx ?
A. Midgut malrotation
B. Duedenal atresia
C. Pyloric stenosis
D. Hischsprung disease

Answer : B
Note : age of presentation is important duodenal atresia usually within 2 days of birth , also double bouble sign
in abdominal x ray , pyloric stenosis presents with non bilious vomiting and at 2 weeks usually , midgut
presents usually at 3rd or 4th day

570. 13 month baby his mother came complains that he is not like other brother and relatives and she
thinks the baby has delayed in milestone , what milestone the mother thinks her baby delayed ? ( very
common Q )
A. Crawling
B. walk alone
C. wave bye bye
D. pencir grasp
Answer : B ( not sure 100% , check choices in exam )

571. child abuse in hymen at with site is common to perforate ? ( very very common Q )
A. 4th oclock
B. 6th oclock
C. 8th oclock
D. 10th oclock

Answer : B

572. 45 female delivered baby with sign palmer creases and epicantheal fold , what is the
chromosomal abnormality ?
A. Trisomy 22
B. Trisomy 19
C. Trisomy 21
D. Trisomy 18

Answer : C
Note : its dowon syndrome , risk of down icrease with mother age increase

573. sickle cell disease , what is the type of mutation ?


A. deletion
B. shifting
C. doubling
D. point

Answer : D ( point mutation or missense)


Note : mutation in B globin in chromosome 11 , Glutamine replaced by Valine

574. bleeding after circumcision due to ? ( common Q )


A.Vwb
B. hwmophilia A
C. ITP
D. Platelets dysfunction

Answer : B ( factor 8 defeciency )


Note : its clotting factor disease , its X linked , Hemophilia B deficiency of factor 9 and less common

575. baby smiles at which age ?


A. 8 Weeks
B. 12 Weeks
C. 16 Weeks
D. 18 weeks

Answer : A
576. Child can’t walk or stand without help he has mild neck stiffness. He had chickenpox 3
weeks ago , what is the dx?
A. GBS
B. Acute cerebral ataxia
C. French ataxia
D. Meningoencephalopathy

Answer : B
Note : its complication of chickenpox

577. Kallman syndrome , how to establish diagnosis ?


A. CT scan
B. Gonadotropin test
C. TSH level
D. Brain MRI

Answer : B
Note : Kallman usually diagnosed at age of Puberty , its genetic disorder og Hypogadotrpic
Hypogonadism due to lack of hypothalamic GnRH , lead to low FSH , LH and then Estrogen ,
Progesteron , Testesterone , other characteristics lack of smell

578. long scenario of child who is aggressive and acting out and his mother Brought to check
up on him. (Labs: He has anemia with normal iron & TIBC) and his peripheral blood smear had
Anisocytosis, Poikilocytosis and basophiliic stippling and a lot of other findings. They also
provided Bone marrow findings I think it was erythroid hyperplasia , what is ur dx ?
A. Iron deficiency anemia
B. Thalassemia
C. Sideoblastic anemia
D. Heomlytic anemia

Answer : C
Note : this is typical for sideoblastic , in bone marrow we find Erythroblast ( ring sideroblast ) , also we
find Iron normal or high and also normal TIBC , the most common cause of sideroblastic is excessive
alcohol use .

579. child came with his mother 1 week after his 18 months vaccination asks you for hepatitis A
vaccine (As it wasn’t available last week) when to give Hepatitis A ?
A. Now
B. 1 week later
C. At age 2 years
D. After 1 months

Answer : A
Note : HAV Vaccines give at age 18 months and 2 nd dose at age 24 months in KSA Vaccination
programme , if first dose delayed should given as soon as possible also The second dose should be given
as soon as possible if miss , the minmum period between two doses is 6 months .

580. minimum age a child can get influnza vaccine is ?


A. 2 months
B. 4 months
C. 6 months
D. 8 months

Answer : C

581. child with episodes of cyanosis in lip and peripheries , precordial murmur , What is best to
reach a diagnosis ?
A. X ray
B. MRI
C. Echo
D. CT scan

Answer : C
Note : its Tetralogy of Fallot , the best to dx is : Echocardiogram

582. child 2 yrs who is did repetitive pattern behavior , and don’t communicate with family ,
what is ur dx ?
A. ADHD
B. Oppositional defiecent syndrome
C. Mutism
D. Autism
Answer : D
Note : Autism diagnosed typically before 3 years , repetitive behavior is characteristic , ADHD usually
diagnosed after age of 6

583. A child who is aggressive behavior with his mother and other
Children , what is ur dx ?
A. ADHD
B. Autism
C. Oppositional defiecent syndrome
D. Mutism

Answer : C

584. 8 yrs old female presented with secondary sexual development ( pubic hair , breast budding
), what to tell her mother ?
A. Precious puberty
B. Premature adrenarche
C. Normal developmental
D. Congenital adrenal hyperplasia

Answer : C
Note : normal puberty in male starts at age 9 and above , in female at age 8 and above , before these
ages consider precious Puberty , and if only pubic hair appear before these ages consider prccious
( premature ) adrenarche

585. which infection can transmit through palcenta causing deafness and cardiac abnormalities
?
A. Measles
B. Rubella
C. CMV
D. Toxoplasmosis

Answer : B ( congenital Rubella )

586. what is the recommendation of varicella vaccine doses and duration in pediatrics and
adults ?
( very common Q )
A. Pediatrics 2 doses 3 months apart , adult 2 doses 3 months apart
B. Pediatrics 2 doses 3 months apart , adult 2 doses 2 weeks apart
C. Pediatrics 2 doses 3 months apart , adult 1 dose
D. Pediatrics 2 doses 3 months apart , adult 2 doses 4 weeks apart

Answer : D
Note : The second dose if given should occur at least three months in pediatrics , For people older than
13 the two doses are administered 4 to 8 weeks apart, minimum age for varicella vaccine : 12
months

587. elderly with 4-month history of of tiredness, hepatospleenmegaly with high leukocytes
What is most likely dx ?
A. Leukmoid reaction
B. ALL
C. AML
D. CML

Answer : D ( not sure 100% if B and C in choices )


Note : CML usually asymptomatic or mild symptoms and median age 64 , while ALL fatal and rapid
progressive , AML has more symptoms and younger .
588. what is the usual age presentation of intussception ?
A. 1 Month
B. 2 Month
C. 6 Month
D. 12 month

Answer : C
Note : Intussusception is most common in infants aged 3-12 months , so choose the nearest number to this .

589.newborn Early after delivery develops jundice + postive indirect coombs test , ur dx ?
A. ABO incompatability
B. G6PD def
C. Criglar najjar
D. Sepsis

Answer : A

590. 3 year old boy with hx of recurrent abdmonial pain and constipation and
hematuria , come with painless abdmonial mass in left side, on examination
there's tenderness in the flank with absent light reflex in eye , ur dx ?
A. Neuroblastoma
B. Nephroblastoma
C. Osteosarcoma
D. Lung Ca

Answer : B
Note : wilms ( Nephroblastoma ) pt came with abdominal mass ( doesn’t cross midline ) hematuria , also (
aniridia : absence of Iris ) , common mets to the lung ,
( Neuroblastoma ) cross midline and came with horner syndrome

591. case of baby with steatorrhea diarrhea , flat buttocks , what investigation to do ? ( common Q )
A. Anti TTG Antibody
B. Anti basement membrane antibody
C. stool culture
D. immunoassay

Answer : A
Note : its Celiac disease , first antibody study ( anti TTG antibody ) and other antibodies u have to know them
, but if ask about the best for diagnosis it ( intestinal biopsy ) so if it was in choices choose it .

592. Baby cry when bath, doesn't have any favorite toy, doesn't share toys
with kids , what u will tell his mother ?
A. Normal
B. Autism
C. ADHD
D. Develpmental delay

Answer A

593. non symmetrical painful joints in early morning in a child , what is


the dx ?
A. Juvenial rheumatoid arthritis
B. Osteoarthritis
C. Septic arthritis
D. Psoriatic arthritis

Answer : A ( u have to read the scenario well )


594. newborn with past Hx of conjunctivitis after delivery with treatment wIth
doxycycline now with white patches on dorsal aspect of tongue , what is the
treatment ?
A. Ketoconazole
B. Miconazole
C. Topical Nystatin
D. Doxycyclin

Answer : C
Note : its case of oral thrush ( candidiasis ) , nystatin is Antifungal drug used for oral thrush , diaper rash ,
esophageal candidiasis and vaginal yeast infection

595. Time between HAV vaccine is ?


A. 3 months
B. 6 months
C. 8 months
D. 10 months

Answer : B

596. Pic of child with umblical swelling with yellow discharge ? ( very very common Q )
A. omphalomesentric cyst
B. umbilical granuloma
C. Urachal cyst
D. Gastroschisis

Answer : B ( sure 100% )

597. Patient presented to the ER after dog bite (it was deep ) he already receive 1 rabies vaccine 1
month ago what to do ?
( common Q )
A. Give immunoglobulin and vaccine of rabies
B. Give 2 doses of rabies vaccine
C. reassurance and discharge
D. Give immunoglobulin only

Answer : B ( Consultant , not sure )

598. TTP case ( hematuria and bleeding from other site , change in LOC , seizure , low platelets ) what
is the tt ?
A. FFP
B. FFP and plasma exchange
C. Hparin
D. Platelets transfusion

Answer : B ( tt of choice )
Note : other common Q , give u scenario of TTP and ask about pathogenesis of disease its due to
(ADAMTS13)

599. which cancer can treat with vaccine ?


A. B cell leukemia
B. Fungoid tumor
C. ALL
D. T cell Leukemia

Answer : D

600. Sickle cell diseased pediatric pt comes with painful penis with pic , ur dx ?
A. Paraphimosis
B. Phimosis
C. Priapism
D. Orchitis

Answer : C

601. 4 y/o had bone marrow transplantation 18 months ago ,, his brother developed fever and
vesicular rash , what will u give the 4 years old as a prophylaxis?
A. IV Acyclovir
B. Varicella immunoglobulin
C. Varicella vaccine
D. Both B&C

Answer : B
Note : Varicella vaccine is live attenuated so its contraindicated for immunocompromised pt

602. Child started to have headache nausea fever one week ago now came with edema grade
2 in lower limps and preorbital she was very pale (palm,skin.mucous) brown colored urine ,
With hypertension , tachycardia , fever , tachypnea The most important diagnosis u will be
concerned about ? ( common Q )
A. diagnosed SCA
B. recurrent UTI
C. abdominal trauma one week ago
D. Impetigo something one month ago

Answer : D ( not sure 100% , may be post.strep GN after impetigo caused by GABH )
603. Retinoblastoma affect the vision , MRI show intact optic N (something like that) what is the tt
?
A. Chemoreduction
B. Steroid
C. Radioablation
D. Enucleation

Answer : A ( main tt )

604. pediatric newborn with purulent eye discharge , Culture showed gram negative diplococci ,
How to treat ?
( very common Q )
A. Topical steroid
B. Topical abx
C. IV Cephalosporin
D. IV Doxycycline

Answer : C
Note : case of conjunctivitis casued by . Nisseria Gonorrhea , usually come in first 3 days of birth with
purulent discharge while conjunctivitis due to chlamydia usually after 5 days with mucopurulent
discharge , important to know :
G+ diplococci : Strep.Pneumonia
G- diplococci : Nisseria Gonorrhea and meningitides
G- coccobacilli : H.influenza , Brucella , Bortedella , chlamydia

605. Contraindications for circumcision is ? ( very common Q )


A. Hypospadias
B. Pyelonephritis
C. PCKD
D. Prematurity

Answer : A

606. ADHD child , can't involve for anything for long time , like in school or even when he
playing a video games he can't finish it and searching for something else to do and so on, ( I
think also don’t listen ) which type of ADHD he has?
A. Hyperactive
B. Inattension
C. Impulsive
D. Aggressive

Answer : B ( read the case well in exam to make sure , it may be A )


Note : common Q what is the criteria of ICD-10 for dx of ADHD for pediatrics the answer is :
1. At least six symptoms of attention for at least 6 months
2. At least three symptoms of hyperactivity for at least 6 months
3. At least one symptom of impulsivity

607. long case about baby always tired since birth with ( lactic acidosis and hyperalalninemia )
what is enzyme deficiency ? ( Very Very common Q )
A. Pyruvate carboxylase
B. Pyruvate dehydrogenase
C. Pyruvate reductase
D. Pyruvate lactase

Answer : B
Note : if said lactic acidosis only the answer also Pyruvate dehydrogenase , if said Lactic acidosis and
HyperAmmonemia the Answer will be : Pyruvate Carboxylase , this is important Q .

608. 17 years old athelet with weight gain and normal puberty and bad mouth smell , this is
due to ?
A. Anabolic steroid
B. normal
C. cushing syndrome
D. Pheochromocytoma

Answer : A
Note : athlete + weight gain + bad smell all go with using of anabolic steroid

609. A 9 month old asthmatic child on corticosteroids which vaccine to give?


A. HAV
B. OPV
C. Influenza vaccine
D. MMR
Answer : C , also we give for asthmatic pt Pnemococcal vaccine

610. A child who’s always alone, doesn’t own any toys and doesn’t play with others.
What is the relation cause or something ?
A. Intellegence
B. Interpersonally
C. Something
D. something

Answer : B ( not clear at all they answer it this answer )

611. A child came with symptoms of meningitis, and culture showed gram --‐ve diplococci, his
brother had contact with him. What to do for his brother?
A. Nothing
B. Rifampcin
C. Vaccine
D. isolation of brother

Answer : B
Note : this is ( N.meningitides we give chemoprophyaxis for close contact )

612. Newborn has tetany after delivery. Calcium: 1 (normal 2--‐3). One year
later, he has recurrent upper respiratory infection with: HiB, HSV, Pneumocystis
jirovecii, aspergillus. And heart murmur , What is his underlying condition?
A 22q11.2 deletion
B.Depletion of cd4 lymphocytes
C. Wiskott Aldrich syndrome
D. NADPH
oxidation
Answer: A
Note : this is case of ( Digeorge syndrome characterized by : hypocalcemia , congenital heart
disease and t cell defieciency ( abscent of thymus )

613. Scenario of a child with meningitis. CSF findings are as follows: High cellular
count and high WBC , normal glucose and high protein. What’s the diagnosis?
A. Bacterial meningitis
B. Viral meningitis
C. Tuberculosis meningitis
D. Fungal meningitis

Answer : C
Note : you have to differniate between all types of meningitis from CSF result

614. A 14 year old boy presented to the clinic to take his second dose of
varicella vaccine (His 1st one was taken a year ago). Which of the following is
the correct management?
A. Give the second dose
B. Check his varicella titer then give the second dose
C. Start from the beginning and give first
D. something

Answer : A

615. Child with frequent vomiting and GERD. What’s the pathophysiology?
A. high LES pressure
B. low LES pressure
C. high gastric pressure
D. low gastric pressure

Answer : B

616. In a female, the spine stops growing after the onset of menarche by how many months?
A. 6 months
B. 12 months
C. 24 months
D. 48 months

Answer : C

617. 13 year old boy with delayed puberty. His growth parameters are normal and
he has fair hair in the groin. How will you manage him?
A. free testosterone
B. wait
C. testicular US
D. give him steroid

Answer : B

618. Aspirin used in kids after viral infection , may lead to ?


A. Meningitis
B. Reyes syndrome
C. Septic shock
D. Tonsillitis

Answer : B
Note : aspirin not use for baby less than 12 years may cause Reyes syndrome ( fulminant
hepatitis and cerebral edema )

619. A child with episodes of cyanosis in lips and peripheries + precordial murmur.
What is best to reach a diagnosis?
A. CXR
B. CT
C. ECHO
D. US

Answer : C
Note : the best investigation for any congenital heart disease is ECHO

620. A child with diarrhea, abdominal bloating, diagnosed with giardiasis. What is
the most sensitive test to diagnose his case? ( very common Q )
A. 3 consecutive stool analysis
B. 3 seperated stool analysis
C. Immune assay
D. stool cyst

Answer : B
Note : the bank Q of SCFHS is old so we depends more on old answer sometimes

621. A child, known case of thalassemia, had fever after blood transfusion. What is
the best measure to take for prevention of fever? ( Very common Q )
A. Antipyretic before transfusion
B. Pre transfusion medications
C. Leukocyte depleted ( prior storage )
D. Leukocyte filters at
transfusion

Answer : C

622. Vaccine that is contraindicated in eczema , psoriasiss and contact dermatitis ? ( very
common Q )
A. MMR
B. Varicella
C. IPV
D. Smallpox

Answer : D

623. Pediatric patient with enuresis. Which method is affective for long time?
A. Voiding before sleep
B. Decrease fluid intake
C. Alarm
D. ADH

Answer : C ( The best way )

624. 3 year old baby have a watery discharge from his eyes since birth and redness
what is the cause?
A. Bacterial conjunctivitis
B. Lacrimal duct obstruction
C. Congenital glaucoma
D. congenital cataract

Answer : B

625. NICU child suddenly developed distress with absent breath sounds. Where will
you place the butterfly needle ?
A. 2nd intercostal space
B. 3RD intercostal space
C. 4TH intercostal space
D. 5TH intercostal space

Answer : A
Note : needle decompression in 2 nd intercosral space midclavicular

626. 6 month old infant brought by his parents with history of repeated vomiting; his
pulse was (190). He had dry mucous membranes and sunken anterior fontanel. What
is the appropriate volume of fluid that should be given initially?
A. Bolus 10 ml/kg of body weight
B. Slow infusion 10 ml/kg of body weight
C. Bolus 20 ml/kg of body weight
D. Oral rehydration

Answer : C
Note : this is case of severe dehydration

627. Child unvaccinated developed fever, SOB and stridor. Lateral X--‐ray shows a
thumb sign. What’s the treatment?
A. Erythromycin
B. Vancomycin
C. Ceftriaxone
D. Metronidazole

Answer : C
This is case of Epiglotittis

628. 10 year old child brought by his parents because they were concerned about
his weight. He eats a lot of fast food and french fries. Your main concern in
managing this patient is?
A. His parents concern about his weight
B. His BMI > 33
C. Family history of heart disease
D. Eating habits (fast food and french fries)

Answer : C ( not sure 100% )

629. 4 month old girl that refuses feeding. She becomes cyanotic and cries
whenever she feeds. On auscultation harsh enection systolic murmur , What is the
diagnosis?
A. Aortic stenosis
B. VSD
C. ASD
D. TOF

Answer : D
Note : cyanotic heart disease and the harsh systolic murmur due to ( Pulmonary atresia ) which
is part of TOF

630. A 4 year old girl presenting with generalized malaise and bony aches
especially in the hand and feet. Her labs show high ANA and the ESR is high. What
is the cause?
A. Latent lupus
B. JRA
C. Growing pain
D. Something

Answer : B ( not sure 100% )

631. Pediatric case scenario about respiratory distress syndrome. Which of the
following is deficient?
( common Q )
A. Sphingomylin
B. Dipalmioylphosphatidylcholine
C. Cholesterol
D. Protein

Answer : B ( SURFACTANT )
632. A pediatric case. Asymptomatic patient with a picture of an ECG (I just noticed
irregular rhythm, bradycardia and PR interval was unequal some lead was 3 little
squares and another 5 little squares). I think it was a case of 1st degree heart block.
Then they asked about the consequences?
A. Syncope
B. Need pacemaker
C. Heart failure
D. Something

Answer : A ( if D was : no need to treat , choose it )

633. Child with rapid blinking of the eyes. Can communicate with his parents during
it. What is the diagnosis?
A. Autism
B. TIC
C. Tourette syndrome
D.Petit mal

Answer : B

634. A mother complains that her child who is 6 months old is crying too much for
the past 2 weeks. The mother said that she was regularly feeding her child then
recently introduced cow milk. Upon examination, there was abdominal distention.
What is the diagnosis?
A. Cow milk allegy
B. Infant colic
C. Galactosemia
D. Gastroenteritis

Answer : A

635. A Researcher wants to measure obesity in children. He took their BMI. What
else he should he take? ( very common and no clear answer )
A. Girth measurement
B. HDL/LDL Level
C. Dieatry habits
D. Skin folded

Answer : C ( not sure 100% )

636. An 18 month old child presented with leg bowing and inability to walk. His labs
show high ca, low phosphate, and high ALP. What’s the diagnosis?
A. Vit d deficiency
B. Familial hypophosphatemia
C. Hypophosphatemia
D. Vit.d dependant

Answer : B

637. Pediatric patient presented with a centripetal vesical rash distribution, more
on the trunk and gradually less towards extremities. What is the incubation period of
this disease ?
A. 0 to 10 days
B. 10 to 21 days
C. 22 to 30 days
D. 15 to 25 days

Answer : B
Note : this is Varicella and the important hint is ( vesicle )

638. A Pediatric patient is complaining of: Intermittent dysphagia, retrosternal pain


and heartburn. Barium swallow showed: dilated esophagus. Synchronous
contractions and gradual distal narrowing. What is the diagnosis?
A. Achalasia
B. Diffuse esophageal spasm
C. GERD
D. Esophageal ring

Answer : A

639. A pediatric patient complaining of daily fever , fatigue and knee swelling for two
months. Also, recently developed epistaxis . Labs: Low platelet, Low hemoglobin,
Normal WBC. What is your next step?
A. ANA
B. Bone marrow aspiration
C. ESR
D. Blood culture

Answer : B
Note : this is mostly leukemia

640. A pediatric patient with sore throat, migrating arthralgia and fever. What’s the
diagnosis?
A. Rheumatic fever
B. JRA
C. Septic arthritis
D. Kawazaki

Answer : A
Note : this is the criteria but the most important in criteria is : evedince of URTI ( tonsillitis
or any thing ) caused by ( GABH ) by throat culture or ASO level , here sore throat
indicate that .

641. A 15--‐year--‐old girl brought by her mother because she did not get her period
yet. On examination she has breast buds, normal pubic hair and her height has
increased during the last year. Which one of the following will support your diagnosis?
A. Bone age
B. onset of menstruation
C. hormonal level
D. Something
Answer : B ( not sure 100% )
Note : not clear , we don’t know what is the aim of Q , But maybe he wants us to know that’s
normal and we wait for onset of mense because in this case the girl normal , to say this is 2 nd
amenorrhea we wait till 16 years if 2nd sexual characterisric present like this case

642. Treatment of baby with iron deficiency anemia is ?


No options , if no gastritis of gastric problem we choose ( ferrous sulphate orally ) if there is
gastric problem choose ( IV if not in choices choose IM )

643. Child with repeated polymicrobial chest infections. Skin test was positive for
candida antigen and all blood test are normal except: high IgG or IgM? And low
lymphocytes.Lymph node biopsyshowed: rudimentary germinal centers. What is the
pathophysiology of this disease?
No options, may be X-linked agammaglobinemia ( Burton ) , check more

644. What is the treatment of typhoid fever in children?


A. Chloramphenicol
B. Ciprofloxacin
C. Penicillin
D. Vancomycin

Answer : A
Note : if say there is resistant to chloramphenicol what is the abx used : its Ceftriaxone

645. A 4 year old child with decreased in head growth, has weird hand movements
(wringing), lost expressive end receptive language skills and lost his interest in his
social environment. What is the diagnosis?
A. Mental retardation
B. ADHD
C. Rett syndrome
D. Autism

Answer : C

646. Child patient came with scenario of chest infection, first day of admission he
was treated with cefotaxime. On the next day, the patient’s state became bad with
decreased perfusion and x--‐ray shows complete right sided opacification +
hydrothorax. What is the causative organism?
A. Strep.pneumonia
B. H.Influenza
C. Pseudomonas
D. Staph.aureus

Answer : D ( not sure 100% , from pedia consultant )


Note : may be MRSA , anyway its between C and D

647.Long scenario of a pediatric patient with the following labs: K: 3, Na: 124. How
would you correct the electrolyte imbalance?
A. Normal saline and Kcl 5 over 20 cc/hr
B. Normal saline and Kcl 40 over 80 cc/hr
C. 1/2 saline and Kcl 5 over 20cc/hr
D. 1/2 saline and Kcl 5 over 20cc/hr

Answer : A ( The case not completed )

648. Long scenario of a boy with diarrhea (gastroenteritis and dehydration


symptoms) with no urine output (acute kidney injury). And they gave you labs (k:
high, urea: high, sodium bicarbonate: low and high creatinine). What is your
immediate action?
A. NaHco3
B. Dialysis
C. Normal saline
D. Antibiotics

Answer : C ( first step correct the cause )

649. A child with short stature and no family history. What’s the most likely
underlying etiology?
A. Constitutional
B. Hormonal
C. Genetics
D. Endocrine

Answer : A
Note : for sure not comleted Q But most common from this is : A

650. Foul smelling urine in a 7 year--‐old child with fever and lower abdominal pain.
What’s the causative organism?
A. E.coli
B. Proteus
C. Klebsiella
D. Pseudomonas

Answer : A ( Not sure 100% )


Note : maybe the Q not complete , the most common cause of UTI is : E.coli but the hint in Q
foul odor urine may indicate Urea splitting bacteria but both B and C are Urea splitting ( proteus
more common ) so I don’t know

651. Child who took 20 pills of aspirin. How are you going to manage him?
A. Deforxamine
B. Urine alkalizing
C. Dialysis
D. syrup

Answer : B
Note : this is case of Aspirin toxicity and the tt is ( Nahco3 ) alkalization of urine but initialy may use
Activated charcoal

652. A child with a honey colored crust rash. What’s the diagnosis ? ( very common Q )
A. HSV
B. Measles
C. Impetigo
D. Celulitis

Answer : C
Note : typical description of impetigo which is usually caused by S. aureus

653. Child born from a mother who has gram negative streptococcus positive at time
of delivery. After birth, child has retraction cyanosis and difficulty breathing. What’s
your next step in management?
A. Chest x ray
B. blood culture and look for infection
C. name of abx
D. name of abx

Answer : A ( not sure 100% )


Note : what I know ant newborn come with any sign of infection we have to rule out sepsis
and give empirical abx

653. Congenital adrenal hyperplasia test? ( no option )


We depend on 17 hydroxyprogesterone

654. 8 month old infant with meningitis. What is the causative organism in this age group?
A. listeria
B. S.pneumonia
C. N. menigitides
D. E.coli
Answer : B
Note : in first 3 months of life : GBS , E.Coli , Listeria then from 3 month till 10 years : s.pneumonia ,
after 10 years : N.meningitides

655. Baby born with 1 artery in umbilical cord?


A. Gestitinal DM
B. Something
Answer : A

656. A 3 year old came to ER with cough and SOB, rhinorrhea for three days. The
cough is worse at night. What is the most appropriate nebulizer medication?
A. racemic epinephrine
B. Salbutamol
C. Ipratropium bromide
D. theophylline

Answer : B
Note : this is case of Asthma exacerbation

657. Newborn with severe jaundice at birth. He was found to be O+ and his
mother is A--‐. What is the mechanism? ( very very common Q )
A. Fetal antibody agglutinating fetal antigen
B. Fetal antibody agglutinating fetal antibody
C. Mother antibody agglutinating fetal antigen
D. Mother antibody agglutanting fetal antibody

Answer ; C
Note : this is mechanism of RH isoimmunization

658. A mother brought her child to ED with itchy skin lesions that appeared after
intake of some food. The lesions appear in a certain area and stay for 2 hours then
disappear then appear in another area. What is it called?
A. Burrow
B. Wheal
C. Macule
D. Vesicle

Answer : B
Note : Wheal: A wheal is a rounded or flat topped, pale red papule or plaque that is
characteristically evanscent disappearing within 4 o 8 h ours.

659. A baby who was recently started on cow’s milk, presenting with wheezing and
SOB. Sputum test shows there is blood. Labs show: microcytic hypochromic
anemia. What’s the treatment?
A. Steroid
B. Cytotic drug
C. Chromolyin
D. Stop cow milk

Answer : D ( not sure of options )

660. A young patient was playing presented with severe epigastric pain, retching
and unable to vomit. Unable to pass NG tube. What is the diagnosis?
A. Volvolus
B. Hiatal hernia
C. Pyloric stenosis
D. esophageal atresia

Answer : B
Note : C and D presented in neonatal age

661. 12 year old presented with growth retardation and abdominal striae. What is
next?
A. Morning and night cortisol
B. Brain MRI

Answer : may be B , ( We have to know all details and choices )

662. Neonate with cyanosis and shortness of breath while feeding with diaphoresis.
Presented with ejection systolic murmur, 3/6, normal S1 loud S2, best heard on left
side of chest. What is the cause?
A. Large ASD
B. Large VSD
C. PDA
D. Mitral regurge

Answer : must be TOF or Pulmonary atresia

663. One year old baby with a prominent forehead. Hemoglobin electrophoresis
show: Hb A2 2--‐3% and Hb F 2%. What’s the diagnosis?
A. Alpha thalassemia
B. B thassemia minor
C. B thalassemia major
D. hydrop fetalis

Answer : B
664. A child presented with difficulty in breathing and mild fever. The parent said
that he had 2 similar episodes two weeks ago. What's your first step?
A. Observe
B. Fluid and supportive care
C. Inhaled steroid
D. Ventolatory support

Answer : B
Note : it depends on age if age between 6 to 24 months , this is mostly bronchiolitis and its
tt supportive with o2 and fluid

665. A scenario of a boy with recurrent infections. His brother died when he was 3
years old. He has 2 sisters who are alive and well. What does he have?
A. SCID
B. Hyperimmunoglobulinemia M
C. X Linked agammaglobulinemia
D. T cell deficiency
Answer : C
Note : the hint is all ( male ) affected while female not affected so x –linked

666. Child had URTI 2 weeks ago, presented with right upper quadrant pain. Labs
show high WBC, low hg and other labs not sure of them. Electrophoresis shows
inclusion bodies. What's the diagnosis?
A. GI infarction
B. Sickle cell anemia
C. Hereditary spherocytosis
D. Appendicitis

Answer : B

667. Child came with knee swelling or bleeding after mild trauma. Bleeding time was
abnormal and it was NOT corrected after we gave fresh frozen plasma. Then
bleeding time got back to normal after giving platelet transfusion (his platelet level
was 50). What does the patient have (PT and PTT were not included in the
question)?
A. VwF deficiency
B. ITP
C. TTP
D. HSP

Answer : A

668. New born totally healthy with left thigh bruise. All examination normal.
Prolonged pt and ptt. What’s the diagnosis?
A. Hemophilia A
B. Hemophilia b
C. Factor 10 def
D. ITP

Answer : C
Note : factor 10 present in common pathway so its deficiency affect both PT and PTT

669. A child with multiple plaques on his face, abdomen and feet. What is the Diagnosis? (In
the Q they mentioned something about histoimmune and positive CD ?
A. Mycosis fungoid
B. Bowen disease
C. Basal cell ca
D. Squamous cell ca

Answer : A ( not sure 100% )


Note : If the plaques are hypopigmented, with CD 3 positive, CD4 positive and CD8
negative this is mycosis fungoides.

670. Pediatric age group, mass on the inner side of lower lip, non--‐tender and bluish
in color. What is the diagnosis?
A. Ranula
B. Mucocele
C. Gingival cyst
D . Basal cell ca

Answer : B
671. A young girl came with leg pain more in night. Examination is normal. Labs:
ESR and ANA high. What’s the diagnosis?
A. JRA
B. Growing pain
C. osteosarcoma
D. Oseiod osteoma

Answer : C ( not sure 100 % , check the options and scenario in exam )
Note : if ESR and ANA normal the answer will be B , I Think its ALP not ANA

672. A baby has depigmentation since birth, photophobia and refractory error
nystagmus. What’s the most likely complication associated with his condition?
A. Skin cancer
B. Renal dysfunction
C. Brain tumor
D. Liver cirrhosis

Answer : A ( case of ocular albinism type 1 )

673. A child with cough, coryza and conjunctivitis. Presented with maculopapular
rash in palm and buttocks. On examination, there are pustules in tonsils. What other
physical signs will find in the patient?
A. spots in sole of foot
B. Diarrhea
C. Swollen tonsil
D. Scratch marks

Answer : A ( both A nd B are corrects )


Note : This is case of measles , if the Q about complication its B

674. Child with autoantibodies.. Chances of getting DM type 1?


A. 40%
B. 60%
C. 80%
D.100%

Answer : B ( nearst answer , not sure 100% )

675. Child with fever and productive cough and diarrhea 5 times a day. Now,
tolerating oral rehydration?
A. Hospitalization
B. Augmentin and discharge
C. Stool analysis
D. Something

Answer : D ,  ( if its oral rehydration )


Note : mostly viral gastroenteritis , no need for stool analysis , admission only if srvere dehydration , no need
for abx

676. Which of the following vaccines is contraindicated in a child with progressive


neurological problems?
A. Pneumococcal
B. Hib
C. Varicella
D. DTap
Answer : D

677. Child with multiple infections: viral, fungal and bacterial. He has a defect in Cd4?
A. chronic granulomatous disease
B. Adenosine deaminase deficiency
C. x- linked agammaglobulinemia
D. B cell deficiency

Answer : B ( not sure 100% )

678. Child with a history of fall without losing consciousness. He vomited twice and
is crying and complaining of a headache. What should you do?
A. CT scan
B. Observe
C. Discharge
D. ICU referral

Answer : A

679. Pediatric patient known case of acute lymphoblastic leukemia (ALL), presented
to the ER with fever and pancytopenia (lab results were provided). What is your
action?
A. Reffer to oncology
B. Blood transfusion
C. Give antibiotics
D. Radiology

Answer : C

680. Most common presentation of congenital heart disease?


A. Ascites
B. JVP
C. Murmur
D. cyanosis

Answer : C ( not sure if present in choices )

681. A 5 year old boy presents with tender, swollen and painful testicle. O/E there
was absent cremasteric reflex. Which of the following is the correct diagnosis?
A. Epydidemitis
B. Testicular torsion
C. Testicular appendage torsion
D. Testicular tumor

Answer : B

682. During delivery of a baby, there was stylomastoid foramen trauma. Which of the
following features will be evident when you examine this baby?
A. loss of eye close
B. Loss of face sensation
C. Loss of mastication
D. Loss of posterior 2/3 of tongue sensation

Answer : A
Note : bells Palsy ( facial palsy )

683. Clear case about crohn’s disease: a child with abdominal cramping, diarrhea.
Endoscopy shows skip lesions and transmural inflammation. What’s the diagnosis?
A. Ulcerative colitis
B. Crohns disease
C. Celiac
D. IBS

Answer : B
Note : transmural in crohns

684. A three year--‐old girl presented to Emergency Department with fever, vomiting
and abdominal pain which began 10 hours ago. Radiological examination confirmed
a dilated intestinal pouch attached to the anterior abdominal wall. Her diagnosis was
the persistence of a Meckel’s diverticulum which of the following sites will the
surgeon look for this diverticulum?
( very very common Q )
A. Lower deudenum
B. Lower jejenum
C. Lower ileum
D. Cecum

Answer : C

685. Which of these patients will most likely be diagnosed with rheumatic fever from
his symptoms?
A. Child with knee swelling and joint pain and sore throat
Answer : A , ( check other options )

686. A mother comes with her 9 year old daughter. She’s looking short and her
mother is also short. The bone age is 7 years and her labs: all normal except insulin
like growth hormone (below the normal range). What will you do?
A. reevaluate after 1 year
B. Give GH
C. Something
D. Something

Answer : B ( not sure 100% )


687. A patient with seizures on anti--‐epileptic treatment. What will u do regarding
his vaccinations?
A. Give all vaccines
B. Give all vaccines except DTaP
C. Stop all vaccines
D. Don’t give live vaccines
Answer : A

688. A baby fell from the stairs. He presented with multiple lacerations of different
degrees and a fracture. What’s your next step?
A. Refer to orthopedic
B. Refer to pediatric
C. Call social worker
D. Splinting
Answer : C
Note : different degree of fracture indicate abuse

689. 3--‐year--‐old child presenting with high fever for the last 2 days, vomiting, refusal
to eat and red ears. Which of the following will help you find the diagnosis?
A. Urine culture
B. CSF analysis
C. Stool analysis
D. Something

Answer : B ( Not sure 100% )


Note : may be meningitis , check option D

690. When to screen for congenital hypothyroidism?


A. At birth by cord
B. After 1 week
C. After 1 month
D. After 6 month

Answer : B
Note : the accurate answer is 2 to 4 days

691. Scenario: a patient developed bloody diarrhea, abdominal pain and vomiting
after eating from a restaurant. History of hematuria. Lab results show anemia. What
is the diagnosis?
A. HUS
B. ITP
C. TTP
D. HSP

Answer : A

692. 14y girl developed abdominal pain - mimcs appendicitis - after URTI what is
The diagnosis ?
A. Appendicitis
B. Mesentric adenitis
C. Gastroenteritis
D. Meckle diverticulosis

Answer : B

693. Child 6 y/o leg pain during night only .. No findings in examination?
A. Normal growing
B. Osteosarcoma
C. Osteoid osteoma
D. JRA

Answer : A ( Growing pain )

694. when to give anti d for baby as prophyaxis ?


A. Within 72 hrs after delivery
B. During delivery
C. After 7 days of birth
D. Last 2 weeks of pregnancy

Answer : A

695. Child presented to the ER with fever and abdominal pain. After a fall 1 day ago,
the mother noticed abdominal distention mainly on the right side. On examination
he is pale and hypertensive , your dx ?
A. Liver contusion
B. Pyelonephritis
C. Wilm's tumor
D. Neuroblastoma

Answer : D

696. 4 year old with hepatosplenomegaly, blue skin nodules and neck mass,
investigation?
A. MRI of the thorax
B. EBV serology
C. Plain xray
D. CT

Answer : B ( not sure 100%)

697. Child (9 months old?) with anemia despite iron supplements given and increased
dose multiple times. Labs shows: low Hb, low MCV, (no iron studies).
Peripheral smear attached (hypocytic hypo-chromic)
What to order?
A. Iron study
B. Hb electrophoresis
C. Vitb12 level
D. Something

Answer: B
Note : maybe thalassemia
698. Child with abdominal pain, vomiting, diarrhea, , fever, . where is the pathology?
A. Brunner cells
B. Payer cells
C. Something
D. Something

Answer : B
Note : I think this is Enteric fever

699. Child with fever, abdominal pain, diarrhea and vomiting. Gram stain showed G-ve
rods, oxidase -ve. What is the MOA of the given antibiotic?
A. inhibit DNA gyrase
B. inhibit 50s subunit
C. Something
D. Something

Answer : B ( we have to know other details )


Note : if inhibit 30s subunit choose it

700. 9 year-old with signs and symptoms of OM. For how long will you give abx in
days?
A. 3
B. 5
C. 7
D. 10

Answer : D

701. Picture of spoon shape nail , What is the dx ?


A. Thalassemia
B. IDA
C. Sickle cell anemia
D. G6PD def

Answer : B

702. Patient had hemophilia A , his wife is carrier , new baby will had hemophilia?
A. 25%
B. 50%
C. 75%
D. 100%

Answer : B

703. A child with difficulty swallowing, sore throat, high fever, drooling saliva, stomach
ache and vomiting. He is unable to eat or drink anything due to pain on
swallowing. Examination shows tonsils with exudate and discharge. Which of the
following will help the patient to swallow?
A. Aspirin
B. Paracetamol
C.Ibuprofen
D. Steroids

Answer : D
Note : this is case of infectious mononeocliosis

704. 8 month old boy starts crying and screaming when stranger comes near him
what is the cause?
A. simple phobia
B. stranger anxiety
C.seperation anxiety
D. Social phobia

Answer : C

705. child with facial edema when waking up with scrotal edema. On auscultation
there is a pansystolic murmur heard in the left lower sternal edge. Labs shows
anemia, low albumin, high cholesterol, high AST
What is the diagnosis?
A- heart failure
B- nephrotic syndrome
C- liver failure
D. something

Answer : B ( not sure 100% )

706. pediatric patient aggressive behavior hits his mother and destroys stuff,
symptoms started since 3 months after his parents divorced because of child
neglect , your dx ?

Answer : Reactive attachment

707. 6 year old child with a weight of 14 kg and a hight of 94 cm presented with a
history of bloody stool mixed with mucus. Stool microscopy did NOT show
parasite and ova and antigen for E. histolotica was negative. What is the likely
cause of her symptoms?
A. irritable bowel syndrome
B. giardiasis
C. Rota virus
D. ulcerative colitis

Answer : D

708. Scenario of a pediatric patient with typical symptoms of otitis media (painful
and otoscope showed bulging tympanic membrane of the affected ear...etc). The
patient had a history of impetigo infection a few weeks ago with was treated with
antibiotics. Which of the following drugs will you choose to treat this patient?

A. amoxicillin
B. ceftriaxone
C. another type of cephalosporins
D. erythromycin

Answer : A

709. Child with fever, headache and nuchal rigidity, CFS showed gram positive cocci.
Which of the following is the most likely finding?
A. Numerous neutrophils, decreased glucose and increased protein
B. Numerous neutrophils, increased glucose, normal protein
C. Numerous lymphocytes, normal glucose, decreased protein
D. Numerous lymphocytes, decreased glucose, normal protein

Answer: A

710. Most significant risk factor of cerebral palsy?


A. Prematurity
B. Perinatal asphyxia
C. Genetic abnormalities
D. Low birth weight

Answer: A ( A and B )

711. Management of hirschsprung disease?


A. Surgical resection
B. Dietary modification
C. Antibiotics
D. Immune suppressive medication

Answer: A

712. Infectivity period in Rubella before the after the rash in days?
A. Three
B. Five
C. Seven
D. Nine

Answer: B

713. 5 year old child, repeated episodes of jaundice and history of multiple blood
transfusion, normocytic hypochromic anemia, blood film showed anisocytosis,
spherocytosis, neuleated RBC and polychromasia, positive osmotic fragility test
and negative DAT. What is the cause?
A. G6PD deficiency
B. Antibody mediated destruction of the RBCs
C. Ankyrin and spectrin deficiency
D. Beta chain abnormality

Answer: C

714. Child with asthma treated with steroid for four weeks and stopped one week ago.
Which of the following vaccines can be given to him?
A. Measles
B. MMR
C. Rota virus vaccine
D. Varicella
Answer: D ( not sure 100% )

715. Treatment of child with absence seizure ?


A. Ethosuximide
B. Diazepam
C. Phenytoin
D. Phenobarbital

Answer : A

716. baby fall down , and he has bloody otorrhea , what is the cause ?
A. SAH
B. Epidural hematoma
C. Cerebral infarction
D. Basal skull fracture

Answer : D

717. patient with long standing fever and dx with Kawasaki fever which of the
folllowing strongly associated ?
A. Jaundice
B. Strawberry tongue
C. Diarrhea
D. Seizure

Answer : B

718. What is the Tonsillectomy indication ?


A. Retropharyngeal abscess
B. Asymptomatic large tonsil
C. obstructive sleep apnea
D. 3 infections per year

Answer : C ( Absolute )
Reference: ministry of health

Das könnte Ihnen auch gefallen